You are on page 1of 177

INTEGRATION FOR CALCULUS,

ANALYSIS, AND DIFFERENTIAL


EQUATIONS Techniques, Examples, and Exercises

11035_9789813272033_TP.indd 1 28/6/18 11:36 AM


b2530   International Strategic Relations and China’s National Security: World at the Crossroads

This page intentionally left blank

b2530_FM.indd 6 01-Sep-16 11:03:06 AM


INTEGRATION FOR CALCULUS,
ANALYSIS, AND DIFFERENTIAL
EQUATIONS Techniques, Examples, and Exercises

Marat V Markin
California State University, Fresno, USA

World Scientific
NEW JERSEY • LONDON • SINGAPORE • BEIJING • SHANGHAI • HONG KONG • TAIPEI • CHENNAI • TOKYO

11035_9789813272033_TP.indd 2 28/6/18 11:36 AM


Published by
World Scientific Publishing Co. Pte. Ltd.
5 Toh Tuck Link, Singapore 596224
USA office: 27 Warren Street, Suite 401-402, Hackensack, NJ 07601
UK office: 57 Shelton Street, Covent Garden, London WC2H 9HE

Library of Congress Cataloging-in-Publication Data


Names: Markin, Marat V., author.
Title: Integration for calculus, analysis, and differential equations : techniques, examples,
and exercises / by Marat V. Markin (California State University, Fresno, USA).
Description: New Jersey : World Scientific, 2018. | Includes bibliographical references and index.
Identifiers: LCCN 2018026833 | ISBN 9789813272033 (hardcover : alk. paper) |
ISBN 9789813275157 (pbk : alk. paper)
Subjects: LCSH: Calculus--Textbooks. | Mathematical analysis--Textbooks. |
Differential equations--Textbooks.
Classification: LCC QA303.2 .M368 2018 | DDC 515--dc23
LC record available at https://lccn.loc.gov/2018026833

British Library Cataloguing-in-Publication Data


A catalogue record for this book is available from the British Library.

Copyright © 2019 by World Scientific Publishing Co. Pte. Ltd.


All rights reserved. This book, or parts thereof, may not be reproduced in any form or by any means,
electronic or mechanical, including photocopying, recording or any information storage and retrieval
system now known or to be invented, without written permission from the publisher.

For photocopying of material in this volume, please pay a copying fee through the Copyright Clearance
Center, Inc., 222 Rosewood Drive, Danvers, MA 01923, USA. In this case permission to photocopy
is not required from the publisher.

For any available supplementary material, please visit


https://www.worldscientific.com/worldscibooks/10.1142/11035#t=suppl

Printed in Singapore

LaiFun - 11035 - Integration for Calculus.indd 1 28-06-18 12:13:00 PM


May 17, 2018 12:7 ws-book9x6 Integration for Calculus, Analysis, and Differential Equations ws-book9x6 page v

To my mother, Svetlana A. Markina, fondly.

v
b2530   International Strategic Relations and China’s National Security: World at the Crossroads

This page intentionally left blank

b2530_FM.indd 6 01-Sep-16 11:03:06 AM


June 21, 2018 10:23 ws-book9x6 Integration for Calculus, Analysis, and Differential Equations ws-book9x6 page vii

Preface

The calculus is the story this


world first told itself as it
became the modern world.

David Berlinski

Amply demonstrated by experience, integral calculus covered in Calculus I


and, mostly, in Calculus II appears to represent a serious challenge for
many students. Passing rates in these courses are often considered to be
indicative for the future graduation rates.
The main purpose of this book is to assist calculus students to gain
a better understanding and command of integration and its applications
and, thus, improving their performance in Calculus I and II courses. Its
writing stems out of my extensive experience of teaching calculus or its
equivalent to diverse groups of students at the California State University,
Fresno, Boston University, the University of North Carolina, Asheville, and
the National University of Food Technologies, Kiev, Ukraine.
The usefulness of the book as a concise and, at the same time, rather
comprehensive review of integration reaches beyond the scope of the fore-
going courses to students in more advanced courses such as Multivari-
able Calculus, Differential Equations, and Intermediate Analysis, where the
ability to effectively integrate is essential for their success, and also those,
who prepare for integration competitions such as the Fresno State Integra-
tion Bee.
Keeping the reader constantly focused on the three principal episte-
mological questions: What for? Why? How?, the book is designated as
a supplementary instructional tool treating the three kinds of integral:
indefinite, definite, and improper and covering various aspects of integral

vii
June 21, 2018 15:58 ws-book9x6 Integration for Calculus, Analysis, and Differential Equations ws-book9x6 page viii

viii Integration for Calculus, Analysis, and Differential Equations

calculus from abstract definitions and theorems (with complete proof when-
ever appropriate) through various integration techniques to applications.
It contains 143 Examples, including 112 Problems with complete step-by-
step solutions, the same problem occasionally solved in more than one way
while encouraging the reader to find the most efficient integration path,
6 Exercises, 162 Practice Problems, and 30 Mixed Integration Problems
“for dessert”, where the reader is expected to independently choose and
implement the best possible integration approach. The answers to all the
192 Problems are provided in the Answer Key. Three Appendices furnish a
table of basic integrals, reduction formulas, and basic identities of algebra
and trigonometry.
The book’s writing was supported by a Fresno State College of Science
and Mathematics Scholarly and Creative Activity Award 2015/16, for which
I would like to express my cordial gratitude.
My utmost appreciation goes to Dr. Maria Nogin (Department of Math-
ematics, CSU, Fresno) for her numerous invaluable contributions into im-
proving the manuscript and to Mr. Andres Zumba Quezada (CSU, Fresno),
the winner of the Fresno State Integration Bee 2015 and 2017, for pain-
stakingly reading the manuscript, solving every single problem in it, and
providing helpful suggestions. I am also very grateful to Dr. Przemyslaw
Kajetanowicz (Department of Mathematics, CSU, Fresno) for his kind
assistance with the figures.
My sincere acknowledgments are also due to the following associates
of World Scientific Publishing Co. Pte. Ltd.: Ms. Rochelle Kronzek for
discerning a value in my manuscript and making the authors, in particular
this one, her high priority; Ms. Lai Fun Kwong for her astounding efficiency
and great editorial work, and to Mr. Rajesh Babu for his expert and very
helpful technical assistance.
The book, my first one, is dedicated to my mother, Svetlana A. Markina,
with affection and appreciation inexpressible with any words.

Marat V. Markin
May 17, 2018 12:7 ws-book9x6 Integration for Calculus, Analysis, and Differential Equations ws-book9x6 page ix

Contents

Preface vii

1. Indefinite and Definite Integrals 1


1.1 Antiderivatives and Indefinite Integral . . . . . . . . . . . 1
1.1.1 Definitions and Examples . . . . . . . . . . . . . . 1
1.1.2 Validation of Indefinite Integrals . . . . . . . . . . 4
1.1.3 Which Functions Are Integrable? . . . . . . . . . . 5
1.1.4 Properties of Indefinite Integral (Integration Rules) 5
1.2 Definite Integral . . . . . . . . . . . . . . . . . . . . . . . . 6
1.2.1 Definitions . . . . . . . . . . . . . . . . . . . . . . 6
1.2.2 Which Functions Are Integrable? . . . . . . . . . . 9
1.2.3 Properties of Definite Integral (Integration Rules) 9
1.2.4 Integration by Definition . . . . . . . . . . . . . . 10
1.2.5 Integral Mean Value Theorem . . . . . . . . . . . 11
1.2.6 Fundamental Theorem of Calculus . . . . . . . . . 12
1.2.7 Total Change Theorem . . . . . . . . . . . . . . . 15
1.2.8 Integrals of Even and Odd Functions . . . . . . . 16

2. Direct Integration 17
2.1 Table Integrals and Useful Integration Formula . . . . . . 17
2.2 What Is Direct Integration and How Does It Work? . . . . 20
2.2.1 By Integration Rules Only . . . . . . . . . . . . . 21
2.2.2 Multiplication/Division Before Integration . . . . 21
2.2.3 Applying Minor Adjustments . . . . . . . . . . . . 22
2.2.4 Using Identities . . . . . . . . . . . . . . . . . . . 23
2.2.5 Transforming Products into Sums . . . . . . . . . 26

ix
May 17, 2018 12:7 ws-book9x6 Integration for Calculus, Analysis, and Differential Equations ws-book9x6 page x

x Integration for Calculus, Analysis, and Differential Equations

2.2.6 Using Conjugate Radical Expressions . . . . . . . 27


2.2.7 Square Completion . . . . . . . . . . . . . . . . . 28
2.3 Direct Integration for Definite Integral . . . . . . . . . . . 29
2.4 Applications . . . . . . . . . . . . . . . . . . . . . . . . . . 31
2.5 Practice Problems . . . . . . . . . . . . . . . . . . . . . . 33

3. Method of Substitution 35
3.1 Substitution for Indefinite Integral . . . . . . . . . . . . . 35
3.1.1 What for? Why? How? . . . . . . . . . . . . . . . 35
3.1.2 Perfect Substitution . . . . . . . . . . . . . . . . . 36
3.1.3 Introducing a Missing Constant . . . . . . . . . . 37
3.1.4 Trivial Substitution . . . . . . . . . . . . . . . . . 39
3.1.5 More Than a Missing Constant . . . . . . . . . . . 40
3.1.6 More Than One Way . . . . . . . . . . . . . . . . 42
3.1.7 More Than One Substitution . . . . . . . . . . . . 43
3.2 Substitution for Definite Integral . . . . . . . . . . . . . . 45
3.2.1 What for? Why? How? . . . . . . . . . . . . . . . 45
3.3 Applications . . . . . . . . . . . . . . . . . . . . . . . . . . 48
3.4 Practice Problems . . . . . . . . . . . . . . . . . . . . . . 49

4. Method of Integration by Parts 51


4.1 Partial Integration for Indefinite Integral . . . . . . . . . . 51
4.1.1 What for? Why? How? . . . . . . . . . . . . . . . 51
4.1.2 Three Special Types of Integrals . . . . . . . . . . 52
4.1.3 Beyond Three Special Types . . . . . . . . . . . . 55
4.1.4 Reduction Formulas . . . . . . . . . . . . . . . . . 57
4.2 Partial Integration for Definite Integral . . . . . . . . . . . 59
4.2.1 What for? Why? How? . . . . . . . . . . . . . . . 59
4.3 Combining Substitution and Partial Integration . . . . . . 62
4.4 Applications . . . . . . . . . . . . . . . . . . . . . . . . . . 63
4.5 Practice Problems . . . . . . . . . . . . . . . . . . . . . . 64

5. Trigonometric Integrals 65
5.1 Direct Integration . . . . . . . . . . . . . . . . . . . . . . . 65
5.2 Using Integration Methods . . . . . . . . . . . . . . . . . . 66
5.2.1 Integration via Reduction Z Formulas . . . . . . . . 66
5.2.2 Integrals of the Form sinm x cosn x dx . . . . . . 70
May 17, 2018 12:7 ws-book9x6 Integration for Calculus, Analysis, and Differential Equations ws-book9x6 page xi

Contents xi

Z
5.2.3 Integrals of the Form tanm x secn x dx . . . . . 76
5.3 Applications . . . . . . . . . . . . . . . . . . . . . . . . . . 79
5.4 Practice Problems . . . . . . . . . . . . . . . . . . . . . . 81

6. Trigonometric Substitutions 83
6.1 Reverse Substitutions . . . . . . . . . . . . . . . . . . . . . 83
6.2 Integrals Containing a2 − x2 . . . . . . . . . . . . . . . . . 84
6.3 Integrals Containing x2 + a2 . . . . . . . . . . . . . . . . . 88
6.4 Integrals Containing x2 − a2 . . . . . . . . . . . . . . . . . 92
6.5 Applications . . . . . . . . . . . . . . . . . . . . . . . . . . 96
6.6 Practice Problems . . . . . . . . . . . . . . . . . . . . . . 98

7. Integration of Rational Functions 99


7.1 Rational Functions . . . . . . . . . . . . . . . . . . . . . . 99
7.2 Partial Fractions . . . . . . . . . . . . . . . . . . . . . . . 100
7.2.1 Integration of Type 1/Type 2 Partial Fractions . . 101
7.2.2 Integration of Type 3 Partial Fractions . . . . . . 101
7.2.3 Integration of Type 4 Partial Fractions . . . . . . 103
7.3 Partial Fraction Decomposition . . . . . . . . . . . . . . . 104
7.4 Partial Fraction Method . . . . . . . . . . . . . . . . . . . 107
7.5 Applications . . . . . . . . . . . . . . . . . . . . . . . . . . 112
7.6 Practice Problems . . . . . . . . . . . . . . . . . . . . . . 114

8. Rationalizing Substitutions 115


8.1 Integrals with Radicals . . . . . . . . . . . . . . !. . . . . . 115
Z r
ax + b
8.1.1 Integrals of the Form R x, n dx . . . 115
cx + d
8.1.2 Integrals
Z of the Form
 
R x, xm1 /n1 , . . . , xmk /nk dx . . . . . . . . . . 116
8.2 Integrals with Exponentials . . . . . . . . . . . . . . . . . 117
8.3 Trigonometric Integrals . . . .Z. . . . . . . . . . . . . . . . 118
8.3.1 Integrals of the Form R (tan x) dx . . . . . . . . 118
Z
8.3.2 Integrals of the Form R (sin x, cos x) dx . . . . . 119
8.4 Applications . . . . . . . . . . . . . . . . . . . . . . . . . . 121
8.5 Practice Problems . . . . . . . . . . . . . . . . . . . . . . 124
May 17, 2018 12:7 ws-book9x6 Integration for Calculus, Analysis, and Differential Equations ws-book9x6 page xii

xii Integration for Calculus, Analysis, and Differential Equations

Can We Integrate Them All Now? 125


9. Improper Integrals 127
9.1 Type 1 Improper Integrals (Unbounded Interval) . . . . . 127
9.1.1 Right-Sided Unboundedness . . . . . . . . . . . . 127
9.1.2 Left-Sided Unboundedness . . . . . . . . . . . . . 130
9.1.3 Two-Sided Unboundedness . . . . . . . . . . . . . 132
9.2 Type 2 Improper Integrals (Unbounded Integrand) . . . . 134
9.2.1 Unboundedness at the Left Endpoint . . . . . . . 135
9.2.2 Unboundedness at the Right Endpoint . . . . . . 137
9.2.3 Unboundedness Inside the Interval . . . . . . . . . 138
9.3 Applications . . . . . . . . . . . . . . . . . . . . . . . . . . 141
9.4 Practice Problems . . . . . . . . . . . . . . . . . . . . . . 142

Mixed Integration Problems 145

Answer Key 147


Appendix A Table of Basic Integrals 153

Appendix B Reduction Formulas 155

Appendix C Basic Identities of Algebra and Trigonometry 157

Bibliography 161

Index 163
May 17, 2018 12:7 ws-book9x6 Integration for Calculus, Analysis, and Differential Equations ws-book9x6 page 1

Chapter 1

Indefinite and Definite Integrals

1.1. Antiderivatives and Indefinite Integral

1.1.1. Definitions and Examples


Definition 1.1 (Antiderivative).
Let f be a function defined on an interval I. A function F is called an
antiderivative of f (x) on I if
F 0 (x) = f (x) for all x in I.

Examples 1.1 (Antiderivatives).


1. The function F (x) = 1 is an antiderivative of f (x) = 0 on (−∞, ∞) as
well as any function of the form F (x) = C, where C is an arbitrary real
constant (written henceforth as C ∈ R).
2. The function F (x) = x is an antiderivative of f (x) = 1 on (−∞, ∞) as
well as any function of the form F (x) = x + C, C ∈ R.
x2
3. The function F (x) = is an antiderivative of f (x) = x on (−∞, ∞)
2
x2
as well as any function of the form F (x) = + C, C ∈ R.
2
4. Any function of the form F (x) = ex + C, C ∈ R, is an antiderivative of
f (x) = ex on (−∞, ∞).

All the above examples have one thing in common: if F is an antiderivative


of f on I, then so is any function of the form
F (x) + C, x ∈ I, (1.1)
where C is an arbitrary real constant (C ∈ R).
The natural question is: are there antiderivatives of f on I not included in
this description? The answer is NO.

1
May 17, 2018 12:7 ws-book9x6 Integration for Calculus, Analysis, and Differential Equations ws-book9x6 page 2

2 Integration for Calculus, Analysis, and Differential Equations

As follows from the Mean Value Theorem (see, e.g., [1, 6]), functions with
the same derivative differ by a constant. Thus, if G is an arbitrary anti-
derivative of f on I, there is a C ∈ R such that

G(x) = F (x) + C, x ∈ I,

and hence, expression (1.1) describes all possible antiderivatives of f on I.


Definition 1.2 (Indefinite Integral).
Let a function f defined on an interval I have an antiderivative F on I.
The indefinite integral (or the general antiderivative) of f on I is the ex-
pression
Z
f (x) dx := F (x) + C, x ∈ I,

Z real constant (C ∈ R).


where C is an arbitrary Z
The integral notation f (x) dx, uses the integral symbol and differential
symbol dx. The function f is called the integrand and x the integration
variable.
The process of finding an indefinite integral is called integration (or anti-
differentiation).

Obviously, integration (antidifferentiation) is the process inverse to differ-


entiation, i.e.,
Z Z
d
f 0 (x) dx = f (x) + C and f (x) dx = f (x).
dx
Thus, the following examples are readily obtained by reversing the stan-
dard table of basic derivatives with some natural minor adjustments, when
required, and become a part of our Table of Basic Integrals (Appendix A).
Examples 1.2 (Basic Indefinite Integrals).
Z
1. 0 dx = C on (−∞, ∞).

xn+1
Z
2. xn dx = + C (n 6= −1)
n+1
on interval(s) depending on the exponent n.
x2
Z Z Z
In particular, 1 dx = x0 dx = x + C, x dx = +C
2
on (−∞, ∞),
May 17, 2018 12:7 ws-book9x6 Integration for Calculus, Analysis, and Differential Equations ws-book9x6 page 3

Indefinite and Definite Integrals 3

Z Z
1
dx = x−2 dx = −x−1 + C
x2
on each of the intervals (−∞, 0), (0, ∞),
√ x3/2
Z Z
2
x dx = x1/2 dx = = x3/2 + C on [0, ∞),
3/2 3
Z
1
3. dx = ln |x| + C on each of the intervals (−∞, 0), (0, ∞).
x
ax
Z
4. ax dx = + C (a > 0, a 6= 1) on (−∞, ∞).
ln a
Z
In particular, ex dx = ex + C on (−∞, ∞).
Z
5. sin x dx = − cos x + C on (−∞, ∞).
Z
6. cos x dx = sin x + C on (−∞, ∞).
Z
7. sec2 x dx = tan x + C on each of the intervals

(−π/2 + nπ, π/2 + nπ), n ∈ Z := {0, ±1, ±2, . . . } .


Z
8. csc2 x dx = − cot x+C on each of the intervals (nπ, π + nπ), n ∈ Z.
Z
9. sec x tan x dx = sec x + C on the same intervals as in 7.
Z
10. csc x cot x dx = − csc x + C on the same intervals as in 8.
Z
11. sinh x dx = cosh x + C on (−∞, ∞).
Z
12. cosh x dx = sinh x + C on (−∞, ∞).
Z
1
13. 2
dx = arctan x + C on (−∞, ∞).
x +1
Z
1
14. √ dx = arcsin x + C on (−1, 1).
1 − x2
Z
1
15. √ dx = arcsec |x| + C
x x2 − 1
on each of the intervals (−∞, −1), (1, ∞)
since, for x > 1,
May 17, 2018 12:7 ws-book9x6 Integration for Calculus, Analysis, and Differential Equations ws-book9x6 page 4

4 Integration for Calculus, Analysis, and Differential Equations

Z Z
1 1
√ dx = √ dx = arcsec |x| + C
x x2 − 1 |x| x2 − 1
and, for x < −1,
Z Z
1 1
√ dx = − √ dx
x x2 − 1 |x| x2 − 1
= − arcsec x+B = arcsec |x|−π+B = arcsec |x|+C (C := −π+B).

1.1.2. Validation of Indefinite Integrals


To validate the equality
Z
f (x) dx = F (x) + C on I

is to show that
F 0 (x) = f (x) on I,
i.e., integration is validated via differentiation.
Examples 1.3 (Appendix A, integrals 11, 13, and 20).
Z
1. tan x dx = ln | sec x| + C = − ln | cos x| + C on each of the intervals
(−π/2 + nπ, π/2 + nπ), n ∈ Z, since, on each of these intervals,
0 sec0 x sec x tan x
[ln | sec x|] = = = tan x
sec x sec x
and
−1
ln | sec x| = ln [| cos x|] = − ln | cos x|.
Z
2. sec x dx = ln | sec x + tan x| + C on the same intervals as in the prior
example since, on each of these intervals,
0 (sec x + tan x)0 sec x tan x + sec2 x
[ln | sec x + tan x|] = = = sec x.
sec x + tan x sec x + tan x
3. For a > 0,
Z
1 p
√ dx = ln |x + x2 ± a2 | + C
x2 ± a 2
on (−∞, ∞) for “+” and on each of the intervals (−∞, −a), (a, ∞) for
“−” since, on each of these intervals,
May 17, 2018 12:7 ws-book9x6 Integration for Calculus, Analysis, and Differential Equations ws-book9x6 page 5

Indefinite and Definite Integrals 5

 2 0
√ x ± a2
 0 1+ √
d p x + x2 ± a 2 2√ x2 ± a2
ln |x + x2 ± a2 | = √ =
dx x + x2 ± a2 x + x2 ± a2

2x x2 ± a 2 + x
1+ √ √
2√ x2 ± a2 2 2 1
= = √ ±a
x
=√ .
x+ x ±a 2 2 x+ x ±a 2 2 x ± a2
2

1.1.3. Which Functions Are Integrable?


As follows from the Fundamental Theorem of Calculus (see Sec. 1.2.6), each
function f continuous on an interval I has an antiderivative F on I (i.e., is
integrable on I). For a function f having discontinuities on an interval I,
this need not be true.
Exercise 1.1 (Sign Function).
Show that the sign function

−1 for x < 0


sgn(x) := 0 for x = 0


 1 for x > 0
has antiderivatives on (−∞, 0), (0, ∞), but has no antiderivative on any
open interval I containing 0.

1.1.4. Properties of Indefinite Integral (Integration Rules)


Integration is governed by the following rules readily proved via differenti-
ation.
Theorem 1.1 (Properties of Indefinite Integral (Integration
Rules)).
Z Z
If f (x) dx = F (x) + C and g(x) dx = G(x) + C on an interval I, then,
on I,
Z Z
1. for any c ∈ R, cf (x) dx = cF (x) + C = c f (x) dx (Constant
Factor
Z Rule) Z Z
2. [f (x) ± g(x)] dx = F (x) ± G(x) + C = f (x) dx ± g(x) dx
(Sum/Difference Rule)
May 17, 2018 12:7 ws-book9x6 Integration for Calculus, Analysis, and Differential Equations ws-book9x6 page 6

6 Integration for Calculus, Analysis, and Differential Equations

Remark 1.1. For integration, unlike differentiation, the are NO Product/


Quotient Rules.

Exercise 1.2 (No Product/Quotient Rules for Integration).


Give an example of functions f and g having antiderivatives F and G on
an interval I, respectively, for which
Z Z
f (x) F (x)
f (x)g(x) dx 6= F (x)G(x) + C and dx 6= + C on I.
g(x) G(x)
Example 1.4 (Using Integration Rules).
Z  
4 x
+ 3 sin x − e dx switching to the power form;
x2
Z
4x−2 + 3 sin x − ex dx

= by the integration rules;
Z Z Z
= 4 x−2 dx + 3 sin x dx − ex dx

using the basic integrals (see Examples 1.2 and Appendix A);
4
=− − 3 cos x − ex + C.
x

1.2. Definite Integral

1.2.1. Definitions
Definition 1.3 (Partition, Regular Partition).
A set of n + 1 points {x0 , x1 , . . . , xn } (n = 1, 2, . . . ) in a closed and bounded
interval [a, b] such that
a = x0 < x1 < · · · < xn−1 < xn = b
is called a partition of [a, b].
The points x0 , x1 . . . , xn , called the grid points, partition [a, b] into n sub-
intervals
[x0 , x1 ], [x1 , x2 ], . . . , [xn−1 , xn ],
the length of the kth partition interval being
∆xk = xk − xk−1 , k = 1, . . . , n.

Remark 1.2. The partition intervals need not be of equal length.


May 17, 2018 12:7 ws-book9x6 Integration for Calculus, Analysis, and Differential Equations ws-book9x6 page 7

Indefinite and Definite Integrals 7

The maximal length of the partition intervals

∆ := max(∆x1 , ∆x2 , . . . , ∆xn )

is called the diameter or norm of the partition.


If all the partition intervals are of equal length
b−a
∆x = ,
n
the partition {x0 , x1 , . . . , xn } is called regular. In this case, the kth grid
point is expressed as follows:

xk = a + k∆x, k = 0, 1, . . . , n.

Definition 1.4 (Riemann Sum).


Let a function f (x) be defined on an interval [a, b]. Consider a partition
{x0 , x1 , . . . , xn } (n = 1, 2, . . . ) of [a, b] (−∞ < a < b < ∞) into n subinter-
vals

[x0 , x1 ], [x1 , x2 ], . . . , [xn−1 , xn ],

(of not necessarily equal length), the length of the kth partition interval
being

∆xk = xk − xk−1 , k = 1, . . . , n.

Let x∗k be an arbitrary intermediate point in the kth partition interval


[xk−1 , xk ], k = 1, 2, . . . , n.
The Riemann1 sum for f corresponding to the partition and chosen inter-
mediate points is
n
X
f (x∗k )∆xk = f (x∗1 )∆x1 + f (x∗2 )∆x2 + · · · + f (x∗n )∆xn .
k=1

If the partition is regular,


b−a
∆xk = ∆x = , k = 1, . . . , n,
n
and we have:
n
X
f (x∗k )∆x = f (x∗1 )∆x + f (x∗2 )∆x + · · · + f (x∗n )∆x.
k=1

In particular,
1 Bernhard Riemann (1826 –1866)
May 17, 2018 12:7 ws-book9x6 Integration for Calculus, Analysis, and Differential Equations ws-book9x6 page 8

8 Integration for Calculus, Analysis, and Differential Equations

• a left Riemann sum (x∗k = xk−1 = a + (k − 1)∆x, k = 1, 2, . . . , n) is


Xn n
X
f (xk−1 )∆x = f (a + (k − 1)∆x) ∆x;
k=1 k=1
• a right Riemann sum (x∗k = xk = a + k∆x, k = 1, 2, . . . , n) is
Xn n
X
f (xk )∆x = f (a + k∆x) ∆x;
k=1 k=1
• a midpoint Riemann sum (x∗k = (xk−1 + xk )/2 = a + (k − 1/2)∆x,
k = 1, 2, . . . , n) is
X n n
X
f ((xk−1 + xk )/2) ∆x = f (a + (k − 1/2)∆x) ∆x.
k=1 k=1

Definition 1.5 (Net Area).


The net area of the region R bounded by the graph of a continuous on an
interval [a, b] (−∞ < a < b < ∞) function f and the x-axis is the sum of
the areas of the parts of R that lie above the x-axis minus the sum of the
areas of the parts of R that lie below the x-axis.
Definition 1.6 (Definite Integral).
We say that a function f defined on an interval [a, b] (−∞ < a < b < ∞)
is (Riemann-)integrable on [a, b] if the limit of Riemann sums
Xn
lim f (x∗k )∆xk ,
∆→0
k=1
where
∆ := max ∆xk
1≤k≤n
is the diameter (or norm) of the partition, exists (over all partitions of [a, b]
and all choices of the intermediate points x∗k ) and is finite.
The limit
Z b n
X
f (x) dx := lim f (x∗k )∆xk
a ∆→0
k=1
is called the definite (or Riemann) integral of f from a to b (or over [a, b]).
Z b Z
The integral notation f (x) dx, uses the integral symbol and differen-
a
tial symbol dx.
The function f is called the integrand, x the integration variable, and a and
b are called the lower and upper limits of integration, respectively.
The process of evaluation of a definite integral is called integration.
May 17, 2018 12:7 ws-book9x6 Integration for Calculus, Analysis, and Differential Equations ws-book9x6 page 9

Indefinite and Definite Integrals 9

1.2.2. Which Functions Are Integrable?


A necessary, but not sufficient, condition of the Riemann integrability of a
function f on an interval [a, b] (−∞ < a < b < ∞) is its boundedness of on
[a, b] (see, e.g., [4]).
Theorem 1.2 (Integrable Functions).
A function f bounded and having a finite number of discontinuities on an
interval [a, b] (−∞ < a < b < ∞) is integrable on [a, b].

Definition 1.7 (Piecewise Continuous Function).


A function f defined on an interval [a, b] (−∞ < a < b < ∞) is called
piecewise continuous on [a, b] if it is continuous everywhere on [a, b], except
at a finite number of jump discontinuities.

Remark 1.3. A function f continuous on [a, b] is trivially piecewise con-


tinuous.

Corollary 1.1 (Piecewise Continuous Functions Are Integrable).


A function f (x) piecewise continuous on [a, b] is integrable on [a, b].

For an integrable on [a, b] function f (x),


Z b
f (x) dx = Net Area.
a

1.2.3. Properties of Definite Integral (Integration Rules)


Theorem 1.3 (Properties of Definite Integral (Integration Rules)).
If functions f (x) and g(x) are integrable on an interval [a, b], then
Z b Z b
1. for any real c, cf (x) dx = c f (x) dx (Constant Factor Rule)
Z b a Z b a Z
b
2. [f (x) ± g(x)] dx = f (x) dx ± g(x) dx (Sum/Difference Rule)
a Z b a Z c a Z b
3. for a < c < b, f (x) dx = f (x) dx + f (x) dx (Additivity)
a a c

Remark 1.4. For definite integral, the are NO Product/Quotient Rules.

Definition 1.8 (More General Use of Integral Notation).


Z a
• For b = a, f (x) dx := 0.
a
May 17, 2018 12:7 ws-book9x6 Integration for Calculus, Analysis, and Differential Equations ws-book9x6 page 10

10 Integration for Calculus, Analysis, and Differential Equations

Z b Z a
• For b < a, f (x) dx := − f (x) dx.
a b

Remark 1.5. These definitions preserve all the above properties of definite
integral. The generalized version of additivity
Z b Z c Z b
f (x) dx = f (x) dx + f (x) dx
a a c

holds for any triple of numbers a, b, and c regardless of the order, provided
the function f is integrable between any two of the three points.

1.2.4. Integration by Definition


Example 1.5 (Integration by Definition).
Evaluate the integral
Z 4
(x2 + 3x − 2) dx
1

by the definition using regular partitions and right Riemann sums.


Solution: For the function f (x) = x2 + 3x − 2 on the interval [1, 4],
considering regular partitions of [1, 4] into n subintervals with
4−1 3
= ∆x =
n n
and choosing intermediate points x∗k to be the right endpoints:
3
x∗k = xk = 1 + k, k = 1, 2, . . . , n,
n
we have:
Z 4 n n  
X X 3 3
(x2 + 3x − 2) dx = lim f (xk )∆x = lim f 1+ k
1 n→∞ n→∞ n n
k=1 k=1
n  2   
X 3 3 3
= lim 1+ k +3 1+ k −2
n→∞ n n n
k=1

transforming and simplifying algebraically;


n   n  
X 3 9 9 3 X 9 2 15 3
= lim 1+2 k+ 2 k 2 +3+ k−2 = lim 2
k + k+2
n→∞ n n n n n→∞ n n n
k=1 k=1
n 
X 27 
45 6
= lim k2 + 2 k + by the summation rules;
n→∞ n3 n n
k=1
May 17, 2018 12:7 ws-book9x6 Integration for Calculus, Analysis, and Differential Equations ws-book9x6 page 11

Indefinite and Definite Integrals 11

 n n n 
27 X 2 45 X 6X
= lim k + 2 k+ 1
n→∞ n3 n n
k=1 k=1 k=1

by the special summation formulas (see, e.g., [1, 6]);


 
27 n(n + 1)(2n + 1) 45 n(n + 1) 6
= lim + 2 + n dividing termwise;
n→∞ n3 6 n 2 n
      
9 1 1 45 1
= lim 1+ 2+ + 1+ +6 by the limit laws;
n→∞ 2 n n 2 n
9 45 9 45 45 75
= (1 + 0)(2 + 0) + (1 + 0) + 6 = · 2 + · 1 + 6 = 15 + = .
2 2 2 2 2 2

1.2.5. Integral Mean Value Theorem

Theorem 1.4 (Integral Mean Value Theorem).


If f is a continuous function on an interval [a, b] (−∞ < a < b < ∞), then
there exists a point c ∈ [a, b] such that
Z b
1
f (c) = f (x) dx
b−a a
or
Z b
f (c)(b − a) = f (x) dx.
a

Proof. By the Extreme Value Theorem (see, e.g., [1, 6]), the continuous
function f attains its absolute minimum m and maximum M on [a, b]. Then
for an arbitrary Riemann sum
n
X n
X n
X
m(b − a) = m ∆xk ≤ f (x∗k )∆xk ≤ M ∆xk = M (b − a).
k=1 k=1 k=1

Passing to the limit as ∆ := max ∆xk → 0, we obtain


1≤k≤n

Z b
m(b − a) ≤ f (x) dx ≤ M (b − a).
a

Whence,
Z b
1
m≤ f (x) dx ≤ M.
b−a a
May 31, 2018 10:33 ws-book9x6 Integration for Calculus, Analysis, and Differential Equations ws-book9x6 page 12

12 Integration for Calculus, Analysis, and Differential Equations

Since, by the Intermediate Value Theorem (see, e.g., [1, 6]), the continu-
ous function f takes on every value between its absolute minimum m and
maximum M on [a,b], there exists a point c ∈ [a, b] such that
Z b
1
f (c) = f (x) dx.
b−a a

Remarks 1.6 (Integral Mean Value Theorem).

• The Integral Mean Value Theorem has the following natural geometric
interpretation.
If f (x) is a continuous function on an interval [a, b], then there is a point
c ∈ [a, b] such that the net area of the region bounded by the graph of
f (x) and the x-axis over [a, b] is equal to the area of the rectangle with
base [a, b] and height f (c) provided f (c) ≥ 0 or the negative of the area
of the rectangle with base [a, b] and height −f (c) provided f (c) < 0.
If f (x) ≥ 0 on [a, b], the net area is the proper area.
• The Integral Mean Value Theorem remains valid for b < a, in which
case, f (x) is required to be continuous between a and b where the point
c is to be found as well.

1.2.6. Fundamental Theorem of Calculus

Definition 1.9 (Area Function).


Let f (x) be a function defined on an interval I and integrable on every
subinterval [a, b] ⊆ I. Fixing a point a in I, we can define the area function
for f (x) with lower limit a as follows:
Z x
A(x) := f (t) dt, x ∈ I.
a

For x ≥ a, A(x) represents the net area of the region bounded by the graph
of f (x) and the x-axis between a and x and, for x < a, considering that
Z x Z a
A(x) = f (t) dt = − f (t) dt,
a x

A(x) represents the negative of the net area of the region bounded by the
graph of f (x) and the x-axis between a and x.
May 17, 2018 12:7 ws-book9x6 Integration for Calculus, Analysis, and Differential Equations ws-book9x6 page 13

Indefinite and Definite Integrals 13

Theorem 1.5 (Fundamental Theorem of Calculus (Part 1)).


If f is a continuous function on an interval I containing a point a, then
the area function for f with lower limit a
Z x
A(x) := f (t) dt, x ∈ I,
a

is differentiable on I and
Z x
0 d
A (x) = f (t) dt = f (x), x ∈ I.
dx a

The derivatives at the endpoints (if any) are understood as one-sided.

Proof. For any x ∈ I and all sufficiently small increments h, such that
x + h ∈ I,
"Z #
x+h x
A(x + h) − A(x)
Z
1
= f (t) dt − f (t) dt
h h a a

by additivity (Theorem 1.3);


Z x+h
1
= f (t) dt
h x
by the Integral Mean Value Theorem, there is a c(h) between x and x + h;
= f (c(h)).

Hence, by the definition of derivative, for any x ∈ I,

A(x + h) − A(x)
A0 (x) = lim = lim f (c(h))
h→0 h h→0

since c(h) → x as h → 0, by the continuity of f (x);


= f (x).

Theorem 1.6 (Fundamental Theorem of Calculus (Part 2)).


If a function f is (Riemann-)integrable and has an antiderivative F (x) on
an interval [a, b] (−∞ < a < b < ∞), then
Z b
f (x) dx = F (b) − F (a) (Newton-Leibniz Formula).
a
May 31, 2018 10:33 ws-book9x6 Integration for Calculus, Analysis, and Differential Equations ws-book9x6 page 14

14 Integration for Calculus, Analysis, and Differential Equations

Proof. Consider an arbitrary partition {x0 , x1 , . . . , xn } of [a, b] into n


subintervals (n = 1, 2, . . . ).
Since F (x) satisfies the conditions of the Mean Value Theorem on [a, b] (see,
e.g., [1, 6]), and thus, on each partition interval [xk−1 , xk ], k = 1, 2, . . . , n,
for each k = 1, 2, . . . , n, there is a point x∗k ∈ (xk−1 , xk ) such that

F (xk ) − F (xk−1 ) = F 0 (x∗k )(xk − xk−1 ) = f (x∗k )∆xk .

Hence,
n
X n
X
F (b) − F (a) = [F (xk ) − F (xk−1 )] = f (x∗k )∆xk
k=1 k=1

and
n
X Z b
F (b) − F (a) = lim f (x∗k )∆xk = f (x) dx,
∆→0 a
k=1

where ∆ := max ∆xk .


1≤k≤n

Remarks 1.7 (Fundamental Theorem of Calculus (Part 2)).


b

• Using the shorthand F (x) for F (b)−F (a), we can rewrite the Newton2 -
a
Leibniz3 Formula as follows:
Z b b

f (x) dx = F (x) .
a a

• The Newton-Leibniz Formula also works when a ≥ b. Indeed, in this


case,
Z b Z a b

f (x) dx = − f (x) dx = −[F (a) − F (b)] = F (b) − F (a) = F (x) .
a b a

• For a continuous function f on an interval [a, b] both conditions of


the Fundamental Theorem of Calculus (Part 2) are met, and thus, the
Newton-Leibniz Formula applies.
• Generally, a function f can be integrable without having an antideriva-
tive on [a, b] and need not be integrable while having an antiderivative
on [a, b].
2 Isaac Newton (1642–1727)
3 Gottfried Wilhelm (von) Leibniz (1646 –1716)
May 31, 2018 10:33 ws-book9x6 Integration for Calculus, Analysis, and Differential Equations ws-book9x6 page 15

Indefinite and Definite Integrals 15

For instance, the sign function



−1 if x < 0


sgn x := 0 if x = 0


 1 if x > 0
is integrable on [−1, 1], being piecewise continuous, but has no anti-
derivative on [−1, 1] (see Exercise 1.1).
Whereas, the function

2x sin 1 − 2 sin 1 if x 6= 0
f (x) = x2 x x2
0 if x = 0
is not integrable on [−1, 1], being unbounded, but has an antiderivative

x2 sin 1 if x 6= 0
F (x) = x2
0 if x = 0
(see [4]).

1.2.7. Total Change Theorem


An immediate consequence of the Fundamental Theorem of Calculus (Part
2) is the following
Theorem 1.7 (Total Change Theorem).
If a function f is a continuously differentiable function on an interval [a, b]
(−∞ < a < b < ∞), then
Z b
f 0 (x) dx = f (b) − f (a) (Total Change Formula),
a
i.e., the integral of the instantaneous rate of change function f 0 over the
interval [a, b] is equal to the total change of f over [a, b].

Remark 1.8. The Total Change Theorem has the following physical inter-
pretation.
If v is the (continuous) velocity function of an object moving along the
x-axis with the position function x, then the displacement of the object
between the moments t = a and t = b is
Z b
x(b) − x(a) = v(t) dt,
a
which follows immediately from the prior theorem in view of v(t) = x0 (t),
a ≤ t ≤ b.
May 17, 2018 12:7 ws-book9x6 Integration for Calculus, Analysis, and Differential Equations ws-book9x6 page 16

16 Integration for Calculus, Analysis, and Differential Equations

1.2.8. Integrals of Even and Odd Functions


Theorem 1.8 (Integrals of Symmetric Functions over Symmetric
Intervals).
Let a > 0 and f be an integrable function on the interval [−a, a].
Z a Z a
• If f is even (f (−x) = f (x)), f (x) dx = 2 f (x) dx.
Z−aa 0

• If f is odd (f (−x) = −f (x)), f (x) dx = 0.


−a

Example 1.6 (Integrals of Symmetric Functions over Symmetric


Intervals).
Evaluate the integral
Z 4
sin7 x
2 10
dx.
−4 (x + 1)

sin7 x
Solution: Since, the continuous function is odd and the in-
(x2 + 1)10
terval [−4, 4] is symmetric about 0,
Z 4
sin7 x
2 10
dx = 0.
−4 (x + 1)
May 17, 2018 12:7 ws-book9x6 Integration for Calculus, Analysis, and Differential Equations ws-book9x6 page 17

Chapter 2

Direct Integration

Recall that the process of finding an indefinite integral is called integration.


Just like for differentiation, for integration purposes, along with the inte-
gration rules (see Theorem 1.1), we need a set of validated or established
basic integrals, called the table of basic integrals.

2.1. Table Integrals and Useful Integration Formula

For our integration purposes, we use the Table of Basic Integrals consisting
of twenty-one integrals (see Appendix A). We have already validated most
of them in the preceding examples. Observe that there exist much more
extensive tables of integrals.
Exercise 2.1 (Validating Table Integrals).
Validate integrals 12 and 14 of the Table of Basic Integrals (Appendix A).

We are almost ready to integrate now. However, at the moment, we are


in the situation of an awkward deficiency demonstrated by the following
examples: knowing the table integrals
Z Z
ex dx = ex + C or sin x dx = − cos x + C,

we do not know how to evaluate their slightest variations such as


Z Z
e−x dx or sin(2x + 3) dx.

These are particular cases of the routine situation of evaluating the integral
of the form
Z
f (ax + b) dx,

17
May 17, 2018 12:7 ws-book9x6 Integration for Calculus, Analysis, and Differential Equations ws-book9x6 page 18

18 Integration for Calculus, Analysis, and Differential Equations

Z
where a and b are real coefficients with a 6= 0, when f (x) dx is known,
which, as we shall see in Sec. 3.1.4, can be treated by the Method of
Substitution.
To deal with this very common situation, let us, however, not wait until the
Substitution Method is developed, but prove and start applying immediately
the following
Theorem 2.1 (Useful Integration Formula).
Let a and b be real coefficients with a 6= 0. If
Z
f (x) dx = F (x) + C,

then
Z
1
f (ax + b) dx = F (ax + b) + C.
a
Proof. Considering that F 0 (x) = f (x), by the Chain Rule (see, e.g., [1, 6]),
we have:
 
d 1 1 1
F (ax + b) = F 0 (ax + b)[ax + b]0 = f (ax + b)a = f (ax + b).
dx a a a

Remarks 2.1 (Useful Integration Formula).

• We intentionally do not indicate the intervals of integration not to ob-


scure the simplicity of the formulation and proof.
• As we see in Sec. 3.1.4, the Useful Integration Formula (Theorem 2.1)
is a special case of the so-called trivial substitution.

Examples 2.1 (Applying the Useful Integration Formula). Here-


after, the boxes contain explanatory formulas and/or text.
Z
1. e−x dx by the useful integration formula with a = −1 and b = 0;
1 −x
= e + C = −e−x + C.
(−1)

Z
2. sin(2x + 3) dx

by the useful integration formula with a = 2 and b = 3;


May 17, 2018 12:7 ws-book9x6 Integration for Calculus, Analysis, and Differential Equations ws-book9x6 page 19

Direct Integration 19

1 1
= (− cos(2x + 3)) + C = − cos(2x + 3) + C.
2 2


Z
3
3. 4 − 10x dx switching to the power form;
Z
= (4 − 10x)1/3 dx

by the useful integration formula with a = −10 and b = 4;


1 1 3
= · (4 − 10x)4/3 + C = − (4 − 10x)4/3 + C.
(−10) 4/3 40

Z
1
4. dx
x/2 + π
by the useful integration formula with a = 1/2 and b = π;
1
= ln |x/2 + π| + C = 2 ln |x/2 + π| + C.
1/2

Table of Basic Integrals (Appendix A), integrals 17 and 21.


For a > 0,
Z Z
1 1 1
5. dx = dx
x2 + a2 a2 (x/a)2 + 1
by the useful integration formula with a = 1/a and b = 0;
1 1 x 1 x
= 2 arctan + C = arctan + C on (−∞, ∞).
a 1/a a a a

Z Z
1 1 1
6. √ dx = dx + C
a2
p
x x2 − a2 (x/a) (x/a)2 − 1
by the useful integration formula with a = 1/a and b = 0;
1 1 x 1 x
= 2 arcsec + C = arcsec + C

a 1/a a a a
on each of the intervals (−∞, −a), (a, ∞).

Exercise 2.2 (Deriving Table Integrals).


Apply the Useful Integration Formula (Theorem 2.1) to derive integral 19
of the Table of Basic Integrals (Appendix A).
May 31, 2018 10:33 ws-book9x6 Integration for Calculus, Analysis, and Differential Equations ws-book9x6 page 20

20 Integration for Calculus, Analysis, and Differential Equations

When evaluating an integral of the form


Z
(ax + b)n dx,

where a and b are real coefficients and n = 1, 2, 3, . . . is a natural number,


the Useful Integration Formula (Theorem 2.1) becomes especially handy
for large values of the exponent n, when the use of the binomial formula
leads to cumbersome computations.
Example 2.2 (Applying the Useful Integration Formula).
Evaluate the integral
Z
(5x − 7)99 dx.

Solution: Using the binomial formula (see Appendix C) to expand (5x −


7)99 would result into 100 terms. Applying the Useful Integration Formula
(Theorem 2.1) “saves the day”:
Z
(5x − 7)99 dx

by the useful integration formula with a = 5 and b = −7;


1 1 1
= (5x − 7)100 + C = (5x − 7)100 + C.
5 100 500

2.2. What Is Direct Integration and How Does It Work?

What Is Direct Integration?


Definition 2.1 (Direct Integration).
By direct integration, we understand the process of integration, which, using
the integration rules (Theorem 1.1) alone, reduces the integral of a given
function to a combination of table integrals. Such integration makes no use
of any special integration techniques, but may employ the Useful Integration
Formula (Theorem 2.1) when appropriate.

Remark 2.2. When executing direct integration, before applying the in-
tegration rules, one may need to manipulate the integrand performing mul-
tiplication/division, applying relevant identities of algebra/trigonometry,
or using such tricks as transforming products into sums, multiplying and
dividing by the conjugate radical expression, or completing the square.
May 17, 2018 12:7 ws-book9x6 Integration for Calculus, Analysis, and Differential Equations ws-book9x6 page 21

Direct Integration 21

How Does Direct Integration Work?


In the following examples, we consider various scenarios of direct integration
explaining its execution step-by-step.

2.2.1. By Integration Rules Only


Example 2.3 (Integration by the Rules Only).
Z
(7x3 + 5e−x + 14) dx by the integration rules;
Z Z Z
= 7 x3 dx + 5 e−x dx + 14 1 dx

by the useful integration formula;


7
= x4 − 5e−x + 14x + C.
4

2.2.2. Multiplication/Division Before Integration


When evaluating an integral one is to be mindful of the absence of the
product and quotient rules for integration.
Thus, the following “solution”
Z Z
(x − 1) dx (x + 2) dx
(x − 1)(x + 2)
Z
√ dx = = ···

Z
x
x dx

is incorrect.
To solve correctly, we are to execute multiplication and division before
integration.
Examples 2.4 (Multiplication/Division Before Integration).

(x − 1)(x + 2)
Z
1. √ dx multiplying and switching to the power form;
x
Z 2
x +x−2
= dx dividing termwise;
x1/2
Z
= (x3/2 + x1/2 − 2x−1/2 ) dx by the integration rules;
Z Z Z
2 2
= x3/2 dx + x1/2 dx − 2 x−1/2 dx = x5/2 + x3/2 − 4x1/2 + C.
5 3
May 17, 2018 12:7 ws-book9x6 Integration for Calculus, Analysis, and Differential Equations ws-book9x6 page 22

22 Integration for Calculus, Analysis, and Differential Equations

x3 − 2x2 + 4x − 5
Z
2. dx
x−1
x3 − 2x2 + 4x − 5 2
dividing = x2 − x + 3 − (verify);
x−1 x−1
Z  
2
= x2 − x + 3 − dx by the integration rules;
x−1
Z Z Z Z
1
= x2 dx − x dx + 3 1 dx − 2 dx
x−1
by the useful integration formula;
x3 x2
= − + 3x − 2 ln |x − 1| + C.
3 2

The latter example represents a particular case of integration of rational


functions considered in detail in Chapter 7.

2.2.3. Applying Minor Adjustments

By “minor adjustments” to the integrand, we understand adding and sub-


tracting or multiplying and dividing by the same constant.

Example 2.5 (Applying Minor Adjustments).


Z
x 5 − 2x dx multiplying and dividing by −2;
Z
1
=− (−2x)(5 − 2x)1/2 dx adding and subtracting 5;
2
Z Z
1 1 h i
=− (5−2x−5)(5−2x)1/2 dx = − (5 − 2x)3/2 − 5(5 − 2x)1/2 dx
2 2
by the integration rules;
Z Z
1 5
=− (5 − 2x)3/2 dx + (5 − 2x)1/2 dx
2 2
by the useful integration formula;
1 1 2 5 1 2
=− (5 − 2x)5/2 + (5 − 2x)3/2 + C
2 (−2) 5 2 (−2) 3
1 5
= (5 − 2x)5/2 − (5 − 2x)3/2 + C.
10 6
June 21, 2018 10:23 ws-book9x6 Integration for Calculus, Analysis, and Differential Equations ws-book9x6 page 23

Direct Integration 23

2.2.4. Using Identities


Certain identities of algebra and trigonometry useful for integration can be
found in Appendix C.
Examples 2.6 (Using Identities).
Z
1. 25x 3−x dx by the exponents laws (see Appendix C);
Z  x
(32/3)x
Z
x 32
= 25 · 3−1 dx = dx = + C.
3 ln(32/3)

Z
cos x + 1
2. dx dividing termwise and rewriting equivalently;
sin2 x
Z   Z "  2 #
cos x 1 1 cos x 1
= + dx = + dx
sin2 x sin2 x sin x sin x sin x
1 cos x
since = csc x, = cot x (see Appendix C);
sin x sin x
Z
csc x cot x + csc2 x dx
 
= by the integration rules;
Z Z
= csc x cot x dx + csc2 x dx = − csc x − cot x + C.

When evaluating the trigonometric integral


Z
cos2 x dx,

in the absence of the product rule for integration, we are to use power-
reduction identity
1 + cos 2θ
cos2 θ =
2
(see Appendix C) as follows:
Example 2.7 (Using Identities).
Z
cos2 x dx

1 + cos 2x
by the power-reduction identity with θ = x, cos2 x = ;
2
May 31, 2018 10:33 ws-book9x6 Integration for Calculus, Analysis, and Differential Equations ws-book9x6 page 24

24 Integration for Calculus, Analysis, and Differential Equations

Z
1 + cos 2x
= dx by the integration rules;
2
Z Z 
1
= 1 dx + cos 2x dx by the useful integration formula;
2
 
1 1 1 1
= x + sin 2x + C = x + sin 2x + C.
2 2 2 4

More generally, using a relevant power-reduction identity allows us to eval-


uate integrals of the form
Z Z
2
cos (ax + b) dx, sin2 (ax + b) dx,

where a and b are real coefficients with a 6= 0.

Similarly, the trigonometric integral


Z
sin 3x cos x dx

containing a product is found via the product-to-sum identity


1
sin α cos β = [sin(α − β) + sin(α + β)]
2
(see Appendix C) as follows:

Example 2.8 (Using Identities).


Z
sin 3x cos x dx

by the relevant product-to-sum identity with α = 3x and β = x,


1 1
sin 3x cos x = [sin(3x − x) + sin(3x + x)] = [sin 2x + sin 4x] ;
2 2
Z
1
= [sin 2x + sin 4x] dx by the integration rules;
2
Z Z 
1
= sin 2x dx + sin 4x dx by the useful integration formula;
2
 
1 1 1 1 1
= (− cos 2x) + (− cos 4x) + C = − cos 2x − cos 4x + C.
2 2 4 4 8
May 31, 2018 10:33 ws-book9x6 Integration for Calculus, Analysis, and Differential Equations ws-book9x6 page 25

Direct Integration 25

More generally, this approach applies to the integrals of the form


Z
cos(ax + b) cos(cx + d) dx,
Z
sin(ax + b) sin(cx + d) dx,
Z
sin(ax + b) cos(cx + d) dx,

where a, b, c, and d are real coefficients with a, c 6= 0.

The following trigonometric integral


Z
tan2 5x dx

can be evaluated via the Pythagorean identity

tan2 θ = sec2 θ − 1

(see Appendix C) as follows:

Example 2.9 (Using Identities).


Z
tan2 5x dx

by the latter identity with θ = 5x, tan2 5x = sec2 5x − 1 ;


Z
 2 
= sec 5x − 1 dx by the integration rules;
Z Z
= sec2 5x dx − 1 dx by the useful integration formula;
1
= tan 5x − x + C.
5

More generally, this approach applies to the integrals of the form


Z Z
2
tan (ax + b) dx, cot2 (ax + b) dx,

where a and b are real coefficients with a 6= 0.


May 17, 2018 12:7 ws-book9x6 Integration for Calculus, Analysis, and Differential Equations ws-book9x6 page 26

26 Integration for Calculus, Analysis, and Differential Equations

2.2.5. Transforming Products into Sums


As is seen from Example 2.8, in the absence of the product rule for integra-
tion, one can attempt transforming a product into a sum. Let us use this
approach for the following problem.
Example 2.10 (Transforming Products into Sums).
Evaluate the integral Z
1
dx.
(2x − 1)(2x + 3)
Solution: The integrand is a product
1 1
· .
2x − 1 2x + 3
However, the simple fact that the polynomials 2x − 1 and 2x + 3 share the
same nonconstant term 2x allows us to rewrite it as follows: 
1 1 1 1 1
· = −
2x − 1 2x + 3 4 2x − 1 2x + 3
by replacing the product with the difference
1 1

2x − 1 2x + 3
and scaling the latter by coefficient 1/4, where the denominator 4 is the
difference of the constant terms: 3 − (−1).
Thus,
Z
1
dx transforming product into sum:
(2x − 1)(2x + 3)
 
1 1 1 1
= − ;
(2x − 1)(2x + 3) 4 2x − 1 2x + 3
Z  
1 1 1
= − dx by the integration rules;
4 2x − 1 2x + 3
Z Z 
1 1 1
= dx − dx by the useful integration formula;
4 2x − 1 2x + 3
 
1 1 1 1
= ln |2x − 1| − ln |2x + 3| +C = [ln |2x − 1| − ln |2x + 3|]+C.
4 2 2 8
The prior problem, as well as problem 2 from Examples 2.4, is a special
case of integration of rational functions studied in Chapter 7.
Example 2.11 (Table of Basic Integrals (Appendix A), integral 18).
For a > 0,
x − a
Z
1 1
dx = ln +C
x2 − a2 2a x + a
May 17, 2018 12:7 ws-book9x6 Integration for Calculus, Analysis, and Differential Equations ws-book9x6 page 27

Direct Integration 27

on each of the intervals (−∞, −a), (−a, a), (a, ∞).


Z
1
dx transforming product into sum:
x2 − a2
 
1 1 1 1 1
= = − ;
x2 − a 2 (x − a)(x + a) 2a x − a x + a
Z  
1 1 1
= − dx by the integration rules;
2a x − a x + a
Z Z 
1 1 1
= dx − dx
2a x−a x+a
by the useful integration formula with a = 1 and b = ±a;
1
= [ln |x − a| − ln |x + a|] + C
2a
by the laws of logarithms (see Appendix C);

1 x − a

= ln + C.
2a x + a

2.2.6. Using Conjugate Radical Expressions


Example 2.12 (Using Conjugate Radical Expressions).
Z
1
√ √ dx
2x + 7 − 2x + 3
multiplying and dividing by the conjugate radical expression;
Z √ √
2x + 7 + 2x + 3
= √ √ √ √ dx simplifying;
( 2x + 7 − 2x + 3)( 2x + 7 + 2x + 3)
Z √ √

Z
2x + 7 + 2x + 3 1 √ 
= dx = 2x + 7 + 2x + 3 dx
2x + 7 − (2x + 3) 4
switching to the power form;
Z
1h i
= (2x + 7)1/2 + (2x + 3)1/2 dx by the integration rules;
4
Z Z 
1 1/2 1/2
= (2x + 7) dx + (2x + 3) dx
4
by the useful integration formula;
112 112 (2x + 7)3/2 (2x + 3)3/2
= (2x+7)3/2 + (2x+3)3/2 +C = + +C.
423 423 12 12
May 31, 2018 10:33 ws-book9x6 Integration for Calculus, Analysis, and Differential Equations ws-book9x6 page 28

28 Integration for Calculus, Analysis, and Differential Equations

2.2.7. Square Completion


The square completion technique is applied when evaluating integrals con-
taining quadratic polynomials of the following two types:
Z Z
1 1
dx or √ dx,
ax2 + bx + c 2
ax + bx + c
where a, b, and c are numeric coefficients with a 6= 0.
Example 2.13 (Square Completion).
Z
1
dx factoring out the leading coefficient;
3x2 − x + 4
Z
1 1
= 2
dx
3 x − x/3 + 4/3
completing the square:
 2
1 4 1 1 1 4 1 47
x2 − x + = x2 − 2 x + − + = x− + ;
3 3 6 36 36 3 6 36
Z
1 1
= √ dx by the useful integration formula;
3
(x − 1/6)2
+ ( 47/6)2
1 1 x − 1/6 2 6x − 1
= √ arctan √ + C = √ arctan √ + C.
3 47/6 47/6 47 47

Remark 2.3. When evaluating an integral of the form


Z
1
dx,
ax2 + bx + c
after completing the square, we always arrive at one of the two possibilities
(up to a constant factor):
Z
1 1 x+h
• 2 + d2
dx = arctan + C (as above) or
(x + h) d d
(x + h) − d
Z
1 1
• dx = ln + C.
(x + h)2 − d2 2d (x + h) + d
Example 2.14 (Square Completion).
Z
1
√ dx completing the square:
1 + 5x − x2
May 31, 2018 10:33 ws-book9x6 Integration for Calculus, Analysis, and Differential Equations ws-book9x6 page 29

Direct Integration 29

   2
2 2 5 25 25 29 5
1 + 5x − x = − x − 2 x + − −1 = − x− ;
2 4 4 4 2
Z
1
= q √ 2 dx by the useful integration formula;
29/2 − (x − 5/2)2
x − 5/2 2x − 5
= arcsin √ + C = arcsin √ + C.
29/2 29

Remark 2.4. When evaluating an integral of the form


Z
1
√ dx,
2
ax + bx + c
after completing the square, we always arrive at one of the two possibilities
(up to a constant factor):
Z
1 x+h
• p dx = arcsin + C (as above) or
2
d − (x + h) 2 d
Z
1 p
• p dx = ln |x + h + (x + h)2 ± d2 | + C.
(x + h)2 ± d2

2.3. Direct Integration for Definite Integral

Definition 2.2 (Direct Integration).


By direct integration for definite integral, we understand the process of inte-
gration, which is completely based on the integration rules (Theorem 1.3),
possibly, the Useful Integration Formula (Theorem 2.1), and the Newton-
Leibniz Formula (Theorem 1.6) and makes no use of special integration
methods.

Examples 2.15 (Direct Integration for Definite Integral).


Z 8 √
3
1. x dx switching to the power form;
1
Z 8
= x1/3 dx by Newton-Leibniz Formula;
1
8
3 4/3 3 3 45
= x = (84/3 − 14/3 ) = (16 − 1) = .
4 1 4 4 4
May 17, 2018 12:7 ws-book9x6 Integration for Calculus, Analysis, and Differential Equations ws-book9x6 page 30

30 Integration for Calculus, Analysis, and Differential Equations

Z π
sin x − 3e−x dx

2. by the integration rules;
Z 0π Z π
= sin x dx − 3 e−x dx
0 0
by the useful integration formula and the Newton-Leibniz Formula;
π π
= − cos x + 3e−x = −(cos π − cos 0) + 3 e−π − e0

0 0
= −(−1 − 1) + 3(e−π − 1) = 3e−π − 1.

2
(x + 1)2
Z
3. dx squaring and dividing termwise;
1 x
Z 2 Z 2
x2 + 2x + 1

1
= dx = x+2+ dx
1 x 1 x
by the integration rules and the Newton-Leibniz Formula;
 2
 2
x 4 1 7
= + 2x + ln |x| = + 2 · 2 + ln 2 − − 2 − ln 1 = + ln 2.
2 1 2 2 2

(
−1, 0 ≤ x < 1,
4. For f (x) = , by additivity (Theorem 1.3),
1≤x≤3 2x ,
Z 3 Z 1 Z 3
f (x) dx = (−1) dx + 2x dx
0 0 1
by the integration rules and the Newton-Leibniz Formula;
1 x
3
2
= −(1 − 0) + (23 − 21 )/ ln 2 = −1 + 6/ ln 2.

= −x +

ln 2 0 1

Z 2
5. |2t − 1| dt
−1
(
−(2t − 1), −1 ≤ t ≤ 1/2,
since |2t − 1| = , by additivity (Theorem 1.3);
2t − 1, 1/2 ≤ t ≤ 2
Z 1/2 Z 2
= [−(2t − 1)] dt + (2t − 1) dt
−1 1/2

by the integration rules and the Newton-Leibniz Formula;


May 17, 2018 12:7 ws-book9x6 Integration for Calculus, Analysis, and Differential Equations ws-book9x6 page 31

Direct Integration 31

Z 1/2 Z 2 1/2 2
2 2

=− (2t − 1) dt + (2t − 1) dt = −(t − t) + (t − t)

−1 1/2 −1 1/2

= − [(1/4 − 1/2) − (1 − (−1))] + [(4 − 2) − (1/4 − 1/2)]


= 1/4 + 2 + 2 + 1/4 = 9/2.

2.4. Applications

Examples 2.16 (Applications).

1. Find the solution of the initial value problem


1
y0 = x − , y(−1) = 1.
x
Solution: Integrating by the rules only, we find:
x2
Z   Z Z
1 1
y= x− dx = x dx − dx = − ln |x| + C
x x 2
on each of the intervals (−∞, 0), (0, ∞), where C is an arbitrary real
constant. Since the initial condition is given at x = −1, the interval of
interest for us is (−∞, 0).
Hence, all possible solutions (the general solution) of the differential
equation on (−∞, 0) are given by the formula
x2
y= − ln |x| + C
2
with an arbitrary real C.
We find the solution of the initial value problem by plugging in x = −1
and solving the obtained equation for C:
(−1)2
y(−1) = − ln | − 1| + C = 1 ⇔ 1/2 + C = 1 ⇔ C = 1/2.
2
Hence, the desired solution is
y = x2 /2 − ln |x| + 1/2, x < 0.
2. Find the volume of the solid of generated by revolving the region
bounded by

y = x and y = x3
(a) about the x-axis,
(b) about the y-axis.
May 31, 2018 10:33 ws-book9x6 Integration for Calculus, Analysis, and Differential Equations ws-book9x6 page 32

32 Integration for Calculus, Analysis, and Differential Equations

Solution: Let us first find where the graphs intersect by solving the
equation

x = x3 ⇔ x3 − x1/2 = 0 ⇔ x1/2 (x5/2 − 1) = 0 ⇔ x = 0 or x = 1.

Remark 2.5. Had we divided through by x, we would have lost the
solution x = 0.

Hence, the region is bounded by the graphs of y = x and y = x3 over
the interval [0, 1]. Observe that

x ≥ x3 on [0, 1].

(a) Since the rotation axis is the axis of definition, we apply the washer
method (see, e.g., [1, 6]):

Z 1 √
Z 1
π ( x)2 − (x3 )2 dx = π x − x6 dx
  
V =
0 0
by the integration rules and the Newton-Leibniz Formula;
Z 1  2  1
x7
 2
17

x 1 5π
x − x6 dx = π un.3 .
 
=π − = π − =
0 2 7
0 2 7 14

(b) Since the rotation axis is perpendicular to the axis of definition, we


apply the shell method (see, e.g., [1, 6]):

Z 1
x − x3 dx
√ 
V = 2πx
0
switching to the power form and multiplying;
Z 1 h i Z 1 h i
1/2 3
= 2πx x − x dx = 2π x3/2 − x4 dx
0 0
by the integration rules and the Newton-Leibniz Formula;
Z 1h  1
x5 2 5/2 15
  
i 2
= 2π x3/2 − x4 dx = 2π x5/2 − = 2π 1 −
0 5 5 0 5 5

= un.3 .
5
May 17, 2018 12:7 ws-book9x6 Integration for Calculus, Analysis, and Differential Equations ws-book9x6 page 33

Direct Integration 33

2.5. Practice Problems

Evaluate the integrals.


Z Z
1 1
1. √ dx 16. dx
5x − 6 x2 + 2x − 3
Z
1
Z
10 17. dx
2. (2 − 3x) dx
4x2 + 4x + 3
Z
1
Z  
1 √
3. 3x2 + 2x − dx 18. dx
x Z 5 − 4x − x2
Z 
e−x
 1
4. ex 1 − 2 dx 19. √ dx
x 3x2 − 2x − 1
1 − cos3 x
Z 2 Z
x +2 20. dx
5. dx cos2 x
x4 Z
Z
2x2 + x + 1 cos 2x
6. dx 21. 2 dx
x−1 sin x cos2 x
Z √
Z
( x − 1)3 22. (cos x + sin x)2 dx
7. dx
x Z
√ sin2 (x/2 + 1) dx
Z
23.
8. x 3 2x − 1 dx
Z
Z
3x + 2 24. cos 5x cos 3x dx
9. dx
2x + 3 Z
1
x4
Z
25. dx
10. dx 1 + cos x
x2 + 2 Z

3x3 − 5x + 2 26. 1 + sin 2x dx
Z
11. dx
x+2 Z 2
(x − 1)6 dx
Z
27.
12. (2x + 3x )2 dx 1

Z √ 4
Z 4
1− x
x + x−4 + 2 28. dx
13. dx 1 x
x3
√ √ Z 1 3
x +1
1 + x2 + 1 − x2
Z
29. dx
14. √ dx 0 x+1
Z 1 − x4 Z π/2
1 30. cot2 x dx
15. √ √ dx
x+4+ x π/4
b2530   International Strategic Relations and China’s National Security: World at the Crossroads

This page intentionally left blank

b2530_FM.indd 6 01-Sep-16 11:03:06 AM


May 17, 2018 12:7 ws-book9x6 Integration for Calculus, Analysis, and Differential Equations ws-book9x6 page 35

Chapter 3

Method of Substitution

3.1. Substitution for Indefinite Integral

3.1.1. What for? Why? How?


What Is the Method for?
The Method of Substitution for indefinite integral is used for finding inte-
grals of the form:
Z
f (g(x))g 0 (x) dx,
e.g.,
Z
10sin x cos x dx

with f (u) = 10u and g(x) = sin x.

Why Does the Method Work?


The method is based on the following
Theorem 3.1 (Substitution Rule for Indefinite Integral).
Let
• a function f have an antiderivative F on an interval I and
• a function g be differentiable on an interval J and take values in I.
Then on the interval J, with the substitution u = g(x),
Z Z
0
f (g(x))g (x) dx = f (u) du.

Proof. Since
Z
f (u) du = F (u) + C on I,

35
May 17, 2018 12:7 ws-book9x6 Integration for Calculus, Analysis, and Differential Equations ws-book9x6 page 36

36 Integration for Calculus, Analysis, and Differential Equations

with the substitution u = g(x), we are to show that


Z
f (g(x))g 0 (x) dx = F (g(x)) + C on J,

which is readily verified by the Chain Rule:


d
F (g(x)) = F 0 (g(x))g 0 (x) = f (g(x))g 0 (x) on J.
dx

Remark 3.1. Thus, the Substitution Rule for Indefinite Integral is the
integral form of the Chain Rule for differentiation (see, e.g., [1, 6]).

How Does the Method Work?


An integral of the form
Z
f (g(x))g 0 (x) dx,

with functions f and g satisfying the conditions of the Substitution Rule, is


found in the following three steps:
Z
f (g(x))g 0 (x) dx

Step 1. Substitute u = g(x), du = g 0 (x) dx


Z
obtaining an integral relative to the new variable: = f (u) du

Step 2. Integrate relative to the new variable u: = F (u) + C


Step 3. Substitute back returning to the old variable x: = F (g(x)) + C.

3.1.2. Perfect Substitution


When the Substitution Method can be applied directly, without manipulat-
ing the integrand, we have the case of a perfect substitution.
Examples 3.1 (Perfect Substitution).
Z
1. 10sin x cos x dx substituting: u = sin x, du = cos x dx ;
Z
= 10u du integrating relative to the new variable;
May 17, 2018 12:7 ws-book9x6 Integration for Calculus, Analysis, and Differential Equations ws-book9x6 page 37

Method of Substitution 37

10u
= +C substituting back ;
ln 10
10sin x
= + C.
ln 10

Z
cos x
2. cot x dx by the trigonometric identity, cot x = ;
sin x
Z
cos x
= dx substituting: u = sin x, du = cos x dx ;
sin x
Z
1
= du integrating relative to the new variable;
u
= ln |u| + C substituting back ;
= ln | sin x| + C (Table of Basic Integrals (Appendix A), integral 12).

ex
Z Z
1
3. 2x
dx = 2 ex dx substituting: u = ex , u = ex dx ;
e +1 (ex ) + 1
Z
1
= du integrating relative to the new variable;
u2 + 1
= arctan u + C substituting back ;
x
= arctan e + C.

Z
ln x 1
4. dx substituting: u = ln x, du = dx ;
x x
Z
= u du integrating relative to the new variable;

u2
= +C substituting back ;
2
2
ln x
= + C.
2

3.1.3. Introducing a Missing Constant

Sometimes, an integral falls short of being the case of a perfect substitution


by a missing constant factor in the integrand only.
This can be easily fixed: the missing constant can be introduced via mul-
May 17, 2018 12:7 ws-book9x6 Integration for Calculus, Analysis, and Differential Equations ws-book9x6 page 38

38 Integration for Calculus, Analysis, and Differential Equations

tiplying and dividing by it. After such a minor adjustment, we arrive at a


perfect substitution.

Examples 3.2 (Introducing a Missing Constant).

Z
sin x
1. tan x dx since tan x = ;
cos x
Z
1
= sin x dx
cos x
since cos0 x = − sin x, introducing the missing constant −1;
Z
1
=− (− sin x) dx substituting: u = cos x, du = − sin x dx ;
cos x
Z
1
=− du integrating relative to the new variable;
u
= − ln |u| + C substituting back ;
= − ln | cos x| + C = ln | sec x| + C
(Table of Basic Integrals (Appendix A), integral 11).

31/x
Z
2. dx
x2
 0
1 1
since = − 2 , introducing the missing constant −1;
x x
Z  
1 1 1
= − 31/x − 2 dx substituting: u = , du = − 2 dx ;
x x x
Z
= − 3u du integrating relative to the new variable;
3u
=− +C substituting back ;
ln 3
31/x
=− + C.
ln 3

Z Z
x
3. √
3
dx = (x2 − 1)−1/3 x dx
x2 − 1
since (x2 − 1)0 = 2x, introducing the missing constant 2;
Z
1
= (x2 − 1)−1/3 2x dx substituting: u = x2 − 1, du = 2x dx ;
2
May 17, 2018 12:7 ws-book9x6 Integration for Calculus, Analysis, and Differential Equations ws-book9x6 page 39

Method of Substitution 39

Z
1
= u−1/3 du integrating relative to the new variable;
2
1 3 2/3 3
= u + C = u2/3 + C substituting back ;
22 4
3 2 2/3
= (x − 1) + C.
4

Z
4. sin(1 − 5x4 )x3 dx

since (1 − 5x4 )0 = −20x3 , introducing the missing constant −20;


Z
1
− sin(1 − 5x4 ) (−20x3 ) dx
 
=
20
substituting: u = 1 − 5x4 , du = −20x3 dx ;
Z
1
= [− sin u] du integrating relative to the new variable;
20
1
= cos u + C substituting back ;
20
1
= cos(1 − 5x4 ) + C.
20


Z Z
5. 3x + 14 dx = (3x + 14)1/2 dx

since (3x + 14)0 = 3, introducing the missing constant 3;


Z
1
= (3x + 14)1/2 3 dx substituting: u = 3x + 14, du = 3 dx ;
3
Z
1
= u1/2 du integrating relative to the new variable;
3
1 2 3/2 2
= u + C = u3/2 + C substituting back ;
33 9
2 3/2
= (3x + 14) + C.
9

3.1.4. Trivial Substitution


The last problem is a typical example of an integral of the form
Z
f (ax + b) dx,

where a, b are real coefficients with a 6= 0.


May 17, 2018 12:7 ws-book9x6 Integration for Calculus, Analysis, and Differential Equations ws-book9x6 page 40

40 Integration for Calculus, Analysis, and Differential Equations

In such a case, a perfect substitution is readily attained by introducing the


missing constant a:
Z
f (ax + b) dx introducing the missing constant a;
Z
1
= f (ax + b)a dx substituting: u = ax + b, du = a dx ;
a
Z
1
= f (u) du integrating relative to the new variable;
a
1
= F (u) + C substituting back ;
a
1
= F (ax + b) + C.
a

We call such a routine substitution trivial. It is recommended to bypass a


trivial substitution by applying the Useful Integration Formula instead:
Z
1
f (ax + b) dx = F (ax + b) + C
a
(see Theorem 2.1 and accompanying Examples 2.1).
For instance, the last problem in the preceding examples can be solved as
follows:

Z Z
12 2
3x + 14 dx = (3x+14)1/2 dx = (3x+14)3/2 +C = (3x+14)3/2 +C.
33 9

3.1.5. More Than a Missing Constant


Sometimes, conducting a substitution calls for more than just noticing and
introducing a missing constant factor in the integrand.
Examples 3.3 (More Than a Missing Constant).

x5
Z
1. 3
dx splitting x2 off x5 ;
x +4
x3
Z
= 3
x2 dx
x +4
since (x3 + 4)0 = 3x2 , introducing the missing constant 3;
x3
Z
1
= 3
3x2 dx
3 x +4
substituting: u = x3 + 4, x3 = u − 4, du = 3x2 dx ;
May 17, 2018 12:7 ws-book9x6 Integration for Calculus, Analysis, and Differential Equations ws-book9x6 page 41

Method of Substitution 41

u−4
Z
1
= du dividing termwise;
3 u
Z  
1 4
= 1− du integrating relative to the new variable;
3 u
Z Z 
1 1 1
= 1 du − 4 du = [u − 4 ln |u|] + B substituting back ;
3 u 3
1 3 1 4
x + 4 − 4 ln |x3 + 4| +B = x3 − ln |x3 +4|+C (C = B +4/3).

=
3 3 3


Z Z
2
2. x 3
x + 1 dx = x2 (x + 1)1/3 dx

substituting: u = x + 1, x = u − 1, dx = du ;
Z
= (u − 1)2 u1/3 du multiplying;
Z Z h i
= (u2 − 2u + 1)u1/3 du = u7/3 − 2u4/3 + u1/3 du

integrating relative to the new variable;


Z Z Z
3 10/3 6 7/3 3 4/3
= u7/3 du − 2 u4/3 du + u1/3 du = u − u + u +C
10 7 4
substituting back ;
3 6 3
= (x + 1)10/3 − (x + 1)7/3 + (x + 1)4/3 + C
10 7 4
3 4/3
 2

= (x + 1) 14(x + 1) − 40(x + 1) + 35 + C
140
3
(x + 1)4/3 14x2 − 12x + 9 + C.
 
=
140

Z
1
3. dx multiplying and dividing by ex ;
ex − 2

ex
Z
= dx substituting: u = ex , du = ex dx ;
ex (ex
− 2)
Z
1
= du transforming product into sum:
u(u − 2)
 
1 1 1 1
= − (cf. Examples 2.10);
u(u − 2) 2 u−2 u
May 17, 2018 12:7 ws-book9x6 Integration for Calculus, Analysis, and Differential Equations ws-book9x6 page 42

42 Integration for Calculus, Analysis, and Differential Equations

Z  
1 1 1
= − du integrating relative to the new variable;
2 u−2 u
Z Z 
1 1 1 1
= du − du = [ln |u − 2| − ln |u|] + C
2 u−2 u 2
substituting back ;
1 1
= [ln |ex − 2| − ln ex ] + C = [ln |ex − 2| − x] + C.
2 2

Z
4. sec x dx multiplying and dividing by sec x + tan x;
Z Z
sec x(sec x + tan x) 1
= dx = (sec x tan x + sec2 x) dx
sec x + tan x sec x + tan x
substituting: u = sec x + tan x, du = (sec x tan x + sec2 x) dx ;
Z
1
= du integrating relative to the new variable;
u
= ln |u| + C substituting back ;
= ln | sec x + tan x| + C
(Table of Basic Integrals (Appendix A), integral 13).

Exercise 3.1 (Deriving Table Integrals).


In the similar manner, obtain integral 14 of the Table of Basic Integrals
(Appendix A).

3.1.6. More Than One Way

There may exist multiple approaches to finding an indefinite integral, some


of which involve substitution as in the following case.

Example 3.4 (More Than One Way).


Evaluate the integral


Z
x 5 − 2x dx.

Approach 1 (Direct Integration): This approach steers clear of using


substitution. A complete solution is given in Examples 2.5.
May 17, 2018 12:7 ws-book9x6 Integration for Calculus, Analysis, and Differential Equations ws-book9x6 page 43

Method of Substitution 43

Approach 2 (Substitution):
√ 5−u
Z
1
x 5 − 2x dx substituting: u = 5 − 2x, x = , dx = − du ;
2 2
 
5 − u 1/2
Z
1
= u − du multiplying;
2 2
Z Z
1 1 3/2
= (u − 5)u1/2 du = (u − 5u1/2 ) du
4 4
integrating relative to the new variable;
Z Z Z 
1 1
= (u3/2 − 5u1/2 ) du = u3/2 du − 5 u1/2 du
4 4
 
1 2 5/2 10 3/2
= u − u +C substituting back ;
4 5 3
1 5
= (5 − 2x)5/2 − (5 − 2x)3/2 + C.
10 6

Approach 3 (Another Substitution): Let us use the substitution u =



5 − 2x now.

Z
x 5 − 2x dx

√ 5 − u2
substituting: u= 5 − 2x, x = , dx = −udu ;
2
5 − u2
Z
= u(−u) du multiplying;
2
Z
1 4
= (u − 5u2 ) du integrating relative to the new variable;
2
Z Z   
1 4 2 1 1 5 5 3
= u du − 5 u du = u − u +C substituting back ;
2 2 5 3
1 5
= (5 − 2x)5/2 − (5 − 2x)3/2 + C.
10 6

3.1.7. More Than One Substitution


Substitutions may be applied repeatedly.
Examples 3.5 (More Than One Substitution).
Z
1. x sin3 (x2 ) cos(x2 ) dx introducing the missing constant 2;
May 17, 2018 12:7 ws-book9x6 Integration for Calculus, Analysis, and Differential Equations ws-book9x6 page 44

44 Integration for Calculus, Analysis, and Differential Equations

Z
1
= sin3 (x2 ) cos(x2 )2x dx substituting: u = x2 , du = 2xdx ;
2
Z
1
= sin3 u cos u du substituting again: v = sin u, dv = cos u du ;
2
Z
1
= v 3 dv integrating relative to the new variable;
2
1 4
= v +C substituting back ;
8
1 1
= sin4 u + C = sin4 (x2 ) + C
8 8

sin3 x cos x
Z
2. dx substituting: u = sin x, du = cos xdx ;
sin2 x + 1
u3 u3
Z
u
= 2
du by long division, 2
=u− 2 (verify);
u +1 u +1 u +1
Z Z
u
= u du − du introducing the missing constant 2;
u2 + 1
u2
Z
1 2u
= − 2
du substituting again: v = u2 + 1, dv = 2u du ;
2 2 u +1
u2
Z
1 1
= − dv integrating relative to the new variable;
2 2 v
u2 1
= − ln |v| + C substituting back ;
2 2
sin2 x 1
= − ln(sin2 x + 1) + C.
2 2
Remark 3.2. When two or more consecutive substitutions can be applied,
a single substitution, perhaps a bulkier one, can always be applied instead.

Thus, the second integral in the prior examples can also be evaluated via a
single substitution as follows:
Example 3.6 (Last Integral Revisited).
sin3 x cos x
Z
dx splitting sin x off sin3 x;
sin2 x + 1
sin2 x
Z
= sin x cos x dx introducing the missing constant 2;
sin2 x + 1
sin2 x
Z
1
= 2 sin x cos x dx
2 sin2 x + 1
May 17, 2018 12:7 ws-book9x6 Integration for Calculus, Analysis, and Differential Equations ws-book9x6 page 45

Method of Substitution 45

substituting: u = sin2 x + 1, sin2 x = u − 1, du = 2 sin x cos x dx ;


u−1
Z
1
= du dividing termwise;
2 u
Z  
1 1
= 1− du integrating relative to the new variable;
2 u
Z Z 
1 1 1
= 1 du − du = [u − ln |u|] + C substituting back ;
2 u 2
1 2 1 1
sin x + 1 − ln(sin2 x + 1) + C = sin2 x − ln(sin2 x + 1) + B

=
2 2 2
(B = C + 1/2).

Exercise 3.2 (Alternate Approach).


Evaluate the first integral in the prior examples via the single substitution
u = sin(x2 ).

3.2. Substitution for Definite Integral

3.2.1. What for? Why? How?

What Is the Method for?


The Method of Substitution for definite integral is used for finding integrals
of the form:
Z b
f (g(x))g 0 (x) dx,
a

e.g.,
Z e

ln x + 1
dx
1 x

with f (u) = u and g(x) = ln x + 1.

Why Does the Method Work?


The method is based on the following

Theorem 3.2 (Substitution Rule for Definite Integral).


Let

• a function f be continuous on an interval I and


May 17, 2018 12:7 ws-book9x6 Integration for Calculus, Analysis, and Differential Equations ws-book9x6 page 46

46 Integration for Calculus, Analysis, and Differential Equations

• a function g be continuously differentiable on an interval [a, b] and take


values in I.
Then, with the substitution u = g(x),
Z b Z g(b)
f (g(x))g 0 (x) dx = f (u) du.
a g(a)

Proof. Observe that the integrals on both sides of the equality exist since
the integrands are functions continuous on the corresponding intervals.
Since f is continuous on I, by the Fundamental Theorem of Calculus
(Part 1) (Theorem 1.5), it has an antiderivative F (x) on I.
By the Substitution Rule for Indefinite Integral (Theorem 3.1),
Z
f (g(x))g 0 (x) dx = F (g(x)) + C.

Hence, by the Newton-Leibniz Formula (Theorem 1.6),


Z b b g(b)
0

f (g(x))g (x) dx = F (g(x)) = F (g(b)) − F (g(a)) = F (u)

a a g(a)
Z g(b)
= f (u) du.
g(a)

How Does the Method Work?


An integral of the form
Z b
f (g(x))g 0 (x) dx,
a

with functions f (x) and g(x) satisfying the conditions of the Substitution
Rule, is found in the following three steps:
Z b
f (g(x))g 0 (x) dx
a

Step 1. Substitute and change the integration limits

x u
0
u = g(x), du = g (x) dx a g(a) :
b g(b)
May 17, 2018 12:7 ws-book9x6 Integration for Calculus, Analysis, and Differential Equations ws-book9x6 page 47

Method of Substitution 47

Z g(b)
= f (u) du
g(a)

Step 2. Integrate (antidifferentiate) relative to the new variable u.

Step 3. Use the Newton-Leibniz Formula:


g(b)

= F (u) = F (g(b)) − F (g(a)).
g(a)

Remark 3.3. The Substitution Method for definite integral

• does not require back substitution, but


• requires the change of the integration limits.

Basically, when executing substitution for a definite integral, we act as if we


have an indefinite integral, not forgetting to change the integration limits,
up to the step of back substitution, instead of which, we find the numeric
value of the obtained definite integral relative to the new integration vari-
able and new integration limits by the Newton-Leibniz Formula.

Examples 3.7 (Substitution for Definite Integral).


Z e
√ Z e
ln x + 1 1
1. dx = (ln x + 1)1/2 dx
1 x 1 x
substituting and changing the integration limits:

x u
1
u = ln x + 1, du = dx 1 1 ;
x
e 2
Z 2
= u1/2 du by the Newton-Leibniz Formula;
1
2
2 3/2 2 4√ 2
= u = [23/2 − 1] = 2− .
3 1 3 3 3
Z 1
2
2. xe−x dx introducing the missing constant −2;
0
Z 1
1 2
=− e−x (−2x) dx
2 0
May 17, 2018 12:7 ws-book9x6 Integration for Calculus, Analysis, and Differential Equations ws-book9x6 page 48

48 Integration for Calculus, Analysis, and Differential Equations

substituting and changing the integration limits:

x u
u = −x2 , du = −2x dx 0 0 ;
1 −1
Z −1
1
=− eu du by the Newton-Leibniz Formula;
2 0
−1  
1 u 1 −1 0 1 1
= − e = − (e − e ) = 1− .
2 0 2 2 e

3.3. Applications

Example 3.8 (Application).


Find√the volume of the solid generated by revolving the region bounded by
y = 4 − x2 and the x-axis on the interval [1, 2] about the y-axis.
Solution: Since the rotation axis is perpendicular to the axis of definition,
by the shell method ,
Z 2 p
V = 2πx 4 − x2 dx
1
factoring out π and switching to the power form;
Z 2
=π (4 − x2 )1/2 2x dx introducing the missing constant −1;
1
Z 2
= −π (4 − x2 )1/2 (−2x) dx
1
substituting and changing the integration limits:

x u
u = 4 − x2 , du = (−2x) dx 1 3 ;
2 0
Z 0
= −π u1/2 du by the Newton-Leibniz Formula;
3
0
2 3/2 2 √
= −π u = −π [03/2 − 33/2 ] = 2π 3 un.3 .
3 3 3
May 17, 2018 12:7 ws-book9x6 Integration for Calculus, Analysis, and Differential Equations ws-book9x6 page 49

Method of Substitution 49

3.4. Practice Problems

Evaluate the integrals.

2x − 3
Z Z p
1 − 2x6 x5 dx
3
1. √ dx 14.
2
x − 3x + 4
ex
Z Z
2. √ dx 15. x3 (1 − 3x2 )10 dx
e2x − 1
x2
Z Z
arcsin x
3. √ dx 16. dx
1 − x2 x6 +4
x3
Z
1
Z
4. dx 17. √ dx
x(ln x − 3) x8 − 1
Z
1
Z
x
5. dx 18. dx
x ln x ln | ln x| x2 + x + 1
x−1
Z Z
x
6. 2
dx 19. √ dx
x +2 1 − x2
Z
3x + 2 x5
Z
7. dx 20. √ dx
x2 − 5 1 − x2
Z Z
8. sin x cos x dx 1
21. √ dx
x x 2+1
Z
sin x Z
1
9. dx 22. dx
cos3 x 2
sin x + 2 cos2 x
Z
sin x + cos x Z
sin(1/x)
10. √3
dx 23. dx
sin x − cos x x2
Z
sin x Z
1
11. √ dx 24. √ x dx
Z cos 2x e +1
3 Z 1
12. ex x2 dx arctan x
25. dx
Z √x 0 x2 + 1
5 Z ln 2
13. √ dx 1
x 26. x
dx
0 e +1
b2530   International Strategic Relations and China’s National Security: World at the Crossroads

This page intentionally left blank

b2530_FM.indd 6 01-Sep-16 11:03:06 AM


May 17, 2018 12:7 ws-book9x6 Integration for Calculus, Analysis, and Differential Equations ws-book9x6 page 51

Chapter 4

Method of Integration by Parts

4.1. Partial Integration for Indefinite Integral

4.1.1. What for? Why? How?


What Is the Method for?
The Method of Integration by Parts (or Partial Integration) for indefinite
integral is used for finding integrals of the form:
Z
u(x)v 0 (x) dx,

where u and v are continuously differentiable functions (i.e., functions with


continuous derivatives) on an interval I, e.g.,
Z
ex cos x dx

with u(x) = ex and v(x) = sin x.

Why Does the Method Work?


The method is based on the following
Theorem 4.1 (Integration by Parts Formula for Indefinite
Integral).
Let u and v be continuously differentiable functions on an interval I. Then,
on I,
Z Z
u(x)v 0 (x) dx = u(x)v(x) − v(x)u0 (x) dx.

Proof. Since the functions u and v are continuously differentiable on I,


then so is their product uv and, by the differentiation product rule (see,
e.g., [1, 6]),
0
[u(x)v(x)] = u0 (x)v(x) + u(x)v 0 (x) on I,

51
May 17, 2018 12:7 ws-book9x6 Integration for Calculus, Analysis, and Differential Equations ws-book9x6 page 52

52 Integration for Calculus, Analysis, and Differential Equations

which implies that uv is an antiderivative of the function u0 v + uv 0 on I.


Thus,
Z
 0
u (x)v(x) + u(x)v 0 (x) dx = u(x)v(x) + C on I.


Since the functions u0 v and uv 0 are continuous on I, by the Fundamental


Theorem of Calculus (Part 1) (Theorem 1.5), they are integrable on I and,
by the integration rules (Theorem 1.1),
Z Z
u0 (x)v(x) dx + u(x)v 0 (x) dx = u(x)v(x) + C on I.

Whence, the partial integration formula follows immediately.

Remarks 4.1 (Partial Integration Formula).


• Thus, the Partial Integration Formula is the integral form of the Prod-
uct Rule for differentiation.
• Using the differential notation dv = v 0 (x)dx, du = u0 (x)dx, we can
informally rewrite the partial integration formula in the following easy-
to-remember shorthand fashion:
Z Z
u dv = uv − v du.

How Does the Method Work?


Partial integration for indefinite integral consists in applying the integra-
tions by parts formula and works nicely for the following three special types
of integrals (and not only for them).

4.1.2. Three Special Types of Integrals


Type 1 Integrals
Type 1 integrals are of the form
Z Z Z
Pn (x)eax+b dx, Pn (x) sin(ax + b) dx, or Pn (x) cos(ax + b) dx

where
Pn (x) = an xn + an−1 xn−1 + · · · + a1 x + a0
is a polynomial of degree n = 1, 2, . . . with real coefficients an , . . . , a0 ,
an 6= 0, a and b are real coefficients, a 6= 0.
May 17, 2018 12:7 ws-book9x6 Integration for Calculus, Analysis, and Differential Equations ws-book9x6 page 53

Method of Integration by Parts 53

To such integrals, we apply partial integration precisely n times setting on


each step
u = polynomial factor and dv = [exponential/trigonometric factor ] dx.
Example 4.1 (Type 1 Integrals).
Z
(x2 + 3x)e2x dx

u = x2 + 3x, du = (2x + 3) dx
integrating by parts: 1 ;
dv = e2x dx, v = e2x dx = e2x
R
2
Z
1 1
= (x + 3x) e2x −
2
(2x + 3)e2x dx
2 2
u = 2x + 3, du = 2 dx
integrating by parts once more: 1 ;
dv = e2x dx, v = e2x
2
 Z 
1 2 1 1
= (x + 3x)e2x − (2x + 3)e2x − e2x dx
2 2 2
 
1 1 1 1
= (x2 + 3x)e2x − (2x + 3)e2x − e2x + C simplifying;
2 2 2 2
 
1 2 1 1 2x 1 2
x + 2x − 1 e2x + C.

= x + 3x − (2x + 3) + e +C =
2 2 2 2

Type 2 Integrals
Type 2 integrals are of the form
Z
Pn (x) ln(ax + b) dx,
Z Z
Pn (x) arcsin(ax + b) dx, Pn (x) arccos(ax + b) dx,
Z Z
Pn (x) arctan(ax + b) dx, or Pn (x) arccot(ax + b) dx,

where
Pn (x) = an xn + an−1 xn−1 + · · · + a1 x + a0
is a polynomial of degree n = 0, 1, 2, . . . with real coefficients an , . . . , a0 ,
an 6= 0, a and b are real coefficients, a 6= 0.
May 17, 2018 12:7 ws-book9x6 Integration for Calculus, Analysis, and Differential Equations ws-book9x6 page 54

54 Integration for Calculus, Analysis, and Differential Equations

To such integrals, we apply partial integration once setting


u = transcendental factor and dv = Pn (x) dx.
Example 4.2 (Type 2 Integrals).

1
Z u = ln x, du = dx
x ln x dx integrating by parts: x
R x2 ;
dv = xdx, v = x dx =
2
x2 x2 x2 x2
Z Z
1 21 1
= ln x − x dx = ln x − x dx = ln x − +C
2 2 x 2 2 2 4
x2
= (2 ln x − 1) + C.
4

Type 3 Integrals
Type 3 integrals are of the form
Z Z
eax+b sin(cx + d) dx or eax+b cos(cx + d) dx

where a, b, c, and d are real coefficients, a, c 6= 0.


To such integrals, we apply partial integration twice setting on each step
u = exponential factor and dv = [trigonometric factor ] dx
or vice versa and arrive at a linear algebraic equation relative to the un-
known integral, solving which we find the integral.
Example 4.3 (Type 3 Integrals).
Z
e−x sin x dx

u = e−x , du =R −e−x dx
integrating by parts: ;
dv = sin x dx, v = sin x dx = − cos x
Z
= −e−x cos x − e−x cos x dx

u = e−x , du =R −e−x dx
integrating by parts once more: ;
dv = cos x dx, v = cos x dx = sin x
 Z 
= −e−x cos x − e−x sin x + e−x sin x dx .
May 17, 2018 12:7 ws-book9x6 Integration for Calculus, Analysis, and Differential Equations ws-book9x6 page 55

Method of Integration by Parts 55

Thus, we arrive at the following equation for the unknown integral:


Z Z
e−x sin x dx = −e−x (cos x + sin x) − e−x sin x dx,

solving which, we obtain


Z
1
e−x sin x dx = − e−x (cos x + sin x) + C.
2

4.1.3. Beyond Three Special Types


As the following examples demonstrate, the applicability of partial integra-
tion extends beyond the three considered types of integrals.
Examples 4.4 (Beyond Three Special Types).
Z
1. ln(x2 + 1) dx applying type 2 partial integration scenario:

2x
u = ln(x2 + 1), du = dx
x2 + 1 ;
dv = dx, v=x
Z 2
x
= x ln(x2 + 1) − 2 2
dx integrating directly;
x +1
Z 2 Z  
2 x +1−1 2 1
= x ln(x + 1) − 2 dx = x ln(x + 1) − 2 1− 2 dx
x2 + 1 x +1
Z Z 
1
= x ln(x2 + 1) − 2 1 dx − 2
dx
x +1
= x ln(x2 + 1) − 2 [x − arctan x] + C.

Z
arcsin x
2. √ dx applying type 2 partial integration scenario:
1+x
1
u = arcsin x, du = √ dx
1 − x2 ;
1
dx, v = (1 + x)−1/2 dx = 2(1 + x)1/2
R
dv = √
1+x
Z √
√ √
Z
1+x 1
= 2 1 + x arcsin x−2 √ dx = 2 1 + x arcsin x−2 √ dx
1−x 2 1−x
√ √ √
Z
= 2 1 + x arcsin x−2 (1−x)−1/2 dx = 2 1 + x arcsin x+4 1 − x+C.
May 17, 2018 12:7 ws-book9x6 Integration for Calculus, Analysis, and Differential Equations ws-book9x6 page 56

56 Integration for Calculus, Analysis, and Differential Equations

Z
3. x tan2 x dx

using the relevant trigonometric identity: tan2 x = sec2 x − 1 ;


Z Z Z
= x(sec2 x − 1) dx = x sec2 x dx − x dx

u = x, du =R dx
integrating by parts: ;
dv = sec2 x dx, v = sec2 x dx = tan x

x2 x2
Z
= x tan x − tan x dx − = x tan x − ln | sec x| − + C.
2 2

4. For a > 0,
Z p
x2 ± a2 dx
√ x
u= x2 ± a2 , du = √ dx
integrating by parts: x2 ± a2 ;
dv = dx, v=x
x2
p Z
=x x2 ± a2 − √ dx
x2 ± a2
adding/subtracting and subtracting/adding a2 ;
x2 ± a2 ∓ a2
p Z
=x x2 ± √ a2 − dx dividing termwise;
x2 ± a2
a2
p Z p 
= x x2 ± a2 − x2 ± a2 ∓ √ dx
x2 ± a2
by the integration rules;
Z Z
p p 1
= x x2 ± a2 − x2 ± a2 dx ± a2 √ dx
x ± a2
2
p Z p p
= x x2 ± a2 − x2 ± a2 dx ± a2 ln |x + x2 ± a2 |.

Thus, we arrive at the following equation for the unknown integral:


Z p p Z p p
x2 ± a2 dx = x x2 ± a2 − x2 ± a2 dx ± a2 ln |x + x2 ± a2 |,

solving which, we obtain


May 17, 2018 12:7 ws-book9x6 Integration for Calculus, Analysis, and Differential Equations ws-book9x6 page 57

Method of Integration by Parts 57

a2
Z p
xp 2 p
x2 ± a2 dx = x ± a2 ± ln |x + x2 ± a2 | + C.
2 2

4.1.4. Reduction Formulas


When integrating
Z
x3 ex dx,

we apply partial integration repeatedly, progressively reducing the power of


x until we arrive at the easy-to-evaluate integral
Z
ex dx = ex + C.

Each step in this process is executed under the same pattern, which can be
described by a corresponding reduction formula, i.e., an integral identity,
in which the power in the integrand is reduced. When applied repeatedly,
such formula progressively simplifies the integral until it can be evaluated
directly.
Reduction formulas become really handy when dealing with integrals con-
taining a high power. Some of them can be found in Appendix B. Let us
establish a few here.
Example 4.5 (Reduction Formulas).
Appendix B, formula 8. For a 6= 0 and n = 1, 2, . . . ,
Z
xn eax dx applying type 1 partial integrating scenario:

u = xn , du = nxn−1 dx
1 ;
dv = eax dx, v = eax dx = eax
R
a
xn eax
Z
n
= − xn−1 eax dx.
a a

Applying the obtained reduction formula threefold to the integral


Z
x3 ex dx

(a = 1 and n = 3), we have:


May 17, 2018 12:7 ws-book9x6 Integration for Calculus, Analysis, and Differential Equations ws-book9x6 page 58

58 Integration for Calculus, Analysis, and Differential Equations

Z Z  Z 
x3 ex dx = x3 ex − 3 x2 ex dx = x3 ex − 3 x2 ex − 2 xex dx
 Z 
= x3 ex − 3x2 ex + 6 xex − ex dx = x3 − 3x2 + 6x − 6 ex + C.
 

Exercise 4.1 (Reduction Formulas).


Prove reduction formulas 9 and 10 of Appendix B.

Examples 4.6 (Reduction Formulas).

1. Appendix B, formula 7. For n = 1, 2, . . . ,


Z
lnn x dx

applying type 2 partial integrating scenario:


1
u = lnn x, du = n lnn−1 x dx
x ;
dv = dx, v = x
Z
= x lnn x − n lnn−1 x dx.

2. Appendix B, formula 11. For a > 0 and n = 2, 3, . . . ,


Z
1
n−1 dx integrating by parts:
(x + a2 )
2

1 −(n−1) x
u= n−1 = (x2 + a2 ) , du = −2(n − 1) n dx
(x2+ a2 ) (x2 + a2 ) ;
dv = dx, v=x
x2
Z
x
= n−1 + (2n − 2) n dx
(x2 + a2 ) (x2 + a2 )
adding and subtracting a2 in the numerator;
Z 2
x x + a2 − a2
= n−1 + (2n − 2) n dx
(x2 + a2 ) (x2 + a2 )
"Z Z #
x 1 2 1
= n−1 + (2n − 2) n−1 dx − a n dx .
(x2 + a2 ) (x2 + a2 ) (x2 + a2 )
Z
1
Thus, we arrive at the following equation for n dx:
(x2 + a2 )
May 31, 2018 10:33 ws-book9x6 Integration for Calculus, Analysis, and Differential Equations ws-book9x6 page 59

Method of Integration by Parts 59

Z Z
1 x 1
n−1 dx = n−1 + (2n − 2) n−1 dx
(x2 + a2 ) (x2 + a2 ) (x2 + a2 )
Z
1
− (2n − 2)a2 n dx,
(x2 + a2 )
solving which, we obtain formula 11 of Appendix B:
Z
1
n dx
(x + a2 )
2
" Z #
1 x 1
= + (2n − 3) n−1 dx + C.
(2n − 2)a2 (x2 + a2 )n−1 (x2 + a2 )

Remark 4.2. The latter formula is useful for the integration of rational
functions (see Sec. 7.2.3). Integration via reduction formulas is also utilized
for trigonometric integrals (see Sec. 5.2.1).

4.2. Partial Integration for Definite Integral

4.2.1. What for? Why? How?


What Is the Method for?
The Method of Integration by Parts (or Partial Integration) for definite
integral is used for evaluation of integrals of the form:
Z b
u(x)v 0 (x) dx,
a

where u and v are continuously differentiable functions on an interval [a, b]


(−∞ < a < b < ∞), e.g.,
Z π
x2 ex dx
0
2 x
with u(x) = x and v(x) = e .

Why Does the Method Work?


The method is based on the following
Theorem 4.2 (Integration by Parts Formula for Definite Integral).
Let u and v be continuously differentiable functions on an interval [a, b]
(−∞ < a < b < ∞). Then
Z b b Z b
u(x)v 0 (x) dx = u(x)v(x) − v(x)u0 (x) dx.

a a a
May 31, 2018 10:33 ws-book9x6 Integration for Calculus, Analysis, and Differential Equations ws-book9x6 page 60

60 Integration for Calculus, Analysis, and Differential Equations

Proof. Since the functions u and v are continuously differentiable on [a, b],
then so is their product uv and, by the differentiation Product Rule (see,
e.g., [1, 6]), uv is an antiderivative of the function u0 v + uv 0 on [a, b] (cf.
the proof of Theorem 4.1).
Hence, by the Newton-Leibniz Formula (Theorem 1.6), we have:
Z b b
 0
u (x)v(x) + u(x)v 0 (x) dx = u(x)v(x) .

a a

Since the functions u0 v and uv 0 are continuous on [a, b], they are (Riemann-)
integrable on [a, b] (Corollary 1.1) and, by the integration rules (Theorem
1.3),
Z b Z b b
0 0

u (x)v(x) dx + u(x)v (x) dx = u(x)v(x) .
a a a

Whence, the partial integration formula follows immediately.

Remark 4.3. Informally, the partial integration formula can be written as


follows:
Z b b Z b

u dv = uv − v du.
a a a

How Does the Method Work?


Partial integration for definite integral, as well as for indefinite one, consists
in applying the formula of integrations by parts.
Partial integration works for definite integral whenever and in the same
manner it does for indefinite integral, in particular, for the three special
types of integrals.

Remark 4.4. When applying partial integration to a definite integral, as


opposed to the substitution method, we do not change the integration limits.

Examples 4.7 (Partial Integration for Definite Integral).


Z π
1. (x − π) sin(7x + π) dx
0
applying type 1 partial integration scenario:

u = x − π, du = dx
R 1 ;
dv = sin(7x + π) dx, v = sin(7x + π) dx = − cos(7x + π)
7
May 17, 2018 12:7 ws-book9x6 Integration for Calculus, Analysis, and Differential Equations ws-book9x6 page 61

Method of Integration by Parts 61

π Z π
1 1
= − (x − π) cos(7x + π) + cos(7x + π) dx
7 7 0
0π π
1 1 1
= − (x − π) cos(7x + π) + sin(7x + π) = − π cos π
7 0 49 0 7
1 π
+ (sin 8π − sin π) = .
49 7

Z 1
2. x arctan x dx applying type 2 partial integration scenario:
0
1
u = arctan x, du = 2 dx
x +1 ;
R x2
dv = x dx, v = x dx =
2
1
x2 arctan x 1 1 x2
Z
= − dx integrating directly;
2
0 2 0 x2 + 1
arctan 1 1 1 x2 + 1 − 1 π 1 1
Z Z  
1
= − dx = − 1 − dx
2 2 0 x2 + 1 8 2 0 x2 + 1
" 1 #
Z 1 Z 1  1
π 1 1 π 1
= − 1 dx − 2
dx = − x − arctan x
8 2 0 0 x +1 8 2 0 0
π 1 π 1 h πi π 1 π π 1
= − [1 − arctan 1 + arctan 0] = − 1 − = − + = − .
8 2 8 2 4 8 2 8 4 2

Z π
3. ex cos x dx applying type 3 partial integration scenario:
0

u = ex , du =R ex dx
;
dv = cos x dx, v = cos x dx = sin x
π Z π
x u = ex , du =R ex dx
ex sin x dx

= e sin x − ;
0 0 dv = sin x dx, v = sin x dx = − cos x
 π Z π  Z π
= 0 − −ex cos x + ex cos x dx = −eπ − 1 − ex cos x dx.

0 0 0

Thus, we arrive at the following equation for the unknown integral:


Z π Z π
x π
e cos x dx = −e − 1 − ex cos x dx,
0 0
May 17, 2018 12:7 ws-book9x6 Integration for Calculus, Analysis, and Differential Equations ws-book9x6 page 62

62 Integration for Calculus, Analysis, and Differential Equations

solving which, we obtain


Z π
1
ex cos x dx = − (eπ + 1).
0 2

4.3. Combining Substitution and Partial Integration

Integration often requires combining the methods of substitution and partial


integration.

Examples 4.8 (Combining Substitution and Partial Integration).

Z
1. x3 sin x2 dx splitting x off x3 ;
Z
= x2 sin x2 · x dx introducing the missing constant 2;
Z
1
= x2 sin x2 · 2x dx substituting: y = x2 , dy = 2x dx ;
2
Z
1
= y sin y dy applying type 1 partial integration scenario:
2
u = y, du =R dy
;
dv = sin y dy, v = sin y dy = − cos y
 Z 
1 1
= −y cos y + cos y dy = (−y cos y + sin y) + C
2 2
substituting back ;
1
= (−x2 cos x2 + sin x2 ) + C.
2

√ √
Z
x
2. e dx substituting: y= x, x = y2 , dx = 2ydy ;
Z
=2 yey dy

u = y, du = dy
applying type 1 partial integration scenario: ;
dv = ey dy, v = ey
 Z 
= 2 yey − ey dy = 2 [yey − ey ] + C substituting back ;
May 17, 2018 12:7 ws-book9x6 Integration for Calculus, Analysis, and Differential Equations ws-book9x6 page 63

Method of Integration by Parts 63

√ √ √ √ √
= 2( xe x − e x ) + C = 2( x − 1)e x + C.

Z
3. arcsin x dx applying type 2 partial integration scenario:

1
u = arcsin x, du = √ dx
1 − x2 ;
dv = dx, v=x
Z
x
= x arcsin x − √ dx introducing the missing constant −2;
1 − x2
Z
1 1
= x arcsin x + √ (−2x) dx
2 1 − x2
substituting: y = 1 − x2 , dy = −2x dx ;
Z
1
= x arcsin x + y −1/2 dy = x arcsin x + y 1/2 substituting back ;
2
p
= x arcsin x + 1 − x2 + C.

4.4. Applications

Example 4.9 (Application).


Find the volume of the solid obtained by rotating the region bounded by
y = cos x and the x-axis on the interval [0, π/2] about the y-axis.
Solution: Since the rotation axis is perpendicular to the axis of definition,
by the shell method ,
Z π/2 Z π/2
V = 2πx cos x dx = 2π x cos x dx
0 0
applying type 1 partial integration scenario:

u = x, du =R dx
;
dv = cos x dx, v = cos x dx = sin x
" π/2 Z π/2 #

= 2π x sin x − sin x dx by the Newton-Leibniz Formula;
0 0
" π/2 #

= 2π (π/2) sin(π/2) + cos x = 2π [π/2 + cos(π/2) − cos 0]
0
May 17, 2018 12:7 ws-book9x6 Integration for Calculus, Analysis, and Differential Equations ws-book9x6 page 64

64 Integration for Calculus, Analysis, and Differential Equations

= 2π [π/2 − 1] un.3 .

4.5. Practice Problems

Evaluate the integrals.


Z Z
3 −x x
1. x e dx 11. 2 dx
Z Z sin x
2
2. x2 cos x dx 12. x3 e−x dx
Z Z
3. ln x dx 13. x(arctan x)2 dx
Z Z
arcsin x
4. x arcsec x dx 14. dx
x2
Z Z ln 3
5. e2x sin 3x dx 15. xex dx
Z 0
ln x Z π/2
6. dx
x2 16. x cos 2x dx
Z0 π
Z
7. x sin x cos x dx
17. ex sin x dx
0
Z
8. arctan x dx Z e2 √
Z 18. x ln x dx
1
9. cos(ln x) dx Z π/2
x cos x
Z 19. dx
10. 2
(ln x) dx π/6 sin3 x
Z 2
20. √ x arccsc x dx
2/ 3
May 17, 2018 12:7 ws-book9x6 Integration for Calculus, Analysis, and Differential Equations ws-book9x6 page 65

Chapter 5

Trigonometric Integrals

Here, we are to consider certain approaches to finding trigonometric inte-


grals, i.e., integrals containing trigonometric functions, e.g.,

Z
sin5 x cos x dx.

5.1. Direct Integration

There are many trigonometric integrals, whose evaluation rests exclusively


upon the integration rules and relevant trigonometric identities (see Ap-
pendix C) and makes no use of substitution and partial integration. Several
such examples and types are considered in Chapter 2 (see Examples 2.6,
2.7, 2.8, and 2.9) and here is one more instance of direct integration.
Example 5.1 (Direct Integration).
Z
1
dx
1 − sin x
multiplying and dividing by the conjugate expression 1 + sin x;
Z Z
1 + sin x 1 + sin x
= dx = dx
(1 − sin x)(1 + sin x) 1 − sin2 x
using the trigonometric identity: 1 − sin2 x = cos2 x ;
Z
1 + sin x
= dx dividing termwise and rewriting equivalently;
cos2 x
Z   Z " 2 #
1 sin x 1 1 sin x
= + dx = + · dx
cos2 x cos2 x cos x cos x cos x
1 sin x
using the trigonometric identities again: = sec x, = tan x ;
cos x cos x

65
May 17, 2018 12:7 ws-book9x6 Integration for Calculus, Analysis, and Differential Equations ws-book9x6 page 66

66 Integration for Calculus, Analysis, and Differential Equations

Z
 2 
= sec x + sec x tan x dx by the integration rules;
Z Z
= sec2 x dx + sec x tan x dx = tan x + sec x + C.

5.2. Using Integration Methods

Not all trigonometric integrals can be evaluated directly. There are many
relying on the use of substitution and partial integration. Several such
examples and types are considered in Chapter 3 (see Examples 3.1, 3.2,
3.3, 3.5, and 3.6) and Chapter 4 (see Examples 4.3, 4.4, 4.7, 4.8, and 4.9)
and more are to be considered here.

5.2.1. Integration via Reduction Formulas


Z
As shown in Example 2.7, the trigonometric integral cos2 x dx can be
evaluated directly via an appropriate power-reduction identity (see Ap-
pendix C).
What should we do if we are to evaluate
Z
cosn x dx,

where the exponent n is a sufficiently large positive integer?


Such integrals can be evaluated by special reduction formulas obtained via
partial integration or substitution.
Theorem 5.1 (Reduction Formulas for Trigonometric Integrals).
For n = 2, 3, . . . ,

cosn−1 x sin x n − 1
Z Z
1. cosn x dx = + cosn−2 x dx
n n
sinn−1 x cos x n − 1
Z Z
2. sinn x dx = − + sinn−2 x dx
n n
secn−2 x tan x n − 2
Z Z
3. secn x dx = + secn−2 x dx
n−1 n−1
cscn−2 x cot x n − 2
Z Z
n
4. csc x dx = − + cscn−2 x dx
n−1 n−1
tann−1 x
Z Z
n
5. tan x dx = − tann−2 x dx
n−1
May 17, 2018 12:7 ws-book9x6 Integration for Calculus, Analysis, and Differential Equations ws-book9x6 page 67

Trigonometric Integrals 67

cotn−1 x
Z Z
6. cotn x dx = − − cotn−2 x dx
n−1

Proof. Here, we prove the reduction formulas for


Z Z Z
n n
cos x dx, sec x dx, and tann x dx.

Similar proofs of the remaining ones are left as an exercise to the reader.

Z
cosn x dx splitting cos x off cosn x;
Z
= cosn−1 x cos x dx integrating by parts:

u = cosn−1 x, du =R (n − 1) cosn−2 x(− sin x)dx


;
dv = cos xdx, v = cos x dx = sin x
Z
= cosn−1 x sin x + (n − 1) cosn−2 x sin2 x dx

by the trigonometric identity: sin2 x = 1 − cos2 x ;


Z
= cosn−1 x sin x + (n − 1) cosn−2 x(1 − cos2 x) dx multiplying;
Z
= cosn−1 x sin x + (n − 1)
 n−2
x − cosn x dx

cos

by the integration rules;


Z Z
= cosn−1 x sin x + (n − 1) cosn−2 x dx − (n − 1) cosn x dx.

Thus, we arrive at the following equation for the initial integral:


Z Z Z
cosn x dx = cosn−1 x sin x+(n−1) cosn−2 x dx−(n−1) cosn x dx,

solving which we obtain the reduction formula


cosn−1 x sin x n − 1
Z Z
cosn x dx = + cosn−2 x dx.
n n

Similarly,
Z
secn x dx splitting sec2 x off secn x;
May 17, 2018 12:7 ws-book9x6 Integration for Calculus, Analysis, and Differential Equations ws-book9x6 page 68

68 Integration for Calculus, Analysis, and Differential Equations

Z
= secn−2 x sec2 x dx integrating by parts:

u = secn−2 x, du =R (n − 2) secn−3 x sec x tan x dx


;
dv = sec2 x dx, v = sec2 x dx = tan x
Z
n−2
= sec x tan x − (n − 2) secn−2 x tan2 x dx

by the trigonometric identity: tan2 x = sec2 x − 1 ;


Z
= secn−2 x tan x − (n − 2) secn−2 x(sec2 x − 1) dx

by the integration rules;


Z Z
= secn−2 x tan x − (n − 2) secn x dx + (n − 2) secn−2 x dx.

Thus, we arrive at the following equation for the initial integral:


Z Z Z
n n−2
sec x dx = sec x tan x−(n−2) sec x dx+(n−2) secn−2 x dx,
n

solving which we obtain the reduction formula


secn−2 x tan x n − 2
Z Z
secn x dx = + secn−2 x dx.
n−1 n−1

The next reduction formula is obtained via substitution:


Z
tann x dx splitting tan2 x off tann x;
Z
= tann−2 x tan2 x dx

by the trigonometric identity: tan2 x = sec2 x − 1 ;


Z
= tann−2 x(sec2 x − 1) dx by the integration rules;
Z Z
= tann−2 x sec2 x dx − tann−2 x dx

substituting for the first integral: u = tan x, du = sec2 x dx ;


un−1
Z Z Z
= un−2 du − tann−2 x dx = − tann−2 x dx
n−1
substituting back ;
June 21, 2018 10:23 ws-book9x6 Integration for Calculus, Analysis, and Differential Equations ws-book9x6 page 69

Trigonometric Integrals 69

tann−1 x
Z
= − tann−2 x dx.
n−1

Remarks 5.1 (Reduction Formulas).


• If the exponent n = 2k is a positive even integer, applying the corre-
sponding reduction formula precisely k times, we arrive at
Z Z Z Z
0
cos x dx = sin x dx = sec x dx = csc0 x dx
0 0

Z Z Z
= tan0 x dx = cot0 x dx = 1 dx = x + C.

• If the exponent n = 2k + 1 is a positive odd integer, applying the


corresponding reduction formula precisely k times, we arrive at
Z Z
cos x dx = sin x + C, sin x dx = − cos x + C,
Z Z
sec x dx = ln | sec x+tan x|+C, csc x dx = ln | csc x−cot x|+C,
Z Z
tan x dx = ln | sec x| + C, or cot x dx = ln | sin x| + C.

• Observe that, whenever the reduction formulas for


Z Z Z Z
n n
cos x dx, sin x dx, secn x dx, or cscn x dx

are used, partial integration is implicitly executed and, whenever the


reduction formulas for
Z Z
n
tan x dx, or cotn x dx

are used, substitution is implicitly executed.

Examples 5.2 (Using Reduction Formulas).


Z
1. cos5 x dx applying the appropriate reduction formula twice;

cos4 x sin x 4 cos4 x sin x 4 cos2 x sin x


Z 
= + cos3 x dx = +
5 5 5 5 3
May 17, 2018 12:7 ws-book9x6 Integration for Calculus, Analysis, and Differential Equations ws-book9x6 page 70

70 Integration for Calculus, Analysis, and Differential Equations

Z 
2 1 4 8
+ cos x dx = cos4 x sin x + cos2 x sin x + sin x + C.
3 5 15 15

Z
2. tan6 x dx applying the appropriate reduction formula threefold ;

tan5 x tan5 x tan3 x


Z  Z 
= − tan4 x dx = − − tan2 x dx
5 5 3
5 3
tan5 x tan3 x
Z
tan x tan x
= − + tan x − 1 dx = − + tan x − x + C.
5 3 5 3

Z
5.2.2. Integrals of the Form sinm x cosn x dx

When evaluating a trigonometric integral of the form


Z
sinm x cosn x dx,

where m and n are real exponents, we may encounter three special cases
allowing clear-cut integration strategies. They, however, do not encompass
all the possibilities and do not apply to, say,

Z
sin 2 x cosπ x dx.

The three special cases are as follows.


• Case 1: m is an odd positive integer, i.e., m = 2k + 1 with some
k = 0, 1, . . . and n is an arbitrary real number. In this case, we use the
following integration strategy:
Step 1: split sin x off sin2k+1 x;
Step 2: express the remaining even power sin2k x in terms of
2 2
cos x via the identity sin x = 1 − cos x :
sin2k x = [sin2 x]k = [1 − cos2 x]k ;
Step 3: integrate via the substitution u = cos x.

Example 5.3.

Z
sin5 x cos x dx switching to the power form;
Z
= sin5 x cos1/2 x dx (m = 5, n = 1/2) splitting sin x off sin5 x;
May 17, 2018 12:7 ws-book9x6 Integration for Calculus, Analysis, and Differential Equations ws-book9x6 page 71

Trigonometric Integrals 71

Z
= sin4 x cos1/2 x sin x dx

expressing sin4 x in terms of cos x via sin2 x = 1 − cos2 x ;


Z
= (1 − cos2 x)2 cos1/2 x sin x dx

introducing the missing constant −1;


Z
=− (1 − cos2 x)2 cos1/2 x(− sin x) dx

substituting: u = cos x, du = − sin x dx ;


Z
=− (1 − u2 )2 u1/2 du

integrating directly relative to the new variable;


Z Z
= − (1 − 2u + u )u du = − (u1/2 − 2u5/2 + u9/2 ) du
2 4 1/2

Z Z Z 
=− u1/2 du − 2 u5/2 du + u9/2 du
 
2 4 2
= − u3/2 − u7/2 + u11/2 + C
3 7 11
substituting back and distributing;
2 4 2
= − cos3/2 x + cos7/2 x − cos11/2 x + C.
3 7 11

• Case 2: n is an odd positive integer, i.e., n = 2k + 1 with some


k = 0, 1, . . . and m is an arbitrary real number. In this case, we use the
following integration strategy:
Step 1: split cos x off cos2k+1 x;
Step 2: express the remaining even power cos2k x in terms of
2 2
sin x via the identity cos x = 1 − sin x :

cos2k x = [cos2 x]k = [1 − sin2 x]k ;

Step 3: integrate via the substitution u = sin x.

Example 5.4.
Z
sin4 x cos3 x dx (m = 4, n = 3) splitting cos x off cos3 x;
May 17, 2018 12:7 ws-book9x6 Integration for Calculus, Analysis, and Differential Equations ws-book9x6 page 72

72 Integration for Calculus, Analysis, and Differential Equations

Z
= sin4 x cos2 x cos x dx

expressing cos2 x in terms of sin x via cos2 x = 1 − sin2 x ;


Z
= sin4 x(1 − sin2 x) cos x dx

substituting: u = sin x, du = cos x dx


Z
= u4 (1 − u2 ) du

multiplying and integrating relative to the new variable;


u5 u7
Z Z Z
= (u4 − u6 ) du = u4 du − u6 du = − +C
5 7
substituting back ;
sin5 x sin7 x
= − + C.
5 7

In Examples 5.2, the integral


Z
cos5 x dx

is evaluated via the appropriate reduction formula. Now, let us apply


the described new strategy to it.
Example 5.5 (Alternate Approach).
Z
cos5 x dx (m = 0, n = 5) splitting cos x off cos5 x;
Z
= cos4 x cos x dx

expressing cos4 x in terms of sin x via cos2 x = 1 − sin2 x ;


Z
= (1 − sin2 x)2 cos x dx

substituting: u = sin x, du = cos x dx ;


Z
= (1 − u2 )2 du

expanding and integrating relative to the new variable;


Z Z Z Z
= (1 − 2u + u ) du = 1 du − 2 u du + u4 du
2 4 2
May 31, 2018 10:33 ws-book9x6 Integration for Calculus, Analysis, and Differential Equations ws-book9x6 page 73

Trigonometric Integrals 73

u3 u5
=u−2 + +C substituting back ;
3 5
2 1
= sin x − sin3 x + sin5 x + C.
3 5

Remark 5.2. Although the answers afforded by the two different ap-
proaches look very different (cf. Examples 5.2), they actually differ by
a constant only.

• Case 3: both exponents m and n are even nonnegative integers, i.e.,


m = 2k and n = 2l with k, l = 0, 1, . . . . In this case, we use the
following integration strategy:
Step 1: use the power reduction identities
1 + cos 2x 1 − cos 2x
cos2 x = , sin2 x =
2 2
to transform the integrand into a polynomial in cos 2x;
Step 2: substitute u = 2x and apply the appropriate reduction
formula to the powers of cos u greater than 1 or use the preceding
strategies, i.e., repeatedly apply the power reduction identity to the
even powers of cos 2x and substitution to the odd ones.
Example 5.6 (Via Power Reduction and Reduction Formula).
Z
sin4 x cos2 x dx (m = 4, n = 2)

by the power reduction identities:


1 + cos 2x 1 − cos 2x
cos2 x = , sin2 x = ;
2 2
Z  2
1 − cos 2x 1 + cos 2x
Z
1
= dx = (1 − cos 2x)2 (1 + cos 2x) dx
2 2 8
multiplying and simplifying;
Z
1
= (1 − 2 cos 2x + cos2 2x)(1 + cos 2x) dx
8
Z
1
= (1 − cos 2x − cos2 2x + cos3 2x) dx
8
by the integration rules;
Z Z Z Z 
1
= 1 dx − cos 2x dx − cos2 2x dx + cos3 2x dx
8
May 17, 2018 12:7 ws-book9x6 Integration for Calculus, Analysis, and Differential Equations ws-book9x6 page 74

74 Integration for Calculus, Analysis, and Differential Equations

Z Z
1 1 1 1
= x− sin 2x − cos2 2x dx + cos3 2x dx
8 16 8 8
1
substituting: u = 2x, du = 2dx, dx = du ;
2
Z Z
1 1 1 1
= x− sin 2x − cos2 u du + cos3 u du
8 16 16 16
applying the appropriate reduction formula to cos2 u and cos3 u;
 Z 
1 1 1 cos u sin u 1
= x− sin 2x − + 1 du
8 16 16 2 2
 2 Z 
1 cos u sin u 2
+ + cos u du
16 3 3
1 1 1 1 1 1
= x− sin 2x− cos u sin u− u+ cos2 u sin u+ sin u+C
8 16 32 32 48 24
substituting back ;
1 1 1 1 1
= x− sin 2x − cos 2x sin 2x − x + cos2 2x sin 2x
8 16 32 16 48
1
+ sin 2x + C simplifying;
24
1 1 1 1
= x− sin 2x − cos 2x sin 2x + cos2 2x sin 2x + C.
16 48 32 48

Example 5.7 (Via Power Reduction and Substitution).


Z
sin4 x cos2 x dx by the power reduction identities:

1 + cos 2x 1 − cos 2x
cos2 x = , sin2 x = ;
2 2
Z  2
1 − cos 2x 1 + cos 2x
Z
1
= dx = (1 − cos 2x)2 (1 + cos 2x) dx
2 2 8
multiplying and simplifying;
Z
1
= (1 − 2 cos 2x + cos2 2x)(1 + cos 2x) dx
8
Z
1
= (1 − cos 2x − cos2 2x + cos3 2x) dx
8
by the integration rules;
Z Z Z Z 
1
= 1 dx − cos 2x dx − cos2 2x dx + cos3 2x dx
8
May 31, 2018 10:33 ws-book9x6 Integration for Calculus, Analysis, and Differential Equations ws-book9x6 page 75

Trigonometric Integrals 75

Z Z
1 1 1 1
= x− sin 2x − cos2 2x dx + cos3 2x dx
8 16 8 8
applying the power reduction identity
and splitting cos 2x off cos3 2x;
Z Z
1 1 1 1 + cos 4x 1
= x− sin 2x − dx + cos2 2x cos 2x dx
8 16 8 2 8
expressing cos2 2x in terms of sin 2x;
Z Z
1 1 1 1 + cos 4x 1
= x− sin 2x − dx + [1 − sin2 2x] cos 2x dx
8 16 8 2 8
by the integration rules and introducing the missing constant 2;
Z Z 
1 1 1
= x− sin 2x − 1 dx + cos 4x dx
8 16 16
Z
1 1
+ · [1 − sin2 2x](2 cos 2x) dx
8 2
substituting:
u = sin 2x, du = 2 cos 2x dx ;
Z
1 1 1 1 1
= x− sin 2x − x − sin 4x + [1 − u2 ] du
8 16 16 64 16
integrating relative to the new variable;
u3
 
1 1 1 1
= x− sin 2x − sin 4x + u− +C
16 16 64 16 3
substituting back ;
 
1 1 1 1 1
= x− sin 2x − sin 4x + sin 2x − sin3 2x + C
16 16 64 16 3
simplifying;
1 1 1
= x− sin 4x − sin3 2x + C.
16 64 48

Remark 5.3. There may also exist an approach based exclusively on


trigonometric identities.

Example 5.8 (Via Trigonometric Identities Only).


Z
sin4 x cos2 x dx

splitting sin2 x off sin4 x and combining with cos2 x;


May 31, 2018 10:33 ws-book9x6 Integration for Calculus, Analysis, and Differential Equations ws-book9x6 page 76

76 Integration for Calculus, Analysis, and Differential Equations

Z
= sin2 x[sin x cos x]2 dx

1
by the double-angle identity: sin x cos x = sin 2x ;
2
Z
1
= sin2 x sin2 2x dx
4
1 − cos 2θ
by the power reduction identity: sin2 θ = ;
2
Z
1
= (1 − cos 2x)(1 − cos 4x) dx multiplying;
16
Z
1
= (1−cos 2x−cos 4x+cos 2x cos 4x) dxby the integration rules;
16
Z Z Z Z
1 1 1 1
= 1 dx− cos 2x dx− cos 4x dx+ cos 4x cos 2x dx
16 16 16 16
by the product-to-sum identity:
1
cos α cos β = [cos(α − β) + cos(α + β)] ;
2
Z
1 1 1 1
= x− sin 2x − sin 4x + (cos 2x + cos 6x) dx
16 32 64 32
by the integration rules;
Z Z 
1 1 1 1
= x− sin 2x − sin 4x + cos 2x dx + cos 6x dx
16 32 64 32
1 1 1 1 1
= x− sin 2x − sin 4x + sin 2x + sin 6x + C
16 32 64 64 192
1 1 1 1
= x− sin 2x − sin 4x + sin 6x + C.
16 64 64 192

Remark 5.4. Although the answers afforded by the three different


approaches look very different, they essentially differ from each other
by a constant only.
Z
5.2.3. Integrals of the Form tanm x secn x dx

When evaluating a trigonometric integral of the form


Z
tanm x secn x dx,
May 17, 2018 12:7 ws-book9x6 Integration for Calculus, Analysis, and Differential Equations ws-book9x6 page 77

Trigonometric Integrals 77

where m and n are real exponents, similarly to


Z
sinm x cosn x dx,
we may encounter three special cases allowing clear-cut integration strate-
gies. They do not encompass all the possibilities and do not apply to,
say,

Z
tanπ x sec 2 x dx.
The three special cases are as follows.
• Case 1: n is an even positive integer, i.e., n = 2k + 2 with k = 0, 1, . . .
and m is an arbitrary real number. In this case, we use the following
integration strategy:
Step 1: split sec2 x off sec2k+2 x;
Step 2: express the remaining even power sec2k x in terms of
tan x via the identity sec2 x = tan2 x + 1 :
sec2k x = [sec2 x]k = [tan2 x + 1]k ;
Step 3: integrate via the substitution u = tan x.
Example 5.9.
Z
tan3 x sec4 x dx (m = 3, n = 4) splitting sec2 x off sec4 x;
Z
= tan3 x sec2 x sec2 x dx

expressing sec2 x in terms of tan x via sec2 x = tan2 x + 1 ;


Z
= tan3 x(tan2 x + 1) sec2 x dx

substituting: u = tan x, du = sec2 x dx ;


Z
= u3 (u2 + 1) du

multiplying and integrating relative to the new variable;


u6 u4
Z Z Z
= (u + u ) du = u du + u3 du =
5 3 5
+ +C
6 4
substituting back ;
tan6 x tan4 x
= + + C.
6 4
May 17, 2018 12:7 ws-book9x6 Integration for Calculus, Analysis, and Differential Equations ws-book9x6 page 78

78 Integration for Calculus, Analysis, and Differential Equations

• Case 2: m is an odd positive integer, i.e., m = 2k + 1 with k = 0, 1, . . .


and n is an arbitrary real number. In this case, we use the following
integration strategy:
Step 1: split sec x tan x off tan2k+1 x secn x;
Step 2: express the remaining even power tan2k x in terms of
sec x via the identity tan2 x = sec2 x − 1 :

tan2k x = [tan2 x]k = [sec2 x − 1]k ;

Step 3: integrate via the substitution u = sec x.

Example 5.10.
Z
tan3 x sec5 x dx(m = 3, n = 5) splitting sec x tan x off tan3 x sec5 x;
Z
= tan2 x sec4 (sec x tan x) dx

expressing tan2 x in terms of sec x via tan2 x = sec2 x − 1 ;


Z
= (sec2 x − 1) sec4 x(sec x tan x) dx

substituting: u = sec x, du = sec x tan x dx ;


Z
= (u2 − 1)u4 du

multiplying and integrating relative to the new variable;


u7 u5
Z Z
= u6 du − u4 du = − +C substituting back ;
7 5
sec7 x sec5 x
= − + C.
7 5

• Case 3: m is an even nonnegative integer, i.e., m = 2k with k =


0, 1, . . . , and n is an odd positive integer, i.e., n = 2l +1 with l = 0, 1, . . . .
In this case, we use the following integration strategy:
Step 1: express the even power tan2k x in terms of sec x via
the identity tan2 x = sec2 x − 1 :

tan2k x = [tan2 x]k = [sec2 x − 1]k ,

to transform the integrand into a polynomial in sec x;


May 31, 2018 10:33 ws-book9x6 Integration for Calculus, Analysis, and Differential Equations ws-book9x6 page 79

Trigonometric Integrals 79

Step 2: apply the appropriate reduction formula to the powers of


sec x greater than 1.

Example 5.11.
Z
tan2 x sec x dx (m = 2, n = 1)

expressing tan2 x in terms of sec x via tan2 x = sec2 x − 1 ;


Z
= (sec2 x − 1) sec x dx multiplying;
Z Z Z
= (sec3 x − sec x) dx = sec3 x dx − sec x dx

applying the appropriate reduction formula to the first integral;


Z Z
sec x tan x 1
= + sec x dx − sec x dx
2 2
sec x tan x 1
= − ln | sec x + tan x| + C.
2 2

Remark 5.5. Similar strategies, in the corresponding cases, apply to inte-


grals of the form
Z
cotm x cscn x dx.

5.3. Applications

Examples 5.12 (Applications).


1. Find the volume of the solid obtained by rotating the region bounded
by y = sin2 x and the x-axis on the interval [0, π/2] about the x-axis.
Solution: Applying the disk method (see, e.g., [1, 6]), we have:
Z π/2
2
π sin2 x dx

V =
0
1 − cos 2x
by the power-reduction identity: sin2 x = ;
2
π/2  2
1 − cos 2x
Z
= π dx squaring;
0 2
Z π/2
π
1 − 2 cos 2x + cos2 2x dx

=
0 4
May 17, 2018 12:7 ws-book9x6 Integration for Calculus, Analysis, and Differential Equations ws-book9x6 page 80

80 Integration for Calculus, Analysis, and Differential Equations

1 + cos 4x
by the power-reduction identity: cos2 2x = ;
2

Z π/2 
π 1 + cos 4x
= 1 − 2 cos 2x + dx by the integration rules;
4
0 2
" Z #
π 3 π/2
Z π/2
1 π/2
Z
= 1 dx − 2 cos 2x + cos 4x dx
4 2 0 0 2 0

by the Newton-Leibniz Formula;


" π/2 π/2 π/2 #
π 3 1 1
= x − sin 2x
+ · sin 4x
4 2 0 0 2 4 0

3π 2
  
π 3 π  1
= − 0 − (sin π − sin 0) + (sin 2π − sin 0) = un.3 .
4 2 2 8 16

2. Find the length of the curve y = ln(sin x), π/4 ≤ x ≤ π/2.


Solution: By the arc length formula,
π/2
sin0 x
Z
cos x
q
2
L= 1 + [y 0 (x)] dx since y0 = = = cot x;
π/4 sin x sin x
Z π/2 p
= 1 + cot2 x dx
π/4

by the trigonometric identity: 1 + cot2 x = csc2 x ;


Z π/2 √
= csc2 x dx considering that
π/4

csc2 x = | csc x| = csc x since csc x > 0 on [π/4, π/2];
Z π/2
= csc x dx by the Newton-Leibniz Formula;
π/4
π/2

= ln | csc x−cot x| = ln | csc(π/2)−cot(π/2)|−ln | csc(π/4)−cot(π/4)|
π/4
√ √ 1 √
= ln |1−0|−ln | 2−1| = − ln( 2−1) = − ln √ = ln( 2+1) un.
2+1
May 17, 2018 12:7 ws-book9x6 Integration for Calculus, Analysis, and Differential Equations ws-book9x6 page 81

Trigonometric Integrals 81

5.4. Practice Problems

Evaluate the integrals.


Z
sin3 x
Z
1. sin 5x cos 2x dx 11. dx
Z cos2 x
cos3 x
Z
2. cos2 3x dx 12. √ dx
3 2
Z sin x
(1 + 2 sin x)2 dx sec4 x
Z
3.
13. dx
Z tan3 x
(1 − cos 2x)2 dx
Z
4.
14. cot3 x csc5 x dx
Z
sin2 x cos2 x dx
Z
5.
15. tan5 x dx
Z
sin2 x cos4 x dx sec3 (ln x)
Z
6.
16. dx
Z x
7. sin3 x cos2 x dx Z π/3
Z 17. cot4 x dx
π/6
8. sin2 x cos3 x dx Z √π/2
Z 18. x cos3 (x2 ) dx
9. sin7 x dx 0
Z
Z π/2 √
4 19. 1 + cos 2x dx
10. cos x dx
Z0 π
20. sin2n x dx for n = 1, 2, . . .
0
b2530   International Strategic Relations and China’s National Security: World at the Crossroads

This page intentionally left blank

b2530_FM.indd 6 01-Sep-16 11:03:06 AM


May 17, 2018 12:7 ws-book9x6 Integration for Calculus, Analysis, and Differential Equations ws-book9x6 page 83

Chapter 6

Trigonometric Substitutions

6.1. Reverse Substitutions

Frequently, an integral
Z Z b
f (x) dx or f (x) dx
a
can be simplified if we substitute a suitably chosen continuously differen-
tiable function g(t) for x:
Z
f (x) dx with x = g(t), dx = g 0 (t) dt ;
Z
= f (g(t))g 0 (t) dt,

in which case, we use the Substitution Rule for Indefinite/Definite Integral


(Theorem 3.1/Theorem 3.2) backwards. Thus, we call such a substitution
reverse.
For the definite integral
Z b
f (x) dx,
a
we also change the integration limits:

Z b x t
f (x) dx with x = g(t), dx = g 0 (t) dt a c ;
a
b d
Z d
= f (g(t))g 0 (t) dt,
c

83
May 17, 2018 12:7 ws-book9x6 Integration for Calculus, Analysis, and Differential Equations ws-book9x6 page 84

84 Integration for Calculus, Analysis, and Differential Equations

Requirements to Reverse Substitution


A suitable choice of a reverse substitution x = g(t) is based on the following
natural criteria:
1. The substitution should simplify the initial integral, i.e., transform it
into an integral, which can be evaluated via known integration strate-
gies.
2. The function g(t) must be continuously differentiable on some interval.
3. The function g(t) must also be one-to-one on its interval of definition
so that there exist an inverse g −1 (x) for us to be able to substitute
back :
t = g −1 (x)
for indefinite integral or have a unique choice for the new integration
limits:
x t
a c = g −1 (a)
b d = g −1 (b)

Trigonometric Substitutions
Trigonometric substitutions, we are about to consider, are reverse by nature
and used to transform integrals containing the terms
a2 − x2 , x2 + a2 , or x2 − a2 ,
where a > 0, into trigonometric integrals.

6.2. Integrals Containing a2 − x2


2 2
√ containing the term a − x , where a > 0, especially in the
For an integral
2 2
radical form a − x , we use the sine substitution:
x = a sin θ, −π/2 ≤ θ ≤ π/2.
Remark 6.1. The range for the new variable θ is purposely chosen to be
[−π/2, π/2], since
• on [−π/2, π/2], the function a sin θ is one-to-one, and thus, we can
substitute back:
x
θ = arcsin ,
a
and has the largest range of values: the entire interval [−a, a];
May 17, 2018 12:7 ws-book9x6 Integration for Calculus, Analysis, and Differential Equations ws-book9x6 page 85

Trigonometric Substitutions 85

• also, on [−π/2,√π/2], cos θ ≥ 0, which is instrumental for the elimination


of the radical a2 − x2 (see below).

By the Pythagorean trigonometric identity


1 − sin2 θ = cos2 θ
(see Appendix C), we have:
a2 − x2 = a2 − a2 sin2 θ = a2 (1 − sin2 θ) = a2 cos2 θ.

Hence, the substitution eliminates the radical a2 − x2 :
p √
a2 − x2 = a2 cos2 θ = |a|| cos θ|
considering that a > 0 and cos θ ≥ 0 for −π/2 ≤ θ ≤ π/2;
= a cos θ.
The substitution transforms the initial integral into a trigonometric integral
relative to the new variable θ.
Example 6.1 (Integrals Containing a2 − x2 ).

Z √2 √ Z √2 √ 2
q
2 − x2 2 − x2
dx = dx
1 x2 1 x2
substituting and changing the integration limits:

x= √ 2 sin θ
dx = 2 cos√θ dθ x θ

2
2 − x = 2 cos θ 1 π/4 ;

x 2 π/2
θ = arcsin √
2
√ 2
π/2
2 cos θ √
Z π/2 Z π/2 
cos2 θ
Z
cos θ
= 2 cos θ dθ = dθ = dθ
π/4 2 sin2 θ 2
π/4 sin θ π/4 sin θ
cos θ
by the trigonometric identity: = cot θ ;
sin θ
Z
= cot2 θ dθ by the trigonometric identity: cot2 θ = csc2 θ − 1 ;
Z π/2
= (csc2 θ − 1) dθ by the integration rules;
π/4
Z π/2 Z π/2
= csc2 θ dθ − 1 dθ by the Newton-Leibniz Formula;
π/4 π/4
May 17, 2018 12:7 ws-book9x6 Integration for Calculus, Analysis, and Differential Equations ws-book9x6 page 86

86 Integration for Calculus, Analysis, and Differential Equations

π/2 π/2

= − cot θ
− θ = − [cot(π/2) − cot(π/4)] − [π/2 − π/4]
π/4 π/4

= −[0 − 1] − π/4 = 1 − π/4.

What If the Integral Is Indefinite?


If the initial integral is indefinite, after having substituted and evaluated
the integral relative to the new variable θ, we obtain an answer in terms of
θ and/or some trigonometric functions of θ. Thus, when substituting back :
x
θ = arcsin ,
a
x
we are to encounter expressions containing arcsin inside trigonometric
a
functions, which should be equivalently transformed into algebraic expres-
x
sions in x not containing the trigonometric functions of arcsin .
a
This can be done based on cancellation and trigonometric identities (taking
π x π
into account that − ≤ arcsin ≤ ):
2 a 2
 x x
1. sin arcsin = ,
a a r
 x  p h x i2
2. cos arcsin = 1 − [sin(arcsin(x/a))] = 1 −2
a a
r √
2
a −x 2 2
a −x 2
= = ,
a2 a
x x
 x  sin arcsin x
3. tan arcsin = a = √ a =√ ,
a
 x 2
a −x 2 a − x2
2
cos arcsin
a a √
 x  1 1 a2 − x2
4. cot arcsin =  x = x = ,
a tan arcsin √ x
a a2 − x2
 x 1 1 a
5. csc arcsin =  x = x = ,
a sin arcsin x
a a
 x 1 1 a
6. sec arcsin
a
=  x  = √a2 − x2 = √a2 − x2 .
cos arcsin
a a

In respect that
x
sin θ = ,
a
May 17, 2018 12:7 ws-book9x6 Integration for Calculus, Analysis, and Differential Equations ws-book9x6 page 87

Trigonometric Substitutions 87

it may be easier, however, to use the reference triangle approach for the
same purposes, i.e., consider θ to be an acute angle in a right triangle with
p
hypotenuse = a, opposite leg = x, and adjacent leg = a2 − x2 .

Example 6.2 (Integrals Containing a2 − x2 ).


For a > 0,

Z p x = a sin θ
a2 − x2 dx substituting: dx
√ = a cos θ dθ ;
a2 − x2 = a cos θ
Z Z
= a cos θa cos θ dθ = a2 cos2 θ dθ

1 + cos 2θ
by the power-reduction identity: cos2 θ = ;
2
a2
Z
= [1 + cos 2θ] dθ by the integration rules;
2
a2 a2
Z Z   
1
= 1 dθ + cos 2θ dθ = θ + sin 2θ + C
2 2 2
1
by the double-angle identity: sin 2θ = sin θ cos θ ;
2
a2
= [θ + sin θ cos θ] + C
2
substituting back and using the reference triangle:
May 17, 2018 12:7 ws-book9x6 Integration for Calculus, Analysis, and Differential Equations ws-book9x6 page 88

88 Integration for Calculus, Analysis, and Differential Equations


x x a2 − x2
θ = arcsin , sin θ = , cos θ = ;
a a a
" √ #
a2 x x a2 − x2 xp 2 a2 x
= arcsin + +C = a − x2 + arcsin + C.
2 a a a 2 2 a

6.3. Integrals Containing x2 + a2


2 2
For an integral
√ containing the term x + a , where a > 0, especially in the
2 2
radical form x + a , we use the tangent substitution:
x = a tan θ, −π/2 < θ < π/2.

Remark 6.2. The range for the new variable θ is purposely chosen to be
(−π/2, π/2), since

• on (−π/2, π/2), the function a tan θ is one-to-one, and thus, we can


substitute back:
x
θ = arctan ,
a
and has the largest range of values: the entire real axis (−∞, ∞);
• also, on (−π/2, π/2),
√ sec θ ≥ 1, which is instrumental for the elimina-
tion of the radical x2 + a2 (see below).

By the Pythagorean trigonometric identity


tan2 θ + 1 = sec2 θ
(see Appendix C), we have:
x2 + a2 = a2 tan2 θ + a2 = a2 (tan2 θ + 1) = a2 sec2 θ.

Hence, the substitution eliminates the radical x2 + a2 :
p √
x2 + a2 = a2 sec2 θ = |a|| sec θ|
considering that a > 0 and sec θ ≥ 1 for −π/2 < θ < π/2
= a sec θ.

The substitution transforms the initial integral into a trigonometric integral


relative to the new variable θ.
May 17, 2018 12:7 ws-book9x6 Integration for Calculus, Analysis, and Differential Equations ws-book9x6 page 89

Trigonometric Substitutions 89

As the following example shows, completing the square may be a necessary


preliminary step before making the required substitution.
Example 6.3 (Integrals Containing x2 + a2 ).
Z 2√3−1 √ 2
x + 2x + 5
dx completing the square:
1 (x + 1)4
x2 + 2x + 5 = x2 + 2x + 1 + 4 = (x + 1)2 + 22 ;

2 3−1
p
(x + 1)2 + 22
Z
= dx
1 (x + 1)4
substituting and changing the integration limits:

x + 1 = 2 tan θ
dx 2 x θ
p = 2 sec θ dθ
(x + 1)2 + 22 = 2 sec θ √1 π/4 ;
x+1 2 3−1 π/3
θ = arctan
2
π/3 Z π/3
1 sec3 θ
Z
2 sec θ 2
= 4 2 sec θ dθ = 4 dθ
π/4 16 tan θ π/4 4 tan θ

1 sin θ
by the trigonometric identities: sec θ = , tan θ = ;
cos θ cos θ
1
Z π/3
1 cos3 θ
= dθ simplifying;
π/4 4 sin4 θ
cos4 θ
Z π/3
1 cos θ
= dθ by the integration rules;
π/4 4 sin4 θ
Z π/3
1 cos θ
= dθ
4 π/4 sin4 θ
θ u

substituting again: u = sin θ, du = cos θ dθ π/4 2/2 ;

π/3 3/2

Z 3/2
1
= √ u−4 du by the Newton-Leibniz Formula;
4 2/2
√3/2 " √ #

 
1 1 −3 1 8 8 1 8 3
= · u √ =− √ − √ =− −2 2
4 (−3) 2/2 12 3 3 2 2 12 9
May 17, 2018 12:7 ws-book9x6 Integration for Calculus, Analysis, and Differential Equations ws-book9x6 page 90

90 Integration for Calculus, Analysis, and Differential Equations

√ √
2 2 3
= − .
6 27
What If the Integral Is Indefinite?
If the initial integral is indefinite, after having substituted and evaluated
the integral relative to the new variable θ, we obtain an answer in terms of
θ and/or some trigonometric functions of θ. Thus, when substituting back :
x
θ = arctan ,
a
x
we are to encounter expressions containing arctan inside trigonometric
a
functions, which should be equivalently transformed into algebraic expres-
x
sions in x not containing the trigonometric functions of arctan .
a
This can be done based on cancellation and trigonometric identities (taking
π x π
into account that − < arctan < ):
2 a 2
 x  x
1. tan arctan = ,
a a
 x 1 1 a
2. cot arctan =  x = x = ,
a tan arctan x
a a
rh i
 x p x 2
3. sec arctan = [tan(arctan(x/a))]2 + 1 = +1
a a
r √
x2 + a2 x2 + a2
= 2
= ,
a a
 x 1 1 a
4. cos arctan
a
=  x  = √x2 + a2 = √x2 + a2 ,
sec arctan
a
 x  x  a x x a
5. sin arctan = tan arctan cos arctan = ·√
a a a a x + a2
2
x
=√ ,
x2 + a2

 x 1 1 x2 + a2
6. csc arctan =  x = x = .
a sin arctan √ x
a x2 + a2

In respect that
x
tan θ = ,
a
May 17, 2018 12:7 ws-book9x6 Integration for Calculus, Analysis, and Differential Equations ws-book9x6 page 91

Trigonometric Substitutions 91

it may be easier, however, to use the reference triangle approach for the
same purposes, i.e., consider θ to be an acute angle in a right triangle with
p
opposite leg = x, adjacent leg = a, and hypotenuse = x2 + a2 .

Example 6.4 (Integrals Containing x2 + a2 ).

x = tan θ
x3 x3
Z Z
√ √ 2
dx = dx substituting: √ = sec θ dθ ;
dx
x2 + 1 x2 + 1 2
x2 + 1 = sec θ
tan3 θ
Z Z
= sec2 θ dθ = tan3 θ sec θ dθ
sec θ
applying the appropriate integration strategy,
we split sec x tan x off tan3 θ sec θ;
Z
= tan2 θ(sec θ tan θ) dθ

expressing tan2 x in terms of sec x via tan2 θ = sec2 θ − 1 ;


Z
= (sec2 θ − 1)(sec θ tan θ) dθ

substituting again: u = sec θ, du = sec θ tan θ dθ ;


Z
= (u2 − 1) du by the integration rules;
May 17, 2018 12:7 ws-book9x6 Integration for Calculus, Analysis, and Differential Equations ws-book9x6 page 92

92 Integration for Calculus, Analysis, and Differential Equations

u3
Z Z
= u2 du − 1 du = −u+C substituting back ;
3
sec3 θ
= − sec θ + C
3
substituting back again and using the reference triangle:
p
θ = arctan x, sec θ = x2 + 1 ;

(x2 + 1)3/2 p 2 x2 + 1 2
= − x +1+C = (x − 2) + C.
3 3

6.4. Integrals Containing x2 − a2


2 2
√ containing the term x − a , where a > 0, especially in the
For an integral
2 2
radical form x − a , we use the secant substitution:
x = a sec θ, 0 ≤ θ < π/2 or π/2 < θ ≤ π.

Remark 6.3. The range for the new variable θ is purposely chosen to be
[0, π/2) ∪ (π/2, π], since on [0, π/2) ∪ (π/2, π], the function a sec θ is one-to-
one and thus, we can substitute back:
x
θ = arcsec ,
a
and has the largest range of values (−∞, a] ∪ [a, ∞).

By the Pythagorean trigonometric identity


sec2 θ − 1 = tan2 θ
(see Appendix C), we have:
x2 − a2 = a2 sec2 θ − a2 = a2 (sec2 θ − 1) = a2 tan2 θ.

Hence, the substitution eliminates the radical x2 − a2 :
p p
x2 − a2 = a2 tan2 θ = |a|| tan θ| considering that a > 0;
(
a tan θ for 0 ≤ θ < π/2 (x ≥ a)
= = sgn x · a tan θ,
−a tan θ for π/2 < θ ≤ π (x ≤ −a)
(
1 for x ≥ a
where sgn x = .
−1 for x ≤ −a
May 17, 2018 12:7 ws-book9x6 Integration for Calculus, Analysis, and Differential Equations ws-book9x6 page 93

Trigonometric Substitutions 93

The substitution transforms the initial integral into a trigonometric integral


relative to the new variable θ.

Example 6.5 (Integrals Containing x2 − a2 ).


Z 2 Z 2
1 1
√ dx = √ √ 3 dx
2 (x − 1)3/2
2
2 ( x2 − 1)
x = sec θ
x θ
dx = sec θ tan θ dθ √
substituting: √ 2 π/4 ;
x2 − 1 = tan θ
2 π/3
θ = arcsec x
Z π/3 Z π/3
1 sec x
= sec x tan x dθ = dθ
π/4 tan3 x π/4 tan2 x
1 sin θ
by the trigonometric identities: sec θ = , tan θ = ;
cos θ cos θ
1
Z π/3
= cos θ dθ simplifying and regrouping;
π/4 sin2 θ
cos2 θ
Z π/3 Z π/3
cos θ 1 cos θ
= dθ = dθ
π/4 sin2 θ π/4 sin θ sin θ
1 cos θ
by the trigonometric identities: = csc θ, = cot θ ;
sin θ sin θ
Z π/3
= csc θ cot θ dθ by the Newton-Leibniz Formula;
π/4
π/3 √
√ √
 
2 2 3
= − csc θ = −[csc(π/3) − csc(π/4)] = − √ − 2 = 2 − .
π/4 3 3

What If the Integral Is Indefinite?

If the initial integral is indefinite, after having substituted and evaluated


the integral relative to the new variable θ, we obtain an answer in terms of
θ and/or some trigonometric functions of θ. Thus, when substituting back :
x
θ = arcsec ,
a
May 17, 2018 12:7 ws-book9x6 Integration for Calculus, Analysis, and Differential Equations ws-book9x6 page 94

94 Integration for Calculus, Analysis, and Differential Equations

x
we are to encounter expressions containing arcsec inside trigonometric
a
functions, which should be equivalently transformed into algebraic expres-
x
sions in x not containing the trigonometric functions of arcsec .
a
This can be done based on cancellation and trigonometric identities (taking
x π π x
into account that 0 ≤ arcsec < or < arcsec ≤ π):
a 2 2 a
 x x
1. sec arcsec = ,
a a
 x  1 1 a
2. cos arcsec =  x = x = ,
a sec arcsec x
a a
r r
 x h  x i2 h a i2
3. sin arcsec = 1 − cos arcsec = 1−
a a x
r √
2
x −a 2 2
x −a 2
= = ,
x2 |x|
 x 1 1 |x|
4. csc arcsec
a
=  x  = √x2 − a2 = √x2 − a2 ,
sin arcsec
a |x|

 x  x2 − a2 √
 x  sin arcsec |x| x x2 − a2
5. tan arcsec = a
a
 x = a =
|x| a
cos arcsec
a x

x2 − a2
= sgn x ,
a
 x  1 1 a
6. cot arcsec =  x = √ = sgn x √ .
a tan arcsec
2
x −a 2 x − a2
2
a sgn x
a

In respect that

x
sec θ = ,
a

it may be easier, however, to use the reference triangle approach for the
same purposes, i.e., consider θ to be an acute angle in a right triangle with
p
hypotenuse = x, adjacent leg = a, and opposite leg = x2 − a 2 .
May 17, 2018 12:7 ws-book9x6 Integration for Calculus, Analysis, and Differential Equations ws-book9x6 page 95

Trigonometric Substitutions 95

π x
Remark 6.4. To include the case of < arcsec ≤ π, when the
2 a
hypotenuse x = a sec θ of the reference triangle is negative, we need
x
to multiply all trigonometric functions of arcsec obtained from it, ex-
a
 x   x 
cept sec arcsec and cos arcsec , by sgn x taking into account that
a a
|x| = sgn x · x, x = sgn x|x|, and sgn2 x = 1. For instance,
√ √
 x x2 − a2 x2 − a2
sin arcsec = sgn x = .
a x |x|
Example 6.6 (Integrals Containing x2 − a2 ).
For x < −1/2 or x > 1/2,
Z Z Z
1 1 1
√ dx = p dx = p dx
2 2
x 4x − 1 2 2
2x x − 1/4 2x x − (1/2)2
2 2

1
x= sec θ
2
1
substituting: dx = sec θ tan θ dθ ;
2
p 1
x2 − 1/4 = sgn x tan θ
2
Z
1 1
= sec θ tan θ dθ simplifying;
1 1 2
2 sec2 θ sgn x tan θ
Z 4 2
2 sgn x
= dθ
sec θ
1
by the trigonometric identity = cos θ and the integration rules;
sec θ
May 17, 2018 12:7 ws-book9x6 Integration for Calculus, Analysis, and Differential Equations ws-book9x6 page 96

96 Integration for Calculus, Analysis, and Differential Equations

Z Z
= 2 sgn x cos θ dθ = 2 sgn x cos θ dθ = 2 sgn x sin θ + C

substituting back and using the reference triangle:


p
x2 − 1/4
sin θ = sgn x ;
x
p
2 x2 − 1/4
= 2 sgn x +C considering that sgn2 x = 1;
√ x
4x2 − 1
= + C.
x

6.5. Applications

Examples 6.7 (Applications).


x2 y2
1. Find the area of the ellipse 2
+ 2 = 1 (a, b > 0).
a b
Solution: The quarter of the ellipse in the first quadrant is explicitly
defined as follows:
bp 2
y= a − x2 , 0 ≤ x ≤ a.
a
Hence, by the symmetry,
Z a p
b
A=4 a2 − x2 dx
0 a
substituting and changing the integration limits:

x = a sin θ
dx x θ
√ = a cos θ dθ 0 0 ;
a2 − x2 = a cos θ
x a π/2
θ = arcsin
a
Z π/2
b 2
=4 a cos2 θ dθ
0 a
1 + cos 2θ
by the power-reduction identity: cos2 θ = ;
2
Z π/2
ab
=4 [1 + cos 2θ] dθ by the integration rules;
0 2
May 17, 2018 12:7 ws-book9x6 Integration for Calculus, Analysis, and Differential Equations ws-book9x6 page 97

Trigonometric Substitutions 97

"Z #
π/2 Z π/2
= 2ab 1 dθ + cos 2θ dθ
0 0

by the Newton-Leibniz Formula;


" #
π/2 1 π/2
 
π 1
= 2ab θ
+ sin 2θ
= 2ab + (sin π − sin 0) = πab un.2
0 2 0 2 2
(cf. Examples 6.2).
Remark 6.5. When a = b = r, we obtain the well-known circle (of
radius r) area formula A = πr2 .
1
2. Find the area of the region bounded by the graph of y = and
(x2 + 4)2
the x-axis on the interval [−2, 2].
Solution:
Z 2
1
A= dx by the evenness of the integrand ;
−2 (x2 + 4)2
Z 2 Z 2
1 1
=2 dx = 2 dx
0 (x2 + 4)2 0 (x2 + 22 )2
substituting and changing the integration limits:

x = 2 tan θ
dx = 2 sec2 θ dθ x θ
x2 + 22 = 4 sec2 θ 0 0 ;
x 2 π/4
θ = arctan
2
Z π/4
1
=2 2 sec2 θ dθ simplifying;
0 42 sec4 θ
Z π/4
1
=2 dθ
0 8 sec2 θ
1
by the trigonometric identity: = cos θ ;
sec θ
Z π/4
1
=2 cos2 θ dθ
0 8
1 + cos 2θ
by the trigonometric identity: cos2 θ = ;
2
Z π/4 Z π/4
1 1
=2 cos2 θ dθ = 2 [1 + cos 2θ] dθ
0 8 0 16
May 17, 2018 12:7 ws-book9x6 Integration for Calculus, Analysis, and Differential Equations ws-book9x6 page 98

98 Integration for Calculus, Analysis, and Differential Equations

by the integration rules and the Newton-Leibniz Formula;


"Z # " π/4 #
π/4 Z π/4 π/4
1 1 1
= 1 dθ + cos 2θ dθ = θ + sin 2θ
8 0 0 8 0 2 0
 
1 π 1 π 1
= + = + un.2 .
8 4 2 32 16

Exercise 6.1 (Alternate Approach).


Solve the last problem using reduction formula 11 of Appendix B.

6.6. Practice Problems

Evaluate the integrals.

1
x2
Z Z
1
1. dx 7. √ dx
0 (4 − x2 )3/2 x2 − 9
√ √
9 − 4x2
Z
3 3
x2
Z
2. dx 8. dx
(x2 + 9)2 x2
3 √
9x2 − 16
Z 2 Z
1 9. dx
3. √
√ dx x3
3 x2 − 1
2/ 3 x Z
Z 0p 1
10. √ dx
4. x2 + 2x + 2 dx (x − 1) x2 − 2
−1
x2 + 2x + 1
Z
x5 √
Z
11. dx
5. √ dx x2 + 2x + 10
1 − x2 Z
1
x2 12. √ dx
Z
6. √ dx (x + 1) x2 + 2x − 5
2
x2 + 2
x2 + 1
Z
13. √ dx
x x4 + 1
14. Prove that, for a > 0,
a2
Z p
xp 2 p
x2 ± a2 dx = x ± a2 ± ln |x+ x2 ± a2 |+C (cf. Examples 4.4).
2 2
Using the results of Examples 6.2 and the prior problem, evaluate the in-
tegrals.
Z p Z p
15. 2 + x − x2 dx 16. x x4 + 2x2 − 1 dx
May 17, 2018 12:7 ws-book9x6 Integration for Calculus, Analysis, and Differential Equations ws-book9x6 page 99

Chapter 7

Integration of Rational Functions

7.1. Rational Functions

Definition 7.1 (Rational Function (Fraction)).


A rational function (or fraction) is a ratio
P (x)
,
Q(x)
of two polynomials
P (x) = am xm + am−1 xm−1 + · · · + a1 x + a0
and
Q(x) = bn xn + bn−1 xn−1 + · · · + b1 x + b0
with degrees m, n = 0, 1, 2, . . . and real coefficients am , . . . , a0 , bn , . . . , b0 ,
am , bn 6= 0.
A fraction is said to be in reduced form if the numerator and denominator
have no common factors. By canceling all common factors, any fraction
can be equivalently written in reduced form.
A fraction is said to be proper if the degree of the numerator is less than
the degree of the denominator (m < n) and improper otherwise (m ≥ n).

Remark 7.1. Any polynomial P (x) is trivially a fraction with the denom-
inator Q(x) = 1.

Examples 7.1 (Rational Functions).


x2 − 1 (x − 1)(x + 1)
1. = is an improper fraction not in reduced form.
x+1 x+1
Canceling the common factor x + 1, we have:
(x − 1)(x + 1)
= x − 1, x 6= −1.
x+1

99
May 17, 2018 12:7 ws-book9x6 Integration for Calculus, Analysis, and Differential Equations ws-book9x6 page 100

100 Integration for Calculus, Analysis, and Differential Equations

x4 − 1 (x − 1)(x + 1)(x2 + 1)
2. 2
= is an improper fraction in reduced
x + 2x x(x + 2)
form (nothing to cancel).
x−2 x−2
3. 2 = is a proper fraction not in reduced form.
x − 3x + 2 (x − 1)(x − 2)
Canceling the common factor x − 2, we have:
x−2 1
= , x 6= 2.
(x − 1)(x − 2) x−1
x−1
4. 2 is a proper fraction in reduced form.
x +x+1
Observe that the quadratic polynomial x2 + x + 1 cannot be factored into
two linear polynomials unlike x2 − 3x + 2 = (x − 1)(x − 2).

The integration of rational functions is executed via the Partial Fraction


Method reducing such to the integration of so-called partial fractions of
rather simple structure.

7.2. Partial Fractions

Definition 7.2 (Partial Fractions).


A partial fraction is a proper fraction of one of the following four types:
A
Type 1: ,
ax + b
A
Type 2: , k = 2, 3, . . . ,
(ax + b)k
Ax + B
Type 3: ,
ax2 + bx + c
Ax + B
Type 4: , l = 2, 3, . . . ,
(ax + bx + c)l
2

where,
• a, b, and c are real coefficients with a 6= 0,
• A and B are real coefficients not simultaneously equal to 0 (i.e., A2 +
B 2 6= 0), and
• ax2 + bx + c is an irreducible quadratic polynomial, i.e., such that cannot
be further factored into two linear polynomials with real coefficients,
which is the case iff D = b2 − 4ac < 0.

Examples 7.2 (Partial Fractions).


2
1. is a type 1 partial fraction (A = 2, a = 3, and b = −1).
3x − 1
May 17, 2018 12:7 ws-book9x6 Integration for Calculus, Analysis, and Differential Equations ws-book9x6 page 101

Integration of Rational Functions 101

1
2. is a type 2 partial fraction (A = 1, a = 2, and b = 5).
(2x + 5)4
x−1
3. 2 is a type 3 partial fraction (A = 1, B = −1, a = b = c = 1).
x +x+1
x−1
4. is a type 4 partial fraction (A = 1, B = −1, a = b = c =
(x2 + x + 1)2
1).
2x + 1
5. 2 is a not a partial fraction, since x2 − 3x + 2 is not irre-
x − 3x + 2
ducible: x2 − 3x + 2 = (x − 1)(x − 2).

7.2.1. Integration of Type 1/Type 2 Partial Fractions

A type 1/type 2 partial fraction is integrated via the trivial substitution (see
Sec. 3.1.4) or, which is the same, the Useful Integration Formula (Theorem
2.1) as follows:
Z Z
A 1 A
dx = A dx = ln |ax + b| + C,
ax + b ax + b a
Z Z
A A 1
dx = A (ax + b)−k dx = · (ax + b)−k+1 + C,
(ax + b)k a −k + 1
k = 2, 3, . . . .

Examples 7.3 (Integration of Type 1/Type 2 Partial Fractions).


Z Z
2 1 2
1. dx = 2 dx = ln |3x − 1| + C.
3x − 1 3x − 1 3
Z Z
1 1 1
2. 4
dx = (2x + 5)−4 dx = · (2x + 5)−3 + C
(2x + 5) 2 −3
1 1
=− + C.
6 (2x + 5)3

7.2.2. Integration of Type 3 Partial Fractions

Generally, a type 3 partial fraction


Z
Ax + B
dx
ax2 + bx + c
is integrated via two procedures: substitution and completing the square
(see Sec. 2.2.7).
May 17, 2018 12:7 ws-book9x6 Integration for Calculus, Analysis, and Differential Equations ws-book9x6 page 102

102 Integration for Calculus, Analysis, and Differential Equations

Example 7.4 (Integration of Type 3 Partial Fractions).


x−1
Z
2
dx
x +x+1
since [x2 + x + 1]0 = 2x + 1, multiplying and dividing by 2;
2x − 2
Z
1
= dx adding and subtracting 1;
2 x2 + x + 1
2x + 1 − 3
Z
1
= dx by the integration rules;
2 x2 + x + 1
Z Z
1 2x + 1 3 1
= 2
dx − 2
dx
2 x +x+1 2 x +x+1
substituting for the first integral: u = x2 + x + 1, du = (2x + 1)dx
and completing the square for the second one:

x2 + x + 1 = x2 + 2(1/2)x + 1/4 − 1/4 + 1 = (x + 1/2)2 + ( 3/2)2 ;
Z Z
1 1 3 1
= du − √ dx
2 u 2 (x + 1/2) + ( 3/2)2
2

1 3 1 x + 1/2
= ln |u| − √ arctan √ +C substituting back ;
2 2 3/2 3/2
1 √ 2x + 1
= ln |x2 + x + 1| − 3 arctan √ +C since x2 + x + 1 > 0;
2 3
1 √ 2x + 1
= ln(x2 + x + 1) − 3 arctan √ + C.
2 3

Observations
When integrating a type 3 partial fraction,
• In certain cases, only one of the two procedures is involved: substitution,
asZis the case for Z
2x + 1 1
dx = du with u = x2 + x + 1, du = (2x + 1)dx ,
x2 + x + 1 u
or completing Z the square, as is the
Z case for
1 1
dx = √ dx.
x2 + x + 1 (x + 1/2)2 + ( 3/2)2
• If substitution is involved, it is always of the form
u = ax2 + bx + c, du = (2ax + b)dx
and leads to the integral of
Z the form
1
du = ln |u| + C.
u
May 17, 2018 12:7 ws-book9x6 Integration for Calculus, Analysis, and Differential Equations ws-book9x6 page 103

Integration of Rational Functions 103

• If completing the square is involved, due to the fact that the quadratic
polynomial ax2 + bx + c is irreducible, it always leads to the integral of
the form (up to a constant factor)
Z
1 1 x+h
dx = arctan + C.
(x + h)2 + d2 d d

7.2.3. Integration of Type 4 Partial Fractions


Just as in the case of a type 3 partial fraction, when integrating a type 4
partial fraction
Z
Ax + B
dx, l = 2, 3, . . . ,
(ax + bx + c)l
2

we apply substitution and completing the square. However, when the latter
is involved, we also use reduction formula 11 of Appendix B (see Examples
4.6), i.e., partial integration implicitly.
Example 7.5 (Integration of Type 4 Partial Fractions).
x−1
Z
dx in the same manner as in the prior example;
(x2 + x + 1)2
Z Z
1 2x + 1 3 1
= dx − dx
2 (x2 + x + 1)2 2 (x2 + x + 1)2
substituting for the first integral: u = x2 + x + 1, du = (2x + 1)dx
and completing the square for the second one (see the prior example);
Z Z
1 −2 3 1
= u du − √ 2 dx
2 2 
(x + 1/2) + ( 3/2)2
2

integrating and using reduction formula 11 of Appendix B



with x + 1/2 for x, a = 3/2, and n = 2 for the second integral;
  
1 1 3 1 x + 1/2
= − − √ √
2 u 2 2( 3/2)2 (x + 1/2)2 + ( 3/2)2
Z 
1
+ √ dx
(x + 1/2)2 + ( 3/2)2
substituting back, integrating, and simplifying;
1 1 x + 1/2 2 2x + 1
=− 2
− 2 − √ arctan √ +C
2x +x+1 x +x+1 3 3
x+1 2 2x + 1
=− 2 − √ arctan √ + C.
x +x+1 3 3
May 17, 2018 12:7 ws-book9x6 Integration for Calculus, Analysis, and Differential Equations ws-book9x6 page 104

104 Integration for Calculus, Analysis, and Differential Equations

Observations
When integrating a type 4 partial fraction,
• In certain cases, only one of the two procedures is involved: substitution,
as is the case for
Z Z
2x + 1
dx = u−2 du
(x2 + x + 1)2
with u = x2 + x + 1, du = (2x + 1)dx ,
or completing the square, as is the case for
Z Z
1 1
dx = √ 2 dx.
(x2 + x + 1)2 
(x + 1/2) + ( 3/2)2
2

• If substitution is involved, it is always of the form

u = ax2 + bx + c, du = (2ax + b)dx


and leads to the integral of the form
Z
1
u−l du = u−l+1 + C, l = 2, 3, . . . .
−l + 1
• If completing the square is involved, due to the fact that the quadratic
polynomial ax2 + bx + c is irreducible, it always leads to the integral of
the form (up to a constant factor)
Z
1
l
dx, l = 2, 3, . . . ,
[(x + h)2 + d2 ]
to which we apply reduction formula 11 of Appendix B l − 1 times.

7.3. Partial Fraction Decomposition

Observe that, by the Fundamental Theorem of Algebra (see, e.g., [1, 6]),
any polynomial Q(x) with real coefficient of degree n = 1, 2, . . . can be
factored into factors of the form
(ax + b)k or (ax2 + bx + c)l ,
where k and l are nonnegative integer exponents, a, b, and c are real coeffi-
cients with a 6= 0 and ax2 + bx + c is an irreducible quadratic polynomial.
Examples 7.6 (Factoring Polynomials).

1. 2x2 − x − 1 = (2x + 1)(x − 1).


May 17, 2018 12:7 ws-book9x6 Integration for Calculus, Analysis, and Differential Equations ws-book9x6 page 105

Integration of Rational Functions 105

2. x3 − 8 = (x − 2)(x2 + 2x + 4).
3. x4 + x3 − x2 − 5x + 4 = (x − 1)(x3 + 2x2 + x − 4) = (x − 1)2 (x2 + 3x + 4).
4. x6 + 1 = (x2 + 1)(x4 − x2 + 1) = (x2 + 1)(x4 + 2x2 + 1 − 3x2 )
h √ i √ √
= (x2 + 1) (x2 + 1)2 − ( 3x)2 = (x2 + 1)(x2 − 3x + 1)(x2 + 3x + 1).

Theorem 7.1 (Partial Fraction Decomposition).


P (x)
Suppose that is a proper fraction, with the denominator Q(x) factored
Q(x)
into factors of the form
(ax + b)k or (ax2 + bx + c)l ,
where k and l are nonnegative integer exponents, a, b, and c are real coef-
ficients with a 6= 0 and ax2 + bx + c is an irreducible quadratic polynomial.
P (x)
Then is decomposed into a sum of partial fractions in the following
Q(x)
fashion:
• Each factor of the form (ax + b)k (k = 1, 2, . . . ) contributes the sum of
k type 1/type 2 partial fractions:
A1 A2 Ak
+ 2
+ ··· + .
ax + b (ax + b) (ax + b)k
• Each factor of the form (ax2 +bx+c)l (l = 1, 2, . . . ) into Q(x) contributes
the sum of l type 3/type 4 partial fractions:
A1 x + B 1 A2 x + B 2 Al x + Bl
+ + ··· + .
ax2 + bx + c (ax2 + bx + c)2 (ax2 + bx + c)l
• The partial fraction decomposition is obtained by adding the contribu-
tions of all the factors of the denominator.

Algorithm of Partial Fraction Decomposition


P (x)
To decompose a proper fraction into partial fractions,
Q(x)
1. Factor the denominator Q(x) completely.
2. Set up the appropriate form of the partial fraction decomposition by
Theorem 7.1.
3. Clear the denominators multiplying through by the common denomina-
tor Q(x).
May 17, 2018 12:7 ws-book9x6 Integration for Calculus, Analysis, and Differential Equations ws-book9x6 page 106

106 Integration for Calculus, Analysis, and Differential Equations

4. Determine the values of the unknown coefficients by the Method of Un-


determined Coefficients, i.e., by matching the like powers and equating
the coefficients of the polynomials in the left and right sides of the equal-
ity at each one.
5. Substitute the found values for the corresponding coefficients into the
form of step 2 to write the desired partial fraction decomposition.

Example 7.7 (Partial Fraction Decomposition).


Decompose the proper fraction
5x3 − 2x2 + 10x − 5
x4 + x3 − x2 − 5x + 4
into partial fractions.
Solution:

1. Factor the denominator:

x4 + x3 − x2 − 5x + 4 = (x − 1)2 (x2 + 3x + 4)

(see Examples 7.6).


2. Set up the appropriate form of the partial fraction decomposition:
5x3 − 2x2 + 10x − 5 A B Cx + D
2 2
= + 2
+ 2
(x − 1) (x + 3x + 4) x − 1 (x − 1) x + 3x + 4
with the coefficients A, B, C, and D to be determined.
3. Clear the denominators multiplying both sides by the common denom-
inator (x − 1)2 (x2 + 3x + 4):

5x3 −2x2 +10x−5 = A(x−1)(x2 +3x+4)+B(x2 +3x+4)+(Cx+D)(x−1)2

4. Determine the values of the unknown coefficients by the Method of Un-


determined Coefficients.
Matching the like powers x3 , x2 , x1 , and x0 and equating the coefficients
of the polynomials in the left and right sides of the equality at each one,
we obtain the following linear system of four equations in four unknowns:
at x3 : A + C = 5
at x2 : 2A + B − 2C + D = −2
at x1 : A + 3B + C − 2D = 10
at x0 : −4A + 4B + D = −5
(verify), solving which via the elimination method :
May 17, 2018 12:7 ws-book9x6 Integration for Calculus, Analysis, and Differential Equations ws-book9x6 page 107

Integration of Rational Functions 107

A + C = 5
2A + B − 2C + D = −2 E2 +(−2)E1 →E2 , E3 +(−1)E1 →E3
−−−−−−−−−−−−−−−−−−−−−−→
A + 3B + C − 2D = 10 E4 +4E1 →E4
−4A + 4B + D = −5

A + C = 5
B − 4C + D = −12 E3 +(−3)E2 →E3
−−−−−−−−−−−→
3B − 2D = 5 E4 +(−4)E2 →E4
4B + 4C + D = 15

A + C = 5
B − 4C + D = −12 3E4 +(−5)E3 →E4
−−−−−−−−−−−→
12C − 5D = 41
20C − 3D = 63

A + C = 5
B − 4C + D = −12
12C − 5D = 41
16D = −16
solving starting with the last equation and back substituting;
A = 5−3=2
B = −12 + 4 · 3 − (−1) = 1
,
C = [41 + 5(−1)] /12 = 3
D= −1

we arrive at the unique solution: A = 2, B = 1, C = 3, D = −1.


5. Substituting the found values for the corresponding coefficients into the
form of step 2, we obtain the partial fraction decomposition
5x3 − 2x2 + 10x − 5 2 1 3x − 1
2 2
= + 2
+ 2 .
(x − 1) (x + 3x + 4) x − 1 (x − 1) x + 3x + 4

7.4. Partial Fraction Method

The Partial Fraction Method is used for the evaluation of integrals of ra-
tional functions.
May 17, 2018 12:7 ws-book9x6 Integration for Calculus, Analysis, and Differential Equations ws-book9x6 page 108

108 Integration for Calculus, Analysis, and Differential Equations

Given an integral of a rational function


Z
P (x)
dx,
Q(x)
the method is executed as follows:

P (x)
• If the fraction is proper,
Q(x)
1. Decompose into partial fractions.
2. Integrate.
P (x)
• If the fraction is improper,
Q(x)
P (x) R(x)
1. Divide: = S(x) + , where the quotient S(x) and the
Q(x) Q(x)
remainder R(x) are polynomials, the degree of R(x) being less than
the degree of Q(x).
R(x)
2. Decompose the proper fraction into partial fractions.
Q(x)
3. Integrate.

Example 7.8 (Partial Fraction Method).


Evaluate the integral

2x5 + 3x4 + 4x3 − 13x2 + 13x − 1


Z
dx.
x4 + x3 − x2 − 5x + 4
Solution:

1. Since the fraction is improper, we use long division first:

2x5 + 3x4 + 4x3 − 13x2 + 13x − 1 5x3 − 2x2 + 10x − 5


= 2x + 1 +
x4 + x3 − x2 − 5x + 4 x4 + x3 − x2 − 5x + 4
(verify).
2. Decomposing the proper fraction

5x3 − 2x2 + 10x − 5


x4 + x3 − x2 − 5x + 4
into partial fractions (see Example 7.7), we have:

5x3 − 2x2 + 10x − 5 2 1 3x − 1


= + + 2 .
x4 + x3 − x2 − 5x + 4 x − 1 (x − 1)2 x + 3x + 4
May 17, 2018 12:7 ws-book9x6 Integration for Calculus, Analysis, and Differential Equations ws-book9x6 page 109

Integration of Rational Functions 109

3. Now, we integrate:
2x5 + 3x4 + 4x3 − 13x2 + 13x − 1
Z
dx
x4 + x3 − x2 − 5x + 4
Z  
2 1 3x − 1
= 2x + 1 + + + 2 dx
x − 1 (x − 1)2 x + 3x + 4
by the integration rules;
3x − 1
Z Z Z Z Z
1 −2
= 2x dx+ 1 dx+2 dx+ (x−1) dx+ dx
x−1 x2 + 3x + 4
it is easier to see what adjustments are required for the
forthcoming substitution when the latter integral is split as follows:
Z Z
1 x 1
= x2 +x+2 ln |x−1|− +3 dx− dx
x−1 x2 + 3x + 4 x2 + 3x + 4
since [x2 + 3x + 4]0 = 2x + 3,
introducing the missing constants 2 and 3 for the first integral;
2x + 3 − 3
Z Z
1 3 1
= x2 +x+2 ln |x−1|− + dx− dx
x−1 2 x2 + 3x + 4 x2 + 3x + 4
by the integration rules;
Z Z
1 3 2x + 3 9 1
= x2 +x+2 ln |x−1|− + 2
dx− 2
dx
x−1 2 x + 3x + 4 2 x + 3x + 4
Z Z
1 1 3 2x + 3
− dx = x2 +x+2 ln |x−1|− + dx
x2 + 3x + 4 x−1 2 x2 + 3x + 4
Z
11 1
− 2
dx
2 x + 3x + 4
substituting for the first integral:
u = x2 + 3x + 4, du = (2x + 3)dx
and completing the square for the second one:

x2 + 3x + 4 = x2 + 2(3/2)x + 9/4 − 9/4 + 4 = (x + 3/2)2 + ( 7/2)2 ;
Z Z
1 3 1 11 1
= x2 +x+2 ln |x−1|− + du− √ dx
x−1 2 u 2 (x + 3/2) + ( 7/2)2
2

1 3 11 1 x + 3/2
= x2 + x + 2 ln |x − 1| − + ln |u| − √ arctan √ +C
x−1 2 2 7/2 7/2
substituting back and simplifying;
1 3 11 2x + 3
= x2 + x + 2 ln |x − 1| − + ln |x2 + 3x + 4| − √ arctan √ + C
x−1 2 7 7
May 17, 2018 12:7 ws-book9x6 Integration for Calculus, Analysis, and Differential Equations ws-book9x6 page 110

110 Integration for Calculus, Analysis, and Differential Equations

since x2 + 3x + 4 > 0;
1 3 11 2x + 3
= x2 +x+2 ln |x−1|− + ln(x2 +3x+4)− √ arctan √ +C.
x−1 2 7 7

Let us consider one more example, in which we are to deal with a type 4
partial fraction.
Example 7.9 (Partial Fraction Method).
Evaluate the integral
x3 + 1
Z
dx.
x(x2 + x + 1)2

Solution:

1. Observe that the integrand is a proper fraction, whose denominator is


completely factored. Its partial fraction decomposition is of the form:
x3 + 1 A Bx + C Dx + E
= + 2 + .
x(x2 + x + 1)2 x x + x + 1 (x2 + x + 1)2
Multiplying through by the common denominator x(x2 + x + 1)2 , we
have:
x3 + 1 = A(x2 + x + 1)2 + (Bx + C)x(x2 + x + 1) + (Dx + E)x.
Considering that
(x2 + x + 1)2 = x4 + 2x3 + 3x2 + 2x + 1
and
x(x2 + x + 1) = x3 + x2 + x,
comparing the corresponding coefficients, we obtain the following 5 × 5
linear system:
at x4 : A +B =0
at x3 : 2A + B + C =1
at x2 : 3A + B + C + D =0
at x1 : 2A +C +E=0
at x4 : A =1
• From the last equation, A = 1.
• Substituting the found value for A into the first equation and solving
it for B, we find B = −1.
May 17, 2018 12:7 ws-book9x6 Integration for Calculus, Analysis, and Differential Equations ws-book9x6 page 111

Integration of Rational Functions 111

• Substituting the found values for A and B into the second equation
and solving it for C, we find C = 1 − 2 · 1 − (−1) = 0.
• Substituting the found values for A, B, and C into the third equation
and solving it for D, we find D = −3 · 1 − (−1) − 0 = −2.
• Substituting the found values for A and C into the fourth equation
and solving it for E, we find E = −2 · 1 − 0 = −2.
Thus, the desired partial fraction decomposition is
x3 + 1 1 (−1)x (−2)x + (−2)
= + 2 + 2 .
x(x2 + x + 1)2 x x +x+1 (x + x + 1)2
2. Now, we integrate:

x3 + 1
Z Z  
1 (−1)x (−2)x + (−2)
dx = + + dx
x(x2 + x + 1)2 x x2 + x + 1 (x2 + x + 1)2
by the integration rules;
Z Z Z
1 x 2x + 2
= dx − dx − dx
x x2 + x + 1 (x2 + x + 1)2
since [x2 + x + 1]0 = 2x + 1,
introducing the missing constants 2 and 1
for the second integral and expressing 2 as 1 + 1 in the third one;
2x + 1 − 1
Z Z
1 2x + 1 + 1
= ln |x| − dx − dx
2 x2 + x + 1 (x2 + x + 1)2
by the integration rules;
Z Z Z
1 2x + 1 1 1 2x + 1
= ln |x|− dx+ dx− dx
2 x2 + x + 1 2 x2 + x + 1 (x2 + x + 1)2
Z
1
− dx
(x2 + x + 1)2
substituting for the first and third integrals:
u = x2 + x + 1, du = (2x + 1) dx
and completing the square for the second and fourth ones:

x2 + x + 1 = x2 + 2(1/2)x + 1/4 − 1/4 + 1 = (x + 1/2)2 + ( 3/2)2 ;
Z Z Z
1 1 1 1
= ln |x| − du + √ dx − u−2 du
2 u 2 (x + 1/2)2 + ( 3/2)2
Z
1
−  √ 2 dx
(x + 1/2) + ( 3/2)2
2
May 17, 2018 12:7 ws-book9x6 Integration for Calculus, Analysis, and Differential Equations ws-book9x6 page 112

112 Integration for Calculus, Analysis, and Differential Equations

integrating and using reduction formula 11 of Appendix B



with x + 1/2 for x, a = 3/2, and n = 2 for the fourth integral;
1 1 1 x + 1/2 1
= ln |x| −ln |u| + √ arctan √ +
2 2 3/2 3/2 u
 Z 
1 x + 1/2 1
− √ √ + √ dx
2( 3/2)2 (x + 1/2)2 + ( 3/2)2 (x + 1/2)2 + ( 3/2)2
substituting back and simplifying;
1 1 2x + 1 1
= ln |x| − ln |x2 + x + 1| + √ arctan √ + 2
2 3 3 x +x+1
 
2 x + 1/2 2 2x + 1
− + √ arctan √ +C
3 x2 + x + 1 3 3
1 2 x−1 1 2x + 1
= ln |x| − ln |x2 + x + 1| − − √ arctan √ +C
2 3 x2 + x + 1 3 3 3
since x2 + x + 1 > 0;
1 2 x−1 1 2x + 1
= ln |x| − ln(x2 + x + 1) − − √ arctan √ + C.
2 3 x2 + x + 1 3 3 3

7.5. Applications

Examples 7.10 (Applications).

1. Find the volume of the solid obtained by rotating the region bounded
1 1
by y = p and y = on the interval [1, 4] about the x-axis.
x(5 − x) 2
Solution: Considering that
1 1
0≤ p ≤ on [1, 4],
x(5 − x) 2
by the washer method ,
 !2 
Z 4  2
1 1
V = π − p  dx by the integration rules;
1 2 x(5 − x)
 Z 4 Z 4 
1 1
=π 1 dx − dx .
4 1 1 x(5 − x)
May 17, 2018 12:7 ws-book9x6 Integration for Calculus, Analysis, and Differential Equations ws-book9x6 page 113

Integration of Rational Functions 113

1
To decompose the proper fraction , let us use the following
x(5 − x)
shortcut, similar to how it is done when transforming products into sums
(see Sec. 2.2.5):
 
1 1 1 1
= + .
x(5 − x) 5 x 5−x
Hence,
 Z 4 Z 4 
1 1
V =π 1 dx − dx by integration rules;
4 1 1 x(5 − x)
 Z 4 Z 4 Z 4 
1 1 1 1
=π 1 dx − dx + dx
4 1 5 1 x 1 5−x
by the Newton-Leibniz Formula;
" 4 4 4 !#
1 1
=π x − ln |x| − ln |5 − x|
4 1 5 1 1
 
1 1
= π (4 − 1) − (ln 4 − ln 1 − [ln 1 − ln 4])
4 5
since ln 1 = 0 and ln 4 = 2 ln 2;
   
3 2 ln 4 3 4 ln 2
=π − =π − un.3 .
4 5 4 5

2. Find the volume of the solid obtained by rotating the region bounded
1
by y = 2 and x-axis on the interval [3, 4] about y-axis.
x (x − 2)
Solution: Since the rotation axis is perpendicular to the axis of defi-
nition, by the shell method ,
Z 4
1
V = 2πx 2 dx by the integration rules;
3 x (x − 2)
Z 4
1
= 2π dx
3 x(x − 2)
decomposing into partial fractions via the shortcut:
 
1 1 1 1
= − (cf. Sec. 2.2.5);
x(x − 2) 2 x−2 x
Z 4  
1 1 1
= 2π − dx by the integration rules;
3 2 x−2 x
May 17, 2018 12:7 ws-book9x6 Integration for Calculus, Analysis, and Differential Equations ws-book9x6 page 114

114 Integration for Calculus, Analysis, and Differential Equations

Z 4 Z 4 
1 1 1
= 2π dx − dx by the Newton-Leibniz Formula;
2 3 x−2 3 x
" 4 4 #

= π ln |x − 2| − ln |x| = π [ln 2 − ln 1 − (ln 4 − ln 3)]
3 3

since ln 1 = 0 and ln 4 = 2 ln 2;
3
= π [ln 3 − ln 2] = π ln un.3 .
2

7.6. Practice Problems

Evaluate the integrals.


Z
2x + 3 x2
Z
1. dx 8. dx
(x + 2)(x − 5) x2 + 2x + 1
Z
x x3
Z
2. 2
dx 9. dx
x +x−2 (x − 1)100
Z
x Z
x11
3. dx 10. dx
(x2 + 1)(x2 + 4) x + 3x4 + 2
8
Z
1 Z
1
4. 3
dx 11. dx
x +1 x(x − 1)2
10
x3 + 1
Z
1 − x7
Z
5. dx 12. dx
x3 − 5x2 + 6x x(x7 + 1)
Z  2
x Z
x4 + 1
6. 2
dx 13. dx
x + 3x + 2 x(x4 + 5)(x5 + 5x + 1)
Z
1 Z
x2 + 1
7. dx 14. dx
(x4 − 1)2 x4 + 3x2 + 1

Hint: In problems 8–14, it is recommended to avoid the direct application


of the Partial Fraction Method.
May 17, 2018 12:7 ws-book9x6 Integration for Calculus, Analysis, and Differential Equations ws-book9x6 page 115

Chapter 8

Rationalizing Substitutions

Definition 8.1 (Rationalizing Substitution).


A rationalizing substitution is a substitution transforming an integral of an
irrational function, which may contain radicals, exponentials, logarithms,
or trigonometric functions, into an integral of a rational function.
In this case, we say that the integral is rationalized.

Here, we consider several integration scenarios implemented via rationaliz-


ing substitutions.

8.1. Integrals with Radicals

Integrals with radicals may be rationalized via an appropriate substitutions.


s !
ax + b
Z
n
8.1.1. Integrals of the Form R x, dx
cx + d
An integral of the form
Z r !
n ax + b
R x, dx,
cx + d
where R(·, ·) is a rational function of two variables, is rationalized via the
substitution
r
ax + b
t= n .
cx + d
Example 8.1 (Rationalizing Substitution).
Z √
x+1
dx (a = b = d = 1, c = 0)
x

115
May 17, 2018 12:7 ws-book9x6 Integration for Calculus, Analysis, and Differential Equations ws-book9x6 page 116

116 Integration for Calculus, Analysis, and Differential Equations


substituting: t = x + 1, x = t2 − 1, dx = 2t dt ;
Z
t
= 2t dt by the integration rules;
t2 − 1
t2 t2
Z
1
=2 dt dividing: =1+ 2 (verify);
t2 −1 t2 −1 t −1
Z  
1
=2 1+ 2 dt by the integration rules;
t −1
Z Z 
1
=2 1 dt + dt
t2 − 1
we need not use the Partial Fraction Method for the second integral,
it is a “tall logarithm” (see the Table of Basic Integrals (Appendix A));
 
1 t − 1
= 2 t + ln +C substituting back and simplifying;
2 t + 1



x + 1 − 1
= 2 x + 1 + ln √ + C.
x + 1 + 1

Z  
8.1.2. Integrals of the Form R x, xm1 /n1 , . . . , xmk /nk dx

An integral of the form


Z  
R x, xm1 /n1 , . . . , xmk /nk dx,

where R(·, . . . , ·) is a rational function of k + 1 variables, is rationalized via


the substitution
t = x1/n or x = tn ,
m1 mk
where n is the least common denominator of all the fractions ,..., ,
n1 nk
which eliminates all the radicals.
Example 8.2 (Rationalizing Substitution).
Z Z
1 1
√ √ dx = dx
3
x+ x
4
x + x1/4
1/3

1 1
since 12 is the least common denominator of and ,
3 4
substituting: x = t12 (t ≥ 0), dx = 12t11 dt ;
May 17, 2018 12:7 ws-book9x6 Integration for Calculus, Analysis, and Differential Equations ws-book9x6 page 117

Rationalizing Substitutions 117

12t11
Z
= 1/3 1/4
dt simplifying;
[t12 ] + [t12 ]
12t11 12t11 12t8
Z Z Z
= dt = dt = dt
t12/3 + t12/4 t4 + t3 t+1
by the integration rules and long/synthetic division (verify);
t8
Z Z  
1
= 12 dt = 12 t7 − t6 + t5 − t4 + t3 − t2 + t − 1 + dt
t+1 t+1
Z Z Z Z Z Z Z
= 12 t7 dt − t6 dt + t5 dt − t4 dt + t3 dt − t2 dt + t dt
Z Z 
1
− 1 dt + dt
t+1
 8
t7 t6 t5 t4 t3 t2

t
= 12 − + − + − + − t + ln |t + 1| + C
8 7 6 5 4 3 2
substituting back: t = x1/12 and simplifying;
3 12 12
= x2/3 − x7/12 + 2x1/2 − x5/12 + 3x1/3 − 4x1/4 + 6x1/6
2 7 5
− 12x1/12 + 12 ln |x1/12 + 1| + C.

8.2. Integrals with Exponentials

An integral of the form


Z
R (ax ) dx,
where R(·) is a rational function, a > 0, a 6= 1, is rationalized via the
substitution
t = ax or x = loga t.
Example 8.3 (Rationalizing Substitution).
Z
1 1
3x x
dx (a = e) substituting: t = ex , x = ln t, dx = dt ;
e −e t
Z Z
1 1 1
= dt = dt
t3 − t t t2 (t2 − 1)
transforming product into sum,
1 1 1
= 2 − (cf. Examples 2.10);
t2 (t2 − 1) t − 1 t2
May 17, 2018 12:7 ws-book9x6 Integration for Calculus, Analysis, and Differential Equations ws-book9x6 page 118

118 Integration for Calculus, Analysis, and Differential Equations

Z  
1 1
= − dt by the integration rules;
t2 − 1 t2
Z Z
1 1
= 2
− dt
t −1 t2
we need not use the Partial Fraction Method for the first integral,
it is a “tall logarithm” (see the Table of Basic Integrals (Appendix A));

1 t − 1 1
= ln + +C substituting back;
2 t + 1 t
x
1 e − 1
= ln x + e−x + C.
2 e + 1

8.3. Trigonometric Integrals


Z
8.3.1. Integrals of the Form R (tan x) dx

An integral of the form


Z
R (tan x) dx,

where R(·) is a rational function, is rationalized via the following substitu-


tion:
1
t = tan x, x = arctan t, dx = 2 dt.
t +1
Example 8.4 (Rationalizing Substitution).
Z
1
tan5 x dx substituting: t = tan x, x = arctan t, dx = dt ;
t2 + 1
t5 t5
Z
t
= dt by long division: = t3 − t + 2 (verify);
t2 +1 t2 +1 t +1
Z  
t
= t3 − t + 2 dt by the integration rules;
t +1
t4 t2
Z Z Z Z
3 t t
= t dt − t dt + dt = − + dt
t2 + 1 4 2 t2 + 1
introducing the missing constant 2;
t4 t2
Z Z Z Z
3 t 1 2t
= t dt − t dt + dt = − + dt
t2 + 1 4 2 2 t2 + 1
May 17, 2018 12:7 ws-book9x6 Integration for Calculus, Analysis, and Differential Equations ws-book9x6 page 119

Rationalizing Substitutions 119

substituting: u = t2 + 1, du = 2tdt
t4 t2 t4 t2
Z
1 1 1
= − + du = − + ln |u| + C substituting back;
4 2 2 u 4 2 2
tan4 x tan2 x 1
= − + ln(tan2 x + 1) + C
4 2 2
by the trigonometric identity tan2 x + 1 = sec2 x and the laws of logarithms;
tan4 x tan2 x √ tan4 x tan2 x
= − + ln sec2 x + C = − + ln | sec x| + C.
4 2 4 2

Remark 8.1. Applying reduction formula 5 of Appendix B twice yields


the same result (check ).
Z
8.3.2. Integrals of the Form R (sin x, cos x) dx

An integral of the form


Z
R (sin x, cos x) dx,

where R(·, ·) is a rational function of two variables, is rationalized via the


following universal substitution (or Weierstrass1 substitution):
x
t = tan or x = 2 arctan t.
2
The substitution is rationalizing since
2
dx = dt,
1 + t2
and, by the trigonometric identities (Appendix C),
x x
2 tan 1 − tan2 2
sin x = 2 = 2t
, cos x = 2 = 1−t .
x 1 + t2 x 1 + t2
1 + tan2 1 + tan2
2 2
Example 8.5 (Rationalizing Substitution).
Z
1
dx
cos x + 2
x 2
t = tan , x = 2 arctan t, dx = dt
substituting: 2 1 + t2 ;
2
1−t
cos x =
1 + t2
1 Karl Weierstrass (1815 –1897)
May 17, 2018 12:7 ws-book9x6 Integration for Calculus, Analysis, and Differential Equations ws-book9x6 page 120

120 Integration for Calculus, Analysis, and Differential Equations

Z
1 2
= 2 dt simplifying and using the integration rules;
1−t 1 + t2
2
+2
Z 1+t Z
2 1 2 t
= 2 2
dt = 2 √ dt = √ arctan √ + C
1 − t + 2(1 + t ) 2
t + ( 3) 2 3 3
substituting back;
 
2 1 x
= √ arctan √ tan + C.
3 3 2

Remark 8.2. Although the universal substitution always rationalizes the


integrals of the above form, it may lead to cumbersome computations and
is recommended to be avoided whenever possible. Thus,

• for the integral


Z
cos x
dx,
sin x + 1
we can avoid the universal substitution applying the simpler one:
t = sin x + 1 ;
• the integral
Z
1
dx,
1 − sin x
can be found directly (see Examples 5.1).

General Guidelines
on Avoiding the Universal Substitution

• If R(− sin x, cos x) = −R(sin x, cos x) (oddness relative to sin x), the
substitution t = cos x can be used (cf. Sec. 5.2.2).
• If R(sin x, − cos x) = −R(sin x, cos x) (oddness relative to cos x), the
substitution t = sin x can be used (cf. Sec. 5.2.2).
• If R(− sin x, − cos x) = R(sin x, cos x) (joint evenness relative to sin x
1
and cos x), the substitution t = tan x, x = arctan t, dx = 2 dt
t +1
can be used considering that
tan2 x t2 1 1
sin2 x = = and cos2 x = = 2 .
tan2 x + 1 t2 + 1 2
tan x + 1 t +1
May 17, 2018 12:7 ws-book9x6 Integration for Calculus, Analysis, and Differential Equations ws-book9x6 page 121

Rationalizing Substitutions 121

Example 8.6 (Avoiding the Universal Substitution).

sin2 x
Z
dx dividing;
sin2 x + 1
Z  
1
= 1− 2 dx by the integration rules;
sin x + 1
Z Z Z
1 1
= 1 dx − 2 dx = x − 2 dx
sin x + 1 sin x + 1
since the integrand is jointly even relative to sin x and cos x,
1
t = tan x, x = arctan t, dx = dt
substituting: t2 +1 ;
t2
sin2 x =
t2 + 1
Z
1 1
=x− 2 dt
simplifying and integrating;
t t2 +1
2
+ 1
Z t +1 Z √
1 1 1
=x− dt = x − √ dt = x − √ arctan( 2t) + C
2t2 + 1 2
( 2t) + 1 2
substituting back;
1 √
= x − √ arctan( 2 tan x) + C.
2

8.4. Applications

Examples 8.7 (Applications).

1
1. Find the area of the region bounded by y = and the x-axis on
ex +1
the interval [0, 1].
1
Solution: Since y = > 0 on [0, 1],
ex + 1
Z 1
1
A= dx
0 ex + 1
substituting and changing the integration limits:
May 17, 2018 12:7 ws-book9x6 Integration for Calculus, Analysis, and Differential Equations ws-book9x6 page 122

122 Integration for Calculus, Analysis, and Differential Equations

x t
1
t = ex , x = ln t, dx = dt 0 1 ;
t
1 e
Z e
1
= dt
1 (t + 1)t
transforming product into sum:
1 1 1
= − (cf. Examples 2.10);
(t + 1)t t t+1
Z e  
1 1
= − dt by the integration rules;
1 t t+1
Z e Z e
1 1
= dt − dt by the Newton-Leibniz Formula;
1 t 1 t+1
e e

= ln |t| − ln |t + 1| = ln e − ln 1 − [ln(e + 1) − ln 2]
1 1
considering that ln e = 1 and ln 1 = 0;
2
= 1 − ln(e + 1) + ln 2 un. .

2. Find the volume of the solid obtained by rotating the region bounded
1
by y = √ 3
and the x-axis on the interval [0, 7] about the y-axis.
x+1
Solution: Since the rotation axis is perpendicular to the axis of def-
inition, by the shell method ,
Z 7
1
V = 2πx √ 3
dx by the integration rules;
0 x+1
Z 7
x
= 2π √
3
dx
0 x +1
to rationalize, substitute:

√ x t
t= 3
x + 1, x = t3 − 1, dx = 3t2 dt 0 1 ;
7 2
2 3
t −1 2
Z
= 2π 3t dt multiplying and dividing;
1 t
Z 2
= 2π 3[t4 − t] dt by the integration rules;
1
May 17, 2018 12:7 ws-book9x6 Integration for Calculus, Analysis, and Differential Equations ws-book9x6 page 123

Rationalizing Substitutions 123

Z 2 Z 2 Z 2 
4 4
= 6π [t − t] dt = 6π t dt − t dt
1 1 1

by the Newton-Leibniz Formula;


" #
2 2
t5 t2
   
1 5 5 1 2 2 31 3
= 6π − = 6π (2 − 1 ) − (2 − 1 ) = 6π −
5 1 2 1 5 2 5 2
141π
= un.3 .
5

Observe that the latter integral can be evaluated without being ratio-
nalized as follows:

Z 7
1
V = 2πx √
3
dx by the integration rules;
0 x+1
Z 7
x
= 2π √
3
dx
0 x+1

x t
substituting : t = x + 1, x = t − 1, dx = dt 0 1 ;
7 8
8 8
t−1 t−1
Z Z
= 2π √ dt = 2π dt
1
3
t 1 t1/3
dividing termwise and using the laws of exponents (see Appendix C);
Z 8  Z 8h
t 1 2/3 −1/3
i
= 2π − dt = 2π t − t dt
1 t1/3 t1/3 1

by the integration rules;


Z 8 Z 8 
−1/3
= 2π t2/3 dt − t dt by the Newton-Leibniz Formula;
1 1
" 8 8 #  
3 5/3 3 2/3 3 5/3 5/3 3 2/3 2/3
= 2π t − t = 2π (8 − 1 ) − (8 − 1 )
5 1 2 1 5 2
   
93 9 93 9 141π
= 2π − = 2π − = un.3 (cf. Sec. 3.1.6).
5 2 5 2 5
May 17, 2018 12:7 ws-book9x6 Integration for Calculus, Analysis, and Differential Equations ws-book9x6 page 124

124 Integration for Calculus, Analysis, and Differential Equations

8.5. Practice Problems

Evaluate the integrals.


Z
x+1 e2x + 2ex
Z
1. √ dx 8. dx
Z
2x + 1 e2x + 1
1
Z
2. √3
dx 9. tan4 x dx
3x − 1 + 1
sin2 x
Z r
x
Z
3. dx 10. dx
x+2 cos4 x
Z
x3 1
Z
4. √ dx 11. dx
x2 + 6 1 + 3 cos2 x
Z √ Z
1
x 12. dx
5. √ dx 5 + 3 cos x
x+1 Z
Z
1 1
6. √ √ dx 13. dx
x+ 3x 3 sin x + 4 cos x
Z
1
e3x
Z
14. dx
7. dx 2 sin x − cos x + 5
ex − 2
May 17, 2018 12:7 ws-book9x6 Integration for Calculus, Analysis, and Differential Equations ws-book9x6 page 125

Can We Integrate Them All Now?

Although the Fundamental Theorem of Calculus (Part 1) (Theorem 1.5)


guarantees the existence of an antiderivative F for each function f contin-
uous on an interval I, it says nothing about how to find it and whether it
is feasible in terms of the known elementary functions.

Definition 8.2 (Elementary Function).


Elementary functions are

1. Constants.
2. Powers xp .
3. Exponentials ax and their inverses, i.e., logarithms loga x.
4. Trigonometric functions cos x, sin x, tan x, cot x, sec x, csc x and their
inverses arccos x, arcsin x, arctan x, arccot x, arcsec x, arccsc x.
5. Hyperbolic functions cosh x, sinh x, tanh x, coth x, sech x, csch x and
their inverses.
6. All the functions that can be obtained from the above via combinations
(addition/subtraction, multiplication/division) and compositions.

Some indefinite integrals, although exist, cannot be evaluated in terms of


the elementary functions, e.g.,
Z x
e
1. dx (integral exponential ),
x
Z
1
2. dx (integral logarithm)
ln x
Z
sin x
3. dx (integral sine),
x
Z
4. sin(ex ) dx,

125
May 31, 2018 10:33 ws-book9x6 Integration for Calculus, Analysis, and Differential Equations ws-book9x6 page 126

126 Integration for Calculus, Analysis, and Differential Equations

Z
2
5. ex dx (Poisson integral ),
Z Z
2
6. sin(x ) dx, cos(x2 ) dx (Fresnel integrals),
Z p
7. x3 + 1 dx.

Remark 8.3. Observe that integral 2 can be reduced to integral 1 via


the substitution x = et and integral 4 to integral 3 via the substitution
t = ex (verify).
May 17, 2018 12:7 ws-book9x6 Integration for Calculus, Analysis, and Differential Equations ws-book9x6 page 127

Chapter 9

Improper Integrals

There are two essential restrictions in the theory of definite integral


Z b
f (x) dx: both the integration interval and the integrand must be
a
bounded (see Sec. 1.2.2).
The notion of improper integral is introduced in order to overcome these
restrictions, i.e., to be able to integrate over unbounded intervals or have
unbounded integrands. There are two types of improper integrals accord-
ingly.

9.1. Type 1 Improper Integrals (Unbounded Interval)

Definition 9.1 (Type 1 Improper Integrals (Unbounded Interval)).


A type 1 improper integral is an integral over an unbounded interval.
Depending on the kind of the unboundedness of the integration interval
(right-sided, left-sided, or two-sided ), there are three cases:
Z ∞ Z b Z ∞
f (x) dx, f (x) dx, or f (x) dx,
a −∞ −∞

where a and b are real numbers.

9.1.1. Right-Sided Unboundedness


Z ∞
Definition 9.2 (Improper Integral f (x) dx).
a
Suppose that a function f : [a, ∞) → R, where a is a real number, is
integrable on the interval [a, t], i.e., the definite integral
Z t
f (x) dx
a

127
May 17, 2018 12:7 ws-book9x6 Integration for Calculus, Analysis, and Differential Equations ws-book9x6 page 128

128 Integration for Calculus, Analysis, and Differential Equations

exists (see Definition 1.6), for each t ≥ a.

Remark 9.1. In particular, this is the case if f is continuous on [a, ∞)


(Corollary 1.1).

The improper integral of f from a to ∞


Z ∞
f (x) dx
a

is said to converge or to be convergent if the limit


Z t
lim f (x) dx
t→∞ a

exists and is finite, in which case we say that the limit is the integral’s value
and write
Z ∞ Z t
f (x) dx = lim f (x) dx.
a t→∞ a

If the limit either does not exists or is infinite, the improper integral is said
to diverge or to be divergent. In the latter case, we assign to the improper
integral the value ±∞, respectively, and write
Z ∞
f (x) dx = ±∞.
a

Examples 9.1 (Improper Integrals).


Evaluate the improper integral or show that it diverges.
Z ∞
1. cos x dx is divergent since
0
Z t t

lim cos x dx = lim sin x = lim [sin t − sin 0] = lim sin t
t→∞ 0 t→∞ t→∞ t→∞
0
does not exist.

Z ∞
2. 1 dx
is divergent since
0
Z t t

lim 1 dx = lim x = lim [t − 0] = lim t = ∞.
t→∞ 0 t→∞ t→∞ t→∞
0
Z ∞
Hence, 1 dx = ∞.
0
May 17, 2018 12:7 ws-book9x6 Integration for Calculus, Analysis, and Differential Equations ws-book9x6 page 129

Improper Integrals 129

Z ∞
1
3. √ dx is divergent since
1
3
x
Z t t
−1/3 3 2/3 3 h 2/3 i
lim x dx = lim x = lim t − 1 = ∞.
t→∞ 1 t→∞ 2 t→∞ 2
1
Z ∞
1
Hence, √ dx = ∞.
1
3
x

Z ∞
1
4. dx is convergent since
1 x2
Z t   t  
1 1 1
lim dx = lim − = lim 1 − = 1 − 0 = 1.
t→∞ 1 x2 t→∞ x 1 t→∞ t
Z ∞
1
Hence, dx = 1.
1 x2

Z ∞
3
5. x2 e−x dx is convergent since
0
Z t
3
lim x2 e−x dx introducing the missing constant 3;
t→∞ 0

1 t −x3 2
Z
= lim e 3x dx
t→∞ 3 0

substituting and changing the integration limits:

x u
u = x3 , du = 3x2 dx, t t3 ;
0 0
Z t3
1
= lim e−u du by the Newton-Leibniz Formula;
t→∞ 3 0
t3
1  −u  1h 3
i 1 1
= lim −e = lim 1 − e−t = [1 − 0] = .
t→∞ 3 t→∞ 3 3 3
0

Theorem 9.1 (Type 1 p-Integrals).


Let a > 0. Then the improper integral
Z ∞
1
p
dx
a x
converges for p > 1 and diverges for p ≤ 1.
May 17, 2018 12:7 ws-book9x6 Integration for Calculus, Analysis, and Differential Equations ws-book9x6 page 130

130 Integration for Calculus, Analysis, and Differential Equations

Proof.
(a) For p > 1,
Z t t  
1 1 1 1
x−p dx = lim x−p+1 = lim

lim −
t→∞ a t→∞ −p + 1 t→∞ −p + 1 tp−1 ap−1
a
considering that p − 1 > 0;
 
1 1 1
= 0 − p−1 = .
−p + 1 a (p − 1)ap−1
Z ∞
1 1
Hence, p
dx = p−1
.
a x (p − 1)a

(b) For p < 1,


Z t t
1 1
x−p dx = lim x−p+1 = lim
 1−p
− a1−p

lim t
t→∞ a t→∞ −p + 1 t→∞ −p + 1
a
considering that 1 − p > 0;
= ∞.
Z ∞
1
Hence, dx = ∞.
a xp

(c) For p = 1,
Z t t
1
lim dx = lim ln |x| = lim [ln t − ln a] = ∞.
t→∞ a x t→∞ t→∞
a
Z ∞
1
Hence, dx = ∞.
a x

9.1.2. Left-Sided Unboundedness


Z b
Definition 9.3 (Improper Integral f (x) dx).
−∞
Suppose that a function f : (−∞, b] → R, where b is a real number, is
integrable on the interval [t, b], i.e., the definite integral
Z b
f (x) dx
t
May 17, 2018 12:7 ws-book9x6 Integration for Calculus, Analysis, and Differential Equations ws-book9x6 page 131

Improper Integrals 131

exists, for each t ≤ b.

Remark 9.2. In particular, this is the case if f is continuous on (−∞, b].

The improper integral of f from −∞ to b


Z b
f (x) dx
−∞

is said to converge or to be convergent if the limit


Z b
lim f (x) dx
t→−∞ t
exists and is finite, in which case we say that the limit is the integral’s value
and write
Z b Z b
f (x) dx = lim f (x) dx.
−∞ t→−∞ t

If the limit either does not exists or is infinite, the improper integral is said
to diverge or to be divergent. In the latter case, we assign to the improper
integral the value ±∞, respectively, and write
Z b
f (x) dx = ±∞.
−∞

Examples 9.2 (Improper Integrals).


Evaluate the improper integral or show that it diverges.
Z 0
1. sin x dx is divergent since
−∞
Z 0 0

lim sin x dx = lim [− cos x] = lim [cos t − cos 0]
t→−∞ t t→−∞ t→−∞
t
= lim [cos t − 1] does not exist.
t→−∞

Z −1
1
2. dx is convergent since
−∞ x2
Z −1   −1  
1 1 1
lim dx = lim − = lim 1 + = 1 + 0 = 1.
t→−∞ t x2 t→−∞ x t t→∞ t
Z −1
1
Hence, dx = 1.
−∞ x2
May 17, 2018 12:7 ws-book9x6 Integration for Calculus, Analysis, and Differential Equations ws-book9x6 page 132

132 Integration for Calculus, Analysis, and Differential Equations

Z 0
3. ex dx is convergent since
−∞
Z 0 0
ex dx = lim ex = lim [1 − et ] = 1 − 0 = 1.

lim
t→−∞ t t→−∞ t→−∞
t
Z 0
Hence, ex dx = 1.
−∞

9.1.3. Two-Sided Unboundedness


Z ∞
Definition 9.4 (Improper Integral f (x) dx).
−∞
Suppose that a function f : (−∞, ∞) → R and, for any real a < b, is
integrable on the interval [a, b], i.e., the definite integral
Z b
f (x) dx
a
exists.

Remark 9.3. In particular, this is the case if f is continuous on (−∞, ∞).

If for some real a, both improper integrals


Z a Z ∞
f (x) dx and f (x) dx (9.1)
−∞ a

converge, the improper integral of f from −∞ to ∞ is said to converge or


to be convergent and its value is defined as the sum of the values of the two
one-sided improper integrals:
Z ∞ Z a Z ∞
f (x) dx = f (x) dx + f (x) dx. (9.2)
−∞ −∞ a

Remark 9.4. If both improper integrals in (9.1) converge for some real a,
by the additivity property of definite integral, they converge for any real a
and their sum in (9.2) is independent of the choice of a.

If for some real a, at least one of the one-sided improper integrals in (9.1)
diverges, the two-sided improper integral
Z ∞
f (x) dx
−∞

is said to diverge or to be divergent.


May 17, 2018 12:7 ws-book9x6 Integration for Calculus, Analysis, and Differential Equations ws-book9x6 page 133

Improper Integrals 133

Examples 9.3 (Improper Integrals).


Evaluate the improper integral or show that it diverges.
Z ∞
1
1. 2
dx is convergent since
−∞ x + 1
Z ∞ Z t t
1 1
(a) 2+1
dx = lim 2+1
dx = lim arctan x = lim [arctan t
0 x t→∞ 0 x t→∞
0
t→∞
π
− arctan 0] = converges and
2
(b) by the symmetry (the integrand is even),
Z 0
1 π
2+1
dx = converges as well.
−∞ x 2
Z ∞ Z 0 Z ∞
1 1 1 π π
Hence, 2
dx = 2
dx+ 2
dx = + = π.
−∞ x + 1 −∞ x + 1 0 x +1 2 2

Z ∞
2. ex dx is divergent since
−∞
Z ∞ Z t t
x
e dx = lim e = lim [et − 1] = ∞,
x x

e dx = lim
0 t→∞ 0 t→∞ t→∞
0
Z 0
although (see Examples 9.2) ex dx = 1.
−∞
Z ∞
x
Hence, e dx = 1 + ∞ = ∞.
−∞

Geometric Interpretation
The value of a type 1 improper integral
Z ∞ Z b Z ∞
f (x) dx, f (x) dx, or f (x) dx,
a −∞ −∞
when exists, finite or infinite, represents the net area of the region bounded
by the graph of f (x) and the x-axis on the corresponding interval.
When such a value does not exist, Z as, e.g., for the improper integral

cos x dx,
0
the net area is undefined.
Theorem 9.2 (Comparison Test).
Suppose that functions f, g : [a, ∞) → R, where a is a real number, are
May 17, 2018 12:7 ws-book9x6 Integration for Calculus, Analysis, and Differential Equations ws-book9x6 page 134

134 Integration for Calculus, Analysis, and Differential Equations

integrable on the interval [a, t] for each t ≥ a and satisfy the inequality
0 ≤ f (x) ≤ cg(x)
with some c > 0 for all sufficiently large x-values.
Then
Z ∞ Z ∞
1. If g(x) dx is convergent, then so is f (x) dx.
a a
Z ∞ Z ∞
2. If f (x) dx is divergent, then so is g(x) dx.
a a

Remark 9.5. The analogous tests are in place for the cases of (−∞, b] and
(−∞, ∞).
Example 9.4 (Using Comparison Test).
Determine whether the improper integral
Z ∞
1 + sin x
√ dx
1 x 10 x
is convergent or divergent.
Solution: Since
−1 ≤ sin x ≤ 1 for x ≥ 1,
then
1 + sin x 1 + sin x 2
0≤ √ = ≤ 1.1 for x ≥ 1
x 10 x x1.1 x
and hence, by the Comparison Test, the improper integral
Z ∞
1 + sin x
√ dx
1 x 10 x
converges along with the improper integral
Z ∞
1
1.1
dx,
1 x
which is a p-integral with p = 1.1 > 1.

9.2. Type 2 Improper Integrals (Unbounded Integrand)

Definition 9.5 (Type 2 Improper Integrals (Unbounded Inte-


grand)).
A type 2 improper integral is an integral of an unbounded function over a
bounded interval.
Depending on the kind of the unboundedness of the integrand (at the left
endpoint, at the right endpoint, or inside the interval ), there are also three
cases.
May 17, 2018 12:7 ws-book9x6 Integration for Calculus, Analysis, and Differential Equations ws-book9x6 page 135

Improper Integrals 135

9.2.1. Unboundedness at the Left Endpoint


Definition 9.6 (Unboundedness at the Left Endpoint).
Suppose that a function f : (a, b] → R, where a < b are real numbers,
satisfy the following conditions
(a) f (x) → ∞ or f (x) → −∞ as x → a+;
(b) f is integrable on the interval [t, b], i.e., the definite integral
Z b
f (x) dx
t
exists, for each a < t ≤ b.
The improper integral of f from a to b
Z b
f (x) dx
a
is said to converge or to be convergent if the limit
Z b
lim f (x) dx
t→a+ t
exists and is finite, in which case we say that the limit is the integral’s value
and write
Z b Z b
f (x) dx = lim f (x) dx.
a t→a+ t
If the limit either does not exists or is infinite, the improper integral is said
to diverge or to be divergent, in the latter case, we assign to the improper
integral the value ±∞, respectively, and write
Z b
f (x) dx = ±∞.
a

Examples 9.5 (Improper Integrals).


Evaluate the improper integral or show that it diverges.
Z 1
1
1. √ dx is convergent since
0 x
Z 1 1
−1/2
dx = lim 2x = lim 2[1 − t1/2 ] = 2[1 − 0] = 2.
1/2

lim x
t→0+ t t→0+ t→0+
t
Z 1
1
Hence, √ dx = 2.
0 x
May 17, 2018 12:7 ws-book9x6 Integration for Calculus, Analysis, and Differential Equations ws-book9x6 page 136

136 Integration for Calculus, Analysis, and Differential Equations

Z 1
1
2. dx is divergent since
0 x2
Z 1   1  
1 1 1
lim dx = lim − = lim − 1 = ∞.
t→0+ t x2 t→0+ x t t→0+ t
Z 1
1
Hence, 2
dx = ∞.
0 x

Theorem 9.3 (Type 2 p-Integrals).


Let b > 0. Then the improper integral
Z b
1
p
dx
0 x

converges for p < 1 and diverges for p ≥ 1.

Proof.

(a) For p < 1,


Z b b
−p 1 −p+1 1  1−p
− t1−p

lim x dx = lim x = t→0+
lim b
t→0+ t t→0+ −p + 1 −p + 1
t
considering that 1 − p > 0;
1 b1−p
b1−p − 0 =
 
= .
−p + 1 1−p
Z b
1 b1−p
Hence, p
dx = .
0 x 1−p

(b) For p > 1,


Z b b  
−p 1 −p+1
1 1 1
lim x dx = lim x = t→0+
lim − p−1
t→0+ t t→0+ −p + 1
t −p + 1 bp−1 t
considering that p − 1 > 0;
= ∞.
Z b
1
Hence, dx = ∞.
0 xp
May 17, 2018 12:7 ws-book9x6 Integration for Calculus, Analysis, and Differential Equations ws-book9x6 page 137

Improper Integrals 137

(c) For p = 1,
Z b b
1
lim dx = lim ln |x| = lim [ln b − ln t] = ∞.
t→0+ t x t→0+ t→0+
t
Z b
1
Hence, dx = ∞.
0 x

9.2.2. Unboundedness at the Right Endpoint


Definition 9.7 (Unboundedness at the Right Endpoint).
Suppose that a function f : [a, b) → R, where a < b are real numbers,
satisfies the following conditions

(a) f (x) → ∞ or f (x) → −∞ as x → b−;


(b) f is integrable on the interval [a, t], i.e., the definite integral
Z t
f (x) dx
a

exists, for each a ≤ t < b.

The improper integral of f from a to b


Z b
f (x) dx
a

is said to converge or to be convergent if the limit


Z t
lim f (x) dx
t→b− a

exists and is finite, in which case we say that the limit is the integral’s value
and write
Z b Z t
f (x) dx = lim f (x) dx.
a t→b− a

If the limit either does not exists or is infinite, the improper integral is said
to diverge or to be divergent. In the latter case, we assign to the improper
integral the value ±∞, respectively, and write
Z b
f (x) dx = ±∞.
a
May 17, 2018 12:7 ws-book9x6 Integration for Calculus, Analysis, and Differential Equations ws-book9x6 page 138

138 Integration for Calculus, Analysis, and Differential Equations

Examples 9.6 (Improper Integrals).


Evaluate the improper integral or show that it diverges.
Z π/2
1. tan x dx diverges since
0
Z π/2 Z t t

tan x dx = lim tan x dx = lim ln | sec x|
0 t→π/2− 0 t→π/2− 0

= lim ln | sec t| since sec t → ∞ as t → π/2− ;


t→π/2−

= ∞.
Z π/2
Hence, tan x dx = ∞.
0

Z 2
1
2. √ dx converges since
0 2−x
Z t h i t √ √
−1/2 1/2
lim (2 − x) dx = lim −2(2 − x) = lim 2[ 2 − 2 − t]
t→2− 0 t→2− t→2−
0
√ √
= 2[ 2 − 0] = 2 2.
Z 2
1 √
Hence, √ = 2 2.
0 2−x

9.2.3. Unboundedness Inside the Interval


Definition 9.8 (Unboundedness Inside the Interval).
Suppose that a function f is defined on an interval [a, b] (−∞ < a < b < ∞),
except, possibly, at a point a < c < b, and satisfies the following conditions:
(a) f (x) → ∞ or f (x) → −∞ as x → c− and x → c+;
(b) f is integrable on the interval [a, t], i.e., the definite integral
Z t
f (x) dx
a
exists, for each a ≤ t < c.
(c) f is integrable on the interval [t, b], i.e., the definite integral
Z b
f (x) dx
t
exists, for each c < t ≤ b.
May 17, 2018 12:7 ws-book9x6 Integration for Calculus, Analysis, and Differential Equations ws-book9x6 page 139

Improper Integrals 139

If both improper integrals


Z c Z b
f (x) dx and f (x) dx (9.3)
a c

converge, the improper integral of f from a to b is said to converge or to be


convergent and its value is defined as the sum
Z b Z c Z b
f (x) dx = f (x) dx + f (x) dx.
a a c

If at least one of the improper integrals in (9.3) diverges, the improper


integral
Z b
f (x) dx
a

is said to diverge or to be divergent.

Examples 9.7 (Improper Integrals).


Evaluate the improper integral or show that it diverges.
Z 1
1
1. dx has unboundedness at 0 and diverges since
−1 x
Z 1
1
dx is a divergent type 2 p-integral with p = 1 ≥ 1.
0 x
Thus, the “solution”:
Z 1 1
1
dx = ln |x| = ln 1 − ln 1 = 0
−1 x −1
is incorrect.

Z 2
1
2. √ dx has unboundedness at 0 and converges since
−1
3
x
Z 0 Z t t
1 −1/3 3 2/3 3
(a) √ dx = lim x dx = lim x = t→0−lim [t2/3 − 1]
−1
3
x t→0− −1 t→0− 2 −1 2
3 3
= [0 − 1] = − is convergent,
2 2
Z 2 Z 2 2
1 −1/3 3 2/3 3
(b) √ dx = lim x dx = lim x lim [22/3 − t2/3 ]
= t→0+
0
3
x t→0+ t t→0+ 2 t 2
3 2/3 3 2/3
= [2 − 0] = 2 is convergent.
2 2
May 17, 2018 12:7 ws-book9x6 Integration for Calculus, Analysis, and Differential Equations ws-book9x6 page 140

140 Integration for Calculus, Analysis, and Differential Equations

Z 2 Z 0 Z 2
1 1 1 3 3 3
Hence, √ dx = √ dx+ √ dx = − + 22/3 = [22/3 −1].
−1
3
x −1
3
x 0
3
x 2 2 2

Remark on Using Newton-Leibniz Formula

As the example of the integral


Z 1
1
dx
−1 x

shows, in the case of a type 2 improper integral with unboundedness inside


the interval, the Newton-Leibniz Formula (Theorem 1.6) cannot be used
without first checking whether the integral converges. If it does, the formula
works just like for a proper definite integral.
Thus, in the prior example,
Z 2 Z 2 2
1 3 2/3 3 h 2/3 i
√ dx = x−1/3 dx = x = 2 −1 .
−1
3
x −1 2 −1 2

Geometric Interpretation

The value of a type 2 improper integral


Z b
f (x) dx,
a

when exists, finite or infinite, represents the net area of the region bounded
by the graph of f (x) and the x-axis on the interval [a, b].
When such a value does not exist, the net area is undefined.

If f (x) ≥ 0 on the interval, the value of the type 2 improper integral when
exists, finite or infinite, represents the area of the region bounded by the
graph of f (x) and the x-axis on the corresponding interval.

Remark 9.6. The analogue of the Comparison Test for type 1 improper
integral also holds for type 2 improper integral.
May 17, 2018 12:7 ws-book9x6 Integration for Calculus, Analysis, and Differential Equations ws-book9x6 page 141

Improper Integrals 141

9.3. Applications

Examples 9.8 (Applications).


1. Find the volume of “Gabriel’s horn”, i.e., the solid obtained by rotating
1
the region bounded by y = and the x-axis on the interval [1, ∞) about
x
the x-axis.
Solution: By the disk method ,
Z ∞  2 Z ∞
1 1
A= π dx = π 2 dx by the integration rules;
1 x 1 x
Z ∞ Z ∞
1 1
=π 2
dx since dx = 1 (see Examples 9.1);
1 x 1 x2
= π un.2 .

2. Find the surface area of the solid of revolution of the preceding problem.
Solution: By the area of the surface of revolution formula,
v "  #2 s
0 2
Z ∞ u Z ∞ 
1u 1 1 1
A= 2π t1+ dx = 2π 1 + − 2 dx
1 x x 1 x x
simplifying and using the integration rules;


x4 + 1
Z
= 2π dx = ∞.
1 x3

√ √
x4 + 1 x4 x2 1
Indeed, since 3
≥ 3
= 3 = on [1, ∞),
x x x x
Z ∞√ 4 Z ∞
x +1 1
3
dx ≥ dx = ∞,
1 x 1 x
the latter being a divergent type 1 p-integral with p = 1.
Remark 9.7. The two latter examples demonstrate a somewhat coun-
terintuitive fact that the surface area of a solid with a finite volume may
be infinite.
3. Find the volume of the solid generated by revolving the region bounded
by y = e−x and the x-axis on the interval [0, ∞) about the y-axis.
Solution: Since the rotation axis is perpendicular to the axis of defi-
nition, by the shell method ,
May 17, 2018 12:7 ws-book9x6 Integration for Calculus, Analysis, and Differential Equations ws-book9x6 page 142

142 Integration for Calculus, Analysis, and Differential Equations

Z ∞ Z t
−x
V = 2πxe dx = lim 2πxe−x dx by the integration rules;
0 t→∞ 0
Z t
= lim 2π xe−x dx
t→∞ 0
by the type 1 partial integration scenario,

u = x, du =R dx
;
dv = e dx, v = e−x dx = −e−x
−x

" t Z t # " t #
= lim 2π −xe−x + e−x dx = lim 2π −te−t − e−x

t→∞ 0 t→∞
0 0
−t −t
 
= lim 2π −te − e + 1 by the limit laws;
t→∞
h i
= 2π − lim te−t − lim e−t + 1 = 2π [−0 − 0 + 1] = 2π un.3 .
t→∞ t→∞

Indeed,
lim e−t = 0
t→∞

and
t n∞o
lim te−t = {∞ · 0} = lim =
t→∞ t→∞ et ∞
by L’Hôpital’s Rule (see, e.g., [1, 6]);
t0 1 1
= lim = lim t = = 0.
t→∞ [et ]0 t→∞ e ∞

9.4. Practice Problems

Evaluate the improper integral or show that it diverges.


Z ∞ Z ∞
1
1. dx 4. e−x cos x dx
2 x2 + x − 2 0
Z ∞ −1
1
Z
sin(π/x)
2. dx 5. dx
x ln x x2
Ze ∞ Z−∞

x 1
3. √ dx 6. dx
2
x +1
0 −∞ (x2 + x + 1)2
May 17, 2018 12:7 ws-book9x6 Integration for Calculus, Analysis, and Differential Equations ws-book9x6 page 143

Improper Integrals 143

Z 2 Z 3
1 1
7. √ dx 10. dx
1 x −1 1 (x − 2)2/3
Z 1 0
x2
Z
8. 3
dx 11. sec x dx
0 x −1 −π/2
Z 3 Z 1
1
9. √ dx 12. ln x dx
−3 9 − x2 0
b2530   International Strategic Relations and China’s National Security: World at the Crossroads

This page intentionally left blank

b2530_FM.indd 6 01-Sep-16 11:03:06 AM


May 17, 2018 12:7 ws-book9x6 Integration for Calculus, Analysis, and Differential Equations ws-book9x6 page 145

Mixed Integration Problems

Evaluate the integrals.


Z Z
1
1. √ dx 14. sin(ln x) dx
2
Z (x2 + 1) x + 1 Z
x −x+1 15. tan3 x dx
2. √ dx
3
x
Z (without Reduction Formulas)
3. (1 − 2x)9 dx
Z
Z 16. sin 3x sin 5x dx
1
4. dx
Z
1 + tan x 17. x cos2 x dx
Z r
1+x Z
5. dx 18. x cot2 x dx
Z 1−x
Z
6. x ln(x4 + 4) dx 19. cos4 x dx
Z
7. sin x cos(cos x) dx (without Reduction Formulas)
Z
1
20. dx
Z
sin(ln x) 2
8.
x
dx Z (sin x + cos x)
Z −√x 1
e 21. 4
dx
9. √ dx Z x +4
x x
Z 22. 4 + 3x2 + 2
dx
10. 2x
e cos 3x dx x
Z 2
cos x
Z 23. dx
11. x arctan x dx Z sin2x
sin x
ex 24. dx
Z
4
12. x
dx Z cos x
Z 1 − 3e3 25.
1
√ 3 dx
x (1 + x)
13. √ dx
x2 + 1
Z
1
26. p dx
x(2 − x)

145
May 17, 2018 12:7 ws-book9x6 Integration for Calculus, Analysis, and Differential Equations ws-book9x6 page 146

146 Integration for Calculus, Analysis, and Differential Equations

Z
1 Z √
27. dx x2 + 2x
3x x 29. dx
Z e −e x3
1 Z
1
28. √ x dx 30. √ dx
e +1 x x3 − 1
July 4, 2018 14:49 ws-book9x6 Integration for Calculus, Analysis, and Differential Equations ws-book9x6 page 147

Answer Key

Chapter 2: Direct Integration


2√

1. 5x − 6 + C 1 x − 1
5 16. ln +C
4 x + 3
1
2. − (2 − 3x)11 + C 1
17. √ arctan √
1 + 2x
+C
33
2 2 2
3. x3 + x2 − ln |x| + C x+2
1 18. arcsin +C
4. ex + + C 3
x 1 p
19. √ ln 3x − 1 + 9x2 − 6x − 3

1 2
5. − − 3 +C 3
x 3x +C
6. x2 + 3x + 4 ln |x − 1| + C
√ 20. tan(x) − sin(x) + C
2 3/2
7. x + 6 x − 3x − ln |x| + C 21. − cot x − tan x = −2 csc 2x + C
3
1
3 22. x − cos 2x + C
8. (2x − 1)4/3 (8x + 3) + C 2
112 1 1
3 5 23. x − sin(x + 2) + C
9. x − ln |2x + 3| + C 2 2
2 4 1 1
1 3 √ x 24. sin 2x + sin 8x + C
10. x − 2x + 2 2 arctan √ + C 4 16
3 2 x
25. − cot x + csc x + C = tan + C
11. x3 − 3x2 + 7x − 12 ln |x + 2| + C 2
4x 6x 9x 26. sgn(cos x + sin x)(sin x − cos x)
12. +2 + +C +C
ln 4 ln 6 ln 9
1 1
13. ln |x| − 4 + C 27.
4x 7
p 28. ln 4 − 2
14. arcsin x + ln(x + x2 + 1) + C 5
= arcsin x + arcsinh x + C 29.
6
1 1 π
15. (x + 4)3/2 − x3/2 + C 30. 1 −
6 6 4

147
May 17, 2018 12:7 ws-book9x6 Integration for Calculus, Analysis, and Differential Equations ws-book9x6 page 148

148 Integration for Calculus, Analysis, and Differential Equations

Chapter 3: Method of Substitution


1
p
1. 2 x2 − 3x + 4 + C 15. (1 − 3x2 )12
p 216
2. ln ex + e2x − 1 + C 1

− (1 − 3x2 )11 + C
1 198
3. arcsin2 x + C 1 x3
2 16. arctan +C
4. ln |ln x − 3| + C 6 2
1 4
p
17. ln x + x8 − 1 + C

5. ln |ln | ln x|| + C 4
1 1  1 2x + 1
6. ln(x2 + 2) + C 18. ln x2 + x + 1 − √ arctan √
2 2 3 3

+C

3 2 1 x − 5
7. ln x − 5 + √ ln
√ p
2 5 x + 5 19. − 1 − x2 − arcsin x + C
+C 2
cos 2x 20. −(1 − x2 )1/2 + (1 − x2 )3/2
8. − +C 3
4 1
− (1 − x2 )5/2 + C
1 1 5
9. tan2 x + C or sec2 x + C √
2 2

1 x2 + 1 − 1
3 21. ln √ +C
10. (sin x − cos x)2/3 + C 2 x2 + 1 + 1
2 1 tan x
√ √
1 22. √ arctan √ + C

11. − √ ln 2 cos x + cos 2x + C

2 2
2
1 3 23. cos(1/x) + C
12. ex + C √
3 ex + 1 − 1
24. ln √ x +C
2 √x e + 1 + 1
13. 5 +C
ln 5 π2
1 25.
14. − (1 − 2x6 )4/3 + C 32
16 26. ln(4/3)

Chapter 4: Method of Integration by Parts

1. −x3 e−x − 3x2 e−x − 6xe−x 1 1


7. − x cos 2x + sin 2x + C
− 6e−x + C 4 8
1
2. x2 sin x + 2x cos x − 2 sin x + C 8. x arctan x − ln(1 + x2 ) + C
2
3. x ln x − x + C 1 1
√ 9. x cos(ln x) + x sin(ln x) + C
x2 x2 − 1 2 2
4. arcsec x − sgn x +C 10. x(ln x)2 − 2x ln x + 2x + C
2 2
2 2x 3 2x 11. −x cot x + ln |sin x| + C
5. e sin 3x − e cos 3x + C
13 13 1 2 1 2
1 1 12. − x2 e−x − e−x + C
6. − ln x − + C 2 2
x x
May 17, 2018 12:7 ws-book9x6 Integration for Calculus, Analysis, and Differential Equations ws-book9x6 page 149

Answer Key 149

x2 16. −1/2
13. (arctan x)2 − x arctan x
2 1 1
1 1 17. eπ +
+ (arctan x)2 + ln(1 + x2 ) + C 2 2
2 2 8 3 4
√ 18. e +
1 1 + 1 − x2 9 9
14. − arcsin x − ln √
x x
19. 3/2 + π/12
+C √
20. π/9 + 3/3
15. 3 ln 3 − 2

Chapter 5: Trigonometric Integrals


1 1 3
1. − cos 7x − cos 3x + C + x+C
14 6 8
1 1 11. sec x + cos x + C
2. x+ sin 6x + C
2 12 3
12. 3 sin1/3 x − sin7/3 x + C
3. 3x − 4 cos x − sin 2x + C 7
3 1 1
4. x − sin 2x + sin 4x + C 13. − + ln |tan x| + C
2 8 2 tan2 x
5.
x

sin 4x
+C csc7 x csc5 x
8 32 14. − + +C
7 5
1 1 1 1
6. x− sin 4x + sin3 2x + C 15. sec4 x − tan2 x − ln |cos x| + C
16 64 48 4
cos3 x cos5 x 1
7. − + +C 16. ln |sec(ln x) + tan(ln x)|
3 5 2
3 5 1
sin x sin x + sec(ln x) tan(ln x) + C
8. − +C 2
3 5
8 π
1 6 17. √ +
9. − sin6 x cos x − sin4 x cos x 9 3 6
7 35
8 16 18. 1/3
− sin2 x cos x − cos x + C √
35 35 19. 2
1 3
10. cos3 x sin x + cos x sin x 1 · 3 · 5 · · · · · (2n − 1)
4 8 20. π
2 · 4 · 6 · · · · · 2n

Chapter 6: Trigonometric Substitutions



1. 3/12 1 p
√ 5. − (3x4 + 4x2 + 8) 1 − x2
15
π 2− 3 +C
2. +
72 24 1 p
p
6. x x2 + 2 − ln x + x2 + 2

3. π/12 2
√ +C
2 1 √
4. + ln( 2 + 1) 1 p 9
p
7. x x2 − 9 + ln x + x2 − 9

2 2
2 2
+C
May 17, 2018 12:7 ws-book9x6 Integration for Calculus, Analysis, and Differential Equations ws-book9x6 page 150

150 Integration for Calculus, Analysis, and Differential Equations

√ √
9 − 4x2 2 x2 + 2x − 5
8. − − 2 arcsin x + C 12. +C
x 3 6(x + 1)
√ √
9 3x 9x2 − 16 1 x2 ( x4 + 1 + x2 )
9. sgn x arcsec − 13. ln √ +C
8 4 2x2 2 x4 + 1 + 1
+C
1 9 2x − 1
10. − arctan √ 15. arcsin
x2 −2 8 3p
√ 1
x2 − 2 + (2x − 1) 2 + x − x2 + C
+ arctan +C 4
x 1 2 p
1p 2 16. (x + 1) x4 + 2x2 − 1
11. x + 2x + 10(x + 1) 4
2 1 p
9 p 2
− ln(x2 + 1 + x4 + 2x2 − 1)
− ln x + 2x + 10 + x + 1
2
2 +C
+C

Chapter 7: Integration of Rational Functions


1 13

3 − 1/16 − 1/16
x + 1
1. ln |x + 2| + ln |x − 5| + C + ln
7 7 16 x − 1 x − 1 x+1
2 1 +C
2. ln |x + 2| + ln |x − 1| + C
3 3 1
8. x − 2 ln |x + 1| − +C
1 x2 + 1 x+1
3. ln +C
6 x2 + 4 1 3
9. − −
1 1 96(x − 1)96 97(x − 1)97
ln |x + 1| − ln x2 − x + 1

4. 3 1
3 6 − − +C
1 2x − 1 98(x − 1)98 99(x − 1)99
+ √ arctan √ +C
3 3 1 4 1
1 9 10. x + ln(x4 + 1) − ln(x4 + 2) + C
5. x + ln |x| − ln |x − 2| 4  4
6 2

1 1
+
28
ln |x − 3| + C 11. ln x10 − ln |x10 − 1| − 10
3 10 x −1
4 +C
6. 4 ln |x + 2| − − 4 ln |x + 1| 2
x+2 12. ln |x| − ln |x7 + 1| + C

1
+C 7
1 x5 + 5x

x+1 13. ln 5 +C
3 x 5 x + 5x + 1
7. arctan x +
8 8(x2 + 1) 1 x2 − 1
14. √ arctan √ +C
5 5x
May 17, 2018 12:7 ws-book9x6 Integration for Calculus, Analysis, and Differential Equations ws-book9x6 page 151

Answer Key 151

Chapter 8: Rationalizing Substitutions


1 1 1
1. (2x + 1)3/2 + (2x + 1)1/2 + C 8. ln(e2x + 1) + 2 arctan ex + C
6 2 2
1 2/3 1
2. (3x − 1) − (3x − 1)1/3 9. tan3 x − tan x + x + C
2 3
+ ln (3x − 1)1/3 + 1 + C 1

10. tan3 x + C
r 3
x  
3. ln − 1 1 x 1
x + 2 11. arctan tan − x +C

2 2
r
x tan
− ln + 1 2
xr+2  
1 1 x
x 12. arctan tan +C
+ (x + 2) +C 2 2 2
x+2
1 2
x
4. (x + 6)3/2 − 6(x2 + 6)1/2 + C 2
1 + 2 tan
3 13. ln r 2
√ √

5. x − 2 x + 2 ln( x + 1) + C 5 x 2 x
4 + 6 tan 2 − 4 tan 2

6. 6x1/6 − 3x1/3 + 2x1/2 − 6 ln(x1/6 +C
+ 1) + C 1

3 x 1

1 14. √ arctan √ tan + √
7. e2x + 2ex + 4 ln |ex − 2| + C 5 5 2 5
2 +C

Chapter 9: Improper Integrals


2 7. 2
1. ln 2
3 8. The integral diverges.
2. The integral diverges.
9. π
3. The integral diverges.
10. 6
4. 1/2
11. The integral diverges.
5. −2/π
√ 12. −1
4 3
6. π
9

Mixed Integration Problems


√ √
3 8/3 3 5/3 3 2/3

1 2 x2 + 1 − x + 1
1. − √ ln 2. x − x + x +C
2 x+1 8 5 2
+C 1
√ √ 3. − (1 − 2x)10 + C
1 2 x2 + 1 + x − 1 20
= √ ln
1
2 x+1
4. ln |tan x + 1|
+C 2
May 17, 2018 12:7 ws-book9x6 Integration for Calculus, Analysis, and Differential Equations ws-book9x6 page 152

152 Integration for Calculus, Analysis, and Differential Equations

1  1 x2
− ln tan2 x + 1 + x + C 18. − − x cot x + ln | sin x| + C
4 2 2
1 1
= x + ln |cos x + sin x| + C 3x sin 2x sin 4x
2 2 19. + + +C
r ! 8 4 32
x+1 1
5. 2 arctan 20. − +C
1−x tan x + 1
r 1
− (1 − x)
x+1
+C 21. − ln(x2 − 2x + 2)
1−x 16
1
2 + arctan(x − 1)
x x2 8
6. ln(x4 + 4) − x2 + 2 arctan 1
2 2 + ln(x2 + 2x + 2)
+C 16
1
7. − sin (cos x) + C + arctan(x + 1) + C
8
8. − cos(ln x) + C 1
ln(x2 + 1) − ln(x2 + 2) + C

√ 22.
9. −2e− x + C 2
23. ln | csc x − cot x| + cos x + C
e2x
10. [2 cos 3x + 3 sin 3x] + C tan3 x
13 24. +C
x2 1 1 3
11. arctan x − x + arctan x + C 2 1
2 2 2 25. − √ + √ +C
1 1+ x (1 + x)2
12. − ln |1 − 3ex | + C 26. arcsin(x − 1) + C
3
1 ex − 1

(x2 + 1)3/2 p 2
13. − x +1+C 27. ln x + e−x + C
3 2 e + 1
x √ x
14. [sin(ln x) − cos(ln x)] + C e +1−1
2 28. ln √ x +C
e +1+1
tan2 x
15. − ln | sec x| + C sgn x

2
3/2
2 29. − 1+ +C
1 1 3 x
16. sin 2x − sin 8x + C
4 16 2 p
1 1 1 30. arctan x3 − 1 + C
17. x2 + x sin 2x + cos 2x + C 3
4 4 8
May 17, 2018 12:7 ws-book9x6 Integration for Calculus, Analysis, and Differential Equations ws-book9x6 page 153

Appendix A

Table of Basic Integrals

Z
1. 0 dx = C on (−∞, ∞).

xn+1
Z
2. xn dx = + C (n 6= −1)
n+1
on interval(s) depending on the exponent n.
Z
1
3. dx = ln |x| + C on each of the intervals (−∞, 0), (0, ∞).
x
ax
Z
4. ax dx = + C (a > 0, a 6= 1) on (−∞, ∞).
ln a
Z
In particular, ex dx = ex + C on (−∞, ∞).
Z
5. sin x dx = − cos x + C on (−∞, ∞).
Z
6. cos x dx = sin x + C on (−∞, ∞).
Z
7. sec2 x dx = tan x + C on each of the intervals

(−π/2 + nπ, π/2 + nπ), n ∈ Z := {0, ±1, ±2, . . . } .


Z
8. csc2 x dx = − cot x + C on each of the intervals (nπ, π + nπ), n ∈ Z.
Z
9. sec x tan x dx = sec x + C on the same intervals as in 7.
Z
10. csc x cot x dx = − csc x + C on the same intervals as in 8.
Z
11. tan x dx = ln | sec x| + C = − ln | cos x| + C

153
May 17, 2018 12:7 ws-book9x6 Integration for Calculus, Analysis, and Differential Equations ws-book9x6 page 154

154 Integration for Calculus, Analysis, and Differential Equations

on the same intervals as in 7.


Z
12. cot x dx = ln | sin x| + C on the same intervals as in 8.
Z
13. sec x dx = ln | sec x + tan x| + C on the same intervals as in 7.
Z
14. csc x dx = − ln | csc x + cot x| + C = ln | csc x − cot x| + C

on the same intervals as in 8.


Z
15. sinh x dx = cosh x + C on (−∞, ∞).
Z
16. cosh x dx = sinh x + C on (−∞, ∞).

For a > 0,
Z
1 1 x
17. dx = arctan + C on (−∞, ∞).
x2 + a2 a a

x − a
Z
1 1
18. (“tall logarithm” ) dx = ln +C
x2 − a2 2a x + a
on each of the intervals (−∞, −a), (−a, a), (a, ∞).
Z
1 x
19. √ dx = arcsin + C on (−a, a).
a2 − x2 a
Z
1 p
20. (“long logarithm” ) √ dx = ln |x + x2 ± a2 | + C
x2 ± a2
on (−∞, ∞) for “+”
and on each of the intervals (−∞, −a), (a, ∞) for “−”.
Z
1 1 x
21. √ dx = arcsec + C

2
x x −a 2 a a
on each of the intervals (−∞, −a), (a, ∞).
May 17, 2018 12:7 ws-book9x6 Integration for Calculus, Analysis, and Differential Equations ws-book9x6 page 155

Appendix B

Reduction Formulas

For n = 2, 3, . . . ,
cosn−1 x sin x n − 1
Z Z
1. cosn x dx = + cosn−2 x dx
n n
sinn−1 x cos x n − 1
Z Z
2. sinn x dx = − + sinn−2 x dx
n n
secn−2 x tan x n − 2
Z Z
3. secn x dx = + secn−2 x dx
n−1 n−1
cscn−2 x cot x n − 2
Z Z
4. cscn x dx = − + cscn−2 x dx
n−1 n−1
tann−1 x
Z Z
5. tann x dx = − tann−2 x dx
n−1
Z n−1 Z
n cot x
6. cot x dx = − − cotn−2 x dx
n−1

Z Z
7. For n = 1, 2, . . . , lnn x dx = x lnn x − n lnn−1 x dx

For a 6= 0 and n = 1, 2, . . . ,
xn eax
Z Z
n
8. xn eax dx = − xn−1 eax dx
a a
xn sin ax n
Z Z
n
9. x cos ax dx = − xn−1 sin ax dx
a a
xn cos ax n
Z Z
10. xn sin ax dx = − + xn−1 cos ax dx
a a

155
May 17, 2018 12:7 ws-book9x6 Integration for Calculus, Analysis, and Differential Equations ws-book9x6 page 156

156 Integration for Calculus, Analysis, and Differential Equations

For a > 0 and n = 2, 3, . . . ,


Z
1
11. n dx
(x + a2 )
2
" Z #
1 x 1
= + (2n − 3) n−1 dx
(2n − 2)a2 (x2 + a2 )n−1 (x2 + a2 )
May 17, 2018 12:7 ws-book9x6 Integration for Calculus, Analysis, and Differential Equations ws-book9x6 page 157

Appendix C

Basic Identities of Algebra and


Trigonometry

Algebra
Binomial Formula
For n = 1, 2, 3, . . . ,
n      
X n n−k k n n−1 n
(A±B)n = A B = An + A B+· · ·+ AB n−1 +B n ,
k 1 n−1
k=0
 
n n!
where := , k = 0, 1, . . . , n.
k k!(n − k)!

In particular,

(A ± B)2 = A2 ± 2AB + B 2 (Perfect Square)


3 3 2 2 3
(A ± B) = A ± 3A B + 3AB ± B (Perfect Cube)

Factoring Formula
For n = 1, 2, 3, . . . ,
n−1
X
An − B n = (A − B) An−1−k B k
k=0
n−1 n−2
= (A − B)(A +A B + · · · + AB n−2 + B n−1 ).
In particular,

A2 − B 2 = (A − B)(A + B) (Difference of Squares)


3 3 2 2
A ± B = (A ± B)(A ∓ AB + B ) (Sum/Difference of Cubes)

157
May 17, 2018 12:7 ws-book9x6 Integration for Calculus, Analysis, and Differential Equations ws-book9x6 page 158

158 Integration for Calculus, Analysis, and Differential Equations

Laws of Exponents
1. Am An = Am+n (Product Rule)
m
A
2. = Am−n (Quotient Rule)
An
n
3. (Am ) = Amn (Power Rule)
n n n
4. (AB) = A B (Power of Product Rule)
 n
A An
5. = n (Power of Quotient Rule)
B B

Laws of Logarithms
1. loga (AB) = loga A + loga B (Product Rule)
A
2. loga = loga A − loga B (Quotient Rule)
B
3. loga Ac = c loga A (Power Rule)

Change of Base Formula

loga A
logb A =
loga b
1
In particular, logb a = .
loga b

Trigonometry
Reciprocal Identities

1 1 1
sin θ = cos θ = tan θ =
csc θ sec θ cot θ
1 1 1
csc θ = sec θ = cot θ =
sin θ cos θ tan θ
May 17, 2018 12:7 ws-book9x6 Integration for Calculus, Analysis, and Differential Equations ws-book9x6 page 159

Basic Identities of Algebra and Trigonometry 159

Quotient Identities

sin θ cos θ
tan θ = cot θ =
cos θ sin θ

Pythagorean Identities

sin2 θ + cos2 θ = 1 tan2 θ + 1 = sec2 θ cot2 θ + 1 = csc2 θ

Even-Odd Identities

sin(−θ) = − sin θ cos(−θ) = cos θ tan(−θ) = − tan θ

csc(−θ) = − csc θ sec(−θ) = sec θ cot(−θ) = − cot θ

Double-Angle Identities

sin 2θ = 2 sin θ cos θ


cos 2θ = cos2 θ − sin2 θ = 2 cos2 θ − 1 = 1 − 2 sin2 θ

Power Reduction/Half-Angle Identities

1 − cos 2θ 1 + cos 2θ 1 − cos 2θ


sin2 θ = cos2 θ = tan2 θ =
2 2 1 + cos 2θ

Product-to-Sum Identities
1
cos α cos β = [cos(α − β) + cos(α + β)]
2
1
sin α sin β = [cos(α − β) − cos(α + β)]
2
1
sin α cos β = [sin(α − β) + sin(α + β)]
2
b2530   International Strategic Relations and China’s National Security: World at the Crossroads

This page intentionally left blank

b2530_FM.indd 6 01-Sep-16 11:03:06 AM


May 17, 2018 12:7 ws-book9x6 Integration for Calculus, Analysis, and Differential Equations ws-book9x6 page 161

Bibliography

[1] W. Briggs, L. Cochran, B. Gillett, et al., Calculus: Early Transcen-


dentals, 2nd ed., Pearson Education, Inc., Boston, 2015.
[2] V.F. Butuzov, N.Ch. Krutitskaya, G.N. Medvedev, and A.A. Shishkin,
Mathematical Analysis in Questions and Problems, 4th ed., Fizmatlit,
Moscow, 2001 (Russian).
[3] B.P. Demidovich, Collection of Problems and Exercises in Mathe-
matical Analysis, 13th ed., Publishing House of Moscow University,
Moscow, 1997 (Russian).
[4] B.R. Gelbaum and J.M.H. Olmsted, Counterexamples in Analysis,
Dover Publications, Inc., Mineola, New York, 2003.
[5] V.P. Minorsky, Collection of Problems in Higher Mathematics, Fiz-
matlit, Moscow, 2006 (Russian).
[6] J. Stewart, Calculus, 3rd ed., Brooks/Cole Publishing Co., Pacific
Grove, California, 1995.

161
b2530   International Strategic Relations and China’s National Security: World at the Crossroads

This page intentionally left blank

b2530_FM.indd 6 01-Sep-16 11:03:06 AM


May 17, 2018 12:7 ws-book9x6 Integration for Calculus, Analysis, and Differential Equations ws-book9x6 page 163

Index

antiderivative, 1 integrable function, 5, 8


arc length formula, 80 Integral Mean Value Theorem, 11
area function, 12 integrals of even and odd functions,
area of the surface of revolution 16
formula, 141 integration by definition, 10
avoiding the universal substitution, Integration by Parts Formula for
120, 121 Definite Integral, 59
Integration by Parts Formula for
back substitution, 36, 47 Indefinite Integral, 51
integration rules for definite integral,
changing integration limits, 46, 47 9
Comparison Test, 133 integration rules for indefinite
convergent improper integral, 128, integral, 5
131, 132, 135, 137, 139
more general use of integral notation,
definite integral, 8 9
direct integration, 20, 29
disk method, 79, 141 Newton-Leibniz Formula, 13
divergent improper integral, 128, 131,
132, 135, 137, 139 Partial Fraction Decomposition, 105,
106
elementary function, 125 Partial Fraction Method, 107, 108,
110
Fundamental Theorem of Calculus partial fractions, 100
(Part 1), 13 Partial Integration Formula, 52
Fundamental Theorem of Calculus perfect substitution, 36
(Part 2), 13 piecewise continuous function, 9
Properties of Definite Integral, 9
improper integral, 127 Properties of Indefinite Integral, 5
indefinite integral, 2
initial value problem, 31 rational function, 99

163
May 17, 2018 12:7 ws-book9x6 Integration for Calculus, Analysis, and Differential Equations ws-book9x6 page 164

164 Integration for Calculus, Analysis, and Differential Equations

rationalizing substitution, 115 Total Change Theorem, 15


reduction formula, 57 trigonometric integrals, 65, 118
Reduction Formulas for trigonometric substitutions, 84
Trigonometric Integrals, 66 trivial substitution, 39
requirements to reverse substitution, type 1 p-integrals, 129
84 type 1 improper integrals, 127
reverse substitutions, 83 type 1 integrals, 52
Riemann sum, 7 type 2 p-integrals, 136
type 2 improper integrals, 134
shell method, 32, 48, 63, 113, 122, 141 type 2 integrals, 53
Substitution Rule for Definite type 3 integrals, 54
Integral, 45
Substitution Rule for Indefinite universal (Weierstrass) substitution,
Integral, 35 119
Useful Integration Formula, 18
table of gasic integrals, 17
three special types of integrals, 52 washer method, 32, 112

You might also like